100% found this document useful (3 votes)
5K views203 pages

NBME 12 - Questions

A 49-year-old man presents with dark urine and weight loss. Examination reveals jaundice and a mass in the abdomen. Imaging shows a 5 cm pancreatic head mass compressing veins and arteries. Biopsy of the mass shows malignant cells. The factor most predictive of a poor prognosis is the tumor encasing the superior mesenteric artery.

Uploaded by

1140271
Copyright
© © All Rights Reserved
We take content rights seriously. If you suspect this is your content, claim it here.
Available Formats
Download as PDF, TXT or read online on Scribd
100% found this document useful (3 votes)
5K views203 pages

NBME 12 - Questions

A 49-year-old man presents with dark urine and weight loss. Examination reveals jaundice and a mass in the abdomen. Imaging shows a 5 cm pancreatic head mass compressing veins and arteries. Biopsy of the mass shows malignant cells. The factor most predictive of a poor prognosis is the tumor encasing the superior mesenteric artery.

Uploaded by

1140271
Copyright
© © All Rights Reserved
We take content rights seriously. If you suspect this is your content, claim it here.
Available Formats
Download as PDF, TXT or read online on Scribd
You are on page 1/ 203

Exam Section 1: ltem 1 of 50 National Board of Medical Examiners Time Remaining.

an Comprehensive Clinical Science Self-Assessment 4 hr 59 min 52 sec

1. A57-year-old man comes lo the physician because of a 1-year history of low back pain that radiates lo the bullocks when he walks. He has not had leg pain or pain al rest. The pain is relieved within 1
minute when he stops walking. He has a 1-year history of erectile dysfunction. He has smoked one pack of cigarettes daily for 30 years. His pulse is 85/min, and blood pressure is 160/100 mm Hg. Flexion
of either hip with the knee extended does not cause pain. Pedal pulses are decreased. Sensation is intact. His hematocrit is 40%, erythrocyte sedimentation rate is 20 mm/h, and serum glucose
concentration is 180 mg/dL. Which of the following is the most likely diagnosis?

0 A) Aortoiliac occlusion
0 B) Dissecting aortic aneurysm
0 C) Herniated nucleus pulposus
0 D) Lumbar discitis
0 E) Pyelonephritis
0 F) Spinal stenosis

https://t.me/USMLENBME2CK ti
Next Lab Values Calculator Review Help Pause
Exam Section 1: Item 2 of 50 National Board of Medical Examiners Time Remaining.
an Comprehensive Clinical Science Self-Assessment 4 hr 59 min 45 sec

2. A 60-year-old female store clerk comes lo the office for an annual health maintenance examination. Her medical history is unremarkable except for gastroesophageal reflux disease, for which she takes
ranitidine at bedtime with good relief. Her last menstrual period was 6 years ago. Mammogram 5 years ago and Pap smear 2 years ago were normal. She does not exercise regularly. She eats a regular
diet. She has smoked one pack of cigarettes daily for 40 years and drinks one to two glasses of wine weekly. Her family history is significant only for diet-controlled diabetes mellitus in her mother. The
patient is 157 cm (5 ft 2 in) tall and weighs 68 kg (150 lb); BMI is 27 kg/m?. Her waist-to-hip ratio is 1.05. Cardiopulmonary, abdominal, and genitourinary examinations are normal for her age and sex
assigned at birth. This patient's greatest risk of mortality at this time is from which of the following?

0 A) Breast cancer
0 B) Cardiovascular disease
0 C) Cervical cancer
0 D) Hip fracture
0 E) lung cancer

https://t.me/USMLENBME2CK
ts e « t
Previous Next Lab Values Calculator Review Help pause
Exam Section 1: ltem 3 of 50 National Board of Medical Examiners Time Remaining.
an Comprehensive Clinical Science Self-Assessment 4 hr 59 min 38 sec

3. A36-year-old man is brought to the emergency department after he developed chest pain and dyspnea while attempting a 10K run. He has had mild asthma for several years but he takes no medications
for it. He says that he has developed a nonproductive cough recently. He has not had fever, but he has noticed a decrease in his appetite and a 2.5-kg (5-lb) weight loss during the past 6 weeks. Vital
signs are pulse 100/min, respirations 26/min, and blood pressure 100/72 mm Hg. Pulse oximetry on room air shows an oxygen saturation of 92%. Breath sounds are diminished on the right and there is
increased resonance on percussion. Chest x-ray is shown. Which of the following is the most appropriate treatment at this time?

0 A) Endotracheal intubation
0 B) Nebulized albuterol
0 C) Observation and administration of nasal oxygen
p Tharaootom

https://t.me/USMLENBME2CK ti
Previous Next Lab Values Calculator Review Help pause
3. A36-year-old man is brought to the emergency department after he developed chest pain and dyspnea while attempting a 10K run. He has had mild asthma for several years but he takes no medications
for it. He says that he has developed a nonproductive cough recently. He has not had fever, but he has noticed a decrease in his appetite and a 2.5-kg (5-lb) weight loss during the past 6 weeks. Vital
signs are pulse 100/min, respirations 26/min, and blood pressure 100/72 mm Hg. Pulse oximetry on room air shows an oxygen saturation of 92%. Breath sounds are diminished on the right and there is
increased resonance on percussion. Chest x-ray is shown. Which of the following is the most appropriate treatment at this time?

0 A) Endotracheal intubation
0 B) Nebulized albuterol
0 C) Observation and administration of nasal oxygen
0 D) Thoracotomy
0 E) Tube thoracostomy

https://t.me/USMLENBME2CK ti
Previous Next Lab Values Calculator Review Help pause
Exam Section 1: ltem 4 of 50 National Board of Medical Examiners Time Remaining.
an Comprehensive Clinical Science Self-Assessment 4 hr 59 min 34 sec

4. A 15-year-old girl is brought to the emergency department by her mother because of a 2-week history of heavy vaginal bleeding. Menarche was at the age of 14 years. Menses occur at irregular 21- to
35-day intervals and last 10 days with moderate flow. She has never been sexually active. She is not in distress. She is at the 55th percentile for height, 70th percentile for weight, and 60th percentile for
BMI. Her temperature is 38°C (100.4F), and blood pressure is 90/55 mm Hg. Examination shows pallor. Laboratory studies show a hemoglobin concentration of 9 g/dL, leukocyte count of 5500/mm3, and
platelet count of 15,000/mm3. Which of the following is the most likely diagnosis?

0 A) Cervical ectropion
0 B) Endometrial polyp
0 C) Immune thrombocytopenic purpura
0 D) Submucous myoma
0 E) Thrombotic thrombocytopenic purpura

https://t.me/USMLENBME2CK ti e
Previous Next Lab Values Calculator Review Help pause
Exam Section 1: Item 5 of 50 National Board of Medical Examiners Time Remaining.
an Comprehensive Clinical Science Self-Assessment 4 hr 59 min 28 sec

5. A49-year-old man comes lo the office because of a 2-week history of dark urine. He has had a 5-kg (11-lb) weight loss during the past month despite no change in appetite. He has no history of serious
illness and takes no medications. His temperature is 37.5°C (99.5F), pulse is 80/min, respirations are 12/min, and blood pressure is 132/86 mm Hg. Examination shows conjunctival icterus. A 5-cm,
nontender mass is palpated in the right upper quadrant of the abdomen. Serum studies show a total bitirubin concentration of 6 mg/dl, with a direct component of 5.2 mg/dl, and a lactate dehydrogenase
activity of 480 U/L. Abdominal ultrasonography shows an enlarged gallbladder with no calculi, a 2-cm-wide common bile duct with no calculi that tapers at the distal end, and a 5-cm mass in the head of the
pancreas. A CT scan of the abdomen shows a 5-cm mass in the head of the pancreas compressing the superior mesenteric vein and partially encasing the superior mesenteric artery. Results of a
fine-needle biopsy show hyperchromatic, pleomorphic cells with a high nucleus-to-cytoplasm ratio. Further diagnostic studies show no evidence of metastatic disease. Which of the following factors is most
predictive of a poor prognosis in this patient?

0 A) Bilirubin concentration
0 8) Patient's age
0 C) Patient's sex assigned at birth
0 D) Tumor encasement of the superior mesenteric vessels
0 E) Tumor location in the head of the pancreas rather than the tail

https://t.me/USMLENBME2CK ti
Previous Next Lab Values Calculator Review Help pause
Exam Section 1: Item 6 of 50 National Board of Medical Examiners Time Remaining.
an Comprehensive Clinical Science Self-Assessment 4 hr 59 min 26 sec

6. A 60-year-old man comes lo the emergency department 6 hours after the onset of shortness of breath associated with a nonproductive cough. One year ago, he was diagnosed with mild hypertension. His
pulse is 100/min, respirations are 26/min, and blood pressure is 140/105 mm Hg. Ophthalmoscopic examination shows arteriolar narrowing but no papilledema. There is no peripheral edema. An S, S, and
crackles al the bases of both lungs are heard, and jugular veins are distended. Which of the following is the most likely diagnosis?

0 A) Cardiac amyloidosis
0 B) Chronic aortic dissection
0 C) Congestive heart failure
0 D) Constrictive pericarditis
0 E) Renal artery stenosis

https://t.me/USMLENBME2CK ti
Previous Next Lab Values Calculator Review Help pause
Exam Section 1: Item 7 of 50 National Board of Medical Examiners Time Remaining.
an Comprehensive Clinical Science Self-Assessment 4 hr 59 min 24 sec

7. Immediately after delivery lo a 17-year-old primigravid patient, a 500-g (1-lb 2-oz) male newborn fails to breathe spontaneously and has a pulse of 30/min. The mother had no prenatal care and says that
her last menstrual period was 2 or 3 months ago. Examination of the newborn shows edematous hands and feet and dark red skin without lanugo. The eyelids are fused. He is hypotonic. All four extremities
are in extension. The nipples are barely visible, and the testes are undescended. After several minutes of bag-valve-mask ventilation with 100% oxygen and chest compressions, the pulse falls below
30/min, and there are still no spontaneous respirations. Which of the following is the most appropriate next step in management?

0 A) Continued bag-valve-mask ventilation and chest compressions until the pulse is absent
0 B) Administration of surfactant
0 C) Endotracheal intubation and continued bag-valve ventilation with 100% oxygen
0 D) Placement of an umbilical venous catheter for infusion of epinephrine
0 E) Discontinuation of resuscitative efforts

https://t.me/USMLENBME2CK ti
Previous Next Lab Values Calculator Review Help pause
Exam Section 1: Item 8 of 50 National Board of Medical Examiners Time Remaining.
an Comprehensive Clinical Science Self-Assessment 4 hr 59 min 22 sec

8. Fifteen days after sustaining third-degree bums of the abdomen and lower extremities, a hospitalized 67-year-old man has fever. He is intubated and mechanically ventilated. He is receiving cefazolin,
lorazepam, and atenolol. His temperature is 38.9C (102F), pulse is 90/min, and blood pressure is 110/70 mm Hg. A small amount of cloudy fluid is draining around the nasogastric tube. The burn wounds
are clean. The lungs are clear to auscultation. The abdomen is soft. Rectal examination shows no abnormalities. Femoral pulses are intact. Laboratory studies show a hematocrit of 35% and leukocyte
count of 15,000/mm. A chest x-ray shows no abnormalities; endotracheal and nasogastric tubes are in proper position. Which of the following is the most appropriate next step in diagnosis?

0 A) CT angiography of the chest


0 B) CT scan of the abdomen and pelvis
0 C) CT scan of the sinuses
0 D) MRI of the anorectal region
OE MRI of the brain

https://t.me/USMLENBME2CK ti
Previous Next Lab Values Calculator Review Help pause
Exam Section 1: Item 9 of 50 National Board of Medical Examiners Time Remaining.
an Comprehensive Clinical Science Self-Assessment 4 hr 59 min 20 sec

9. A 13-month-old boy is brought lo the physician for a follow-up examination. He completed a 10-day course of amoxicillin for an episode of pneumonia 10 days ago. He has had four episodes of pneumonia
since birth and had severe chickenpox at the age of 8 months. He is below the 5th percentile for length and weight. His temperature is 37.4°C (99.3°F), pulse is 120/min, respirations are 30/min, and blood
pressure is 94/50 mm Hg. Examination shows no abnormalities except for oral thrush. Laboratory studies show a leukocyte count of 1500/mm3 (50% segmented neutrophils, 8% eosinophils, 1% basophils,
35% lymphocytes, and 6% monocytes), a CD4+ T-lymphocyte count of 225/mm3 (Normal>SOO), and a low lymphocyte proliferative response. Which of the following is the most likely diagnosis?

0 A) Combined immunodeficiency
0 B) Cystic fibrosis
0 C) Hypogammaglobulinemia
0 D) Thymic-parathyroid dysplasia (DiGeorge syndrome)
0 E) X-linked agammaglobulinemia

https://t.me/USMLENBME2CK ti
Previous Next Lab Values Calculator Review Help pause
Exam Section 1: ltem 10 of 50 National Board of Medical Examiners Time Remaining.
an Comprehensive Clinical Science Self-Assessment 4 hr 59 min 18 sec

1 0. Twenty-eight hours after undergoing uncomplicated elective right total knee replacement, a 76-year-old woman is alert and fully oriented but has nausea and pain in her right hip. Eight hours later, she is
found unresponsive. She has hypertension. Her preadmission medication is lisinopril. Promethazine, hydromorphone, and enoxaparin were added to the regimen postoperatively. Her temperature is 37°C
(98.6°F), pulse is 90/min, respirations are 6/min, and blood pressure is 90/50 mm Hg. Physical examination shows a clean surgical site with no drainage or bleeding. On neurologic examination, she does
not respond to auditory stimuli. She responds to painful stimuli with a faint grimace. The eyes are closed; when the physician manually opens the patient's eyes, the pupils are 3 mm. There are no
spontaneous movements of the extremities. The extremities do not move with painful stimuli. Which of the following is most likely lo show the cause of this patient's change in level of consciousness?

0 A) Arterial blood gas analysis


0 B) Complete blood count
0 C) Complete metabolic profile
0 D) CT scan of the head
0 E) Pulmonary CT angiography

https://t.me/USMLENBME2CK ti
Previous Next Lab Values Calculator Review Help pause
Exam Section 1: Item 11 of 50 National Board of Medical Examiners Time Remaining.
an Comprehensive Clinical Science Self-Assessment 4 hr 59 min 16 sec

11. A 78-year-old woman comes to the physician because of a 2-week history of mild right knee pain, especially when she walks up stairs. During this lime, she also has had stiffness in her knee in the
morning. She enjoys walking around her neighborhood but has been unable to walk because of the pain. She has gastroesophageal reflux disease and takes omeprazole occasionally. Vital signs are within
normal limits. On examination, active and passive range of motion of the right knee is full; there is moderate crepitus and no effusion. Which of the following is the most appropriate pharmacotherapy?

0 A) Acetaminophen
0 B) Prednisone
0 C) Colchicine
0 D) Ibuprofen
0 E) Ciprofloxacin

https://t.me/USMLENBME2CK ti
Previous Next Lab Values Calculator Review Help pause
Exam Section 1: Item 12 0f 50 National Board of Medical Examiners@ Time Remaining:
i ark Comprehensive Clinical Science Self-Assessment 4hr59min 13sec

12. A meta-analysis of nine small, randomized-controlled trials is performed to evaluate the efficacy of inhaled corticosteroid therapy in patients with a persistent cough after an
upper respiratory tract infection. Results of one of the trials show a statistically significant benefit with an average reduction in cough of 5 days in the treated group as
compared with the control group. Two other studies show p-values for benefit of 0.07. The results of the studies are shown. Which of the following is the strongest rationale
for performing a meta-analysis?

0 A) Decrease confounding
0 B) Decrease selection bias
0 C) Decrease type I error
0 D) Increase the confidence interval
0 E) Increase the power

20 10 0 -10
Reduction of cough
(Days)

https://t.me/USMLENBME2CK ts e « t
Previous Next Lab Values Calculator Review Help pause
Exam Section 1: ltem 13 of 50 National Board of Medical Examiners Time Remaining:
an Comprehensive Clinical Science Self-Assessment 4 hr 59 min 11 sec

13. A 63-year-old woman comes to the office for an examination before undergoing ventral hernia repair. She has a 3-month history of increasingly severe abdominal cramps and low back pain, which are
worse at night, and heartburn, which typically occurs after she pushes her grandson on a swing. She avoids most physical activity because of these symptoms. She has type 2 diabetes mellitus treated
with metformin. Fifteen years ago, she underwent total abdominal hysterectomy for dysfunctional uterine bleeding. She smoked one pack of cigarettes daily for 8 years but quit 35 years ago. She is 160 cm
(5 ft 3 in) tall and weighs 72 kg (160 lb); BMI is 28 kg/m2. Vital signs are within normal limits. Cardiopulmonary examination shows no abnormalities. There is truncal obesity. A 10x 10-cm, easily reducible
mass is palpable under the midline scar. Which of the following is the most appropriate next step in management?

0 A) Cardiac stress scinligraphy


0 B) Pulmonary function testing
0 C) Recommendation for a low-sodium, low-fat diet
0 D) Recommendation to wear an abdominal binder
0 E) Switching from metformin to insulin

https://t.me/USMLENBME2CK ti e
Previous Next Lab Values Calculator Review Help pause
Exam Section 1: Item 14 of 50 National Board of Medical Examiners Time Remaining.
an Comprehensive Clinical Science Self-Assessment 4 hr 59 min 8 sec

14. Physician A is approaching the end of his shift at the outpatient clinic when he hears the sound of glass breaking in Physician B's office. When Physician A enters Physician B's office to ensure that she is
OK, he sees her cleaning up pieces of a wine glass. Physician B says she bought a bottle of wine earlier today and initially intended to bring it home, but she decided to open it and drink "a little" because
she was stressed following a meeting with the department chair. Physician B says she was just about to go see her last patient of the day. Which of the following is the most appropriate immediate action
by Physician A?

0 A) Explain the situation to the waiting patient


0 B) Inform Physician B that another staff physician will see the waiting patient
0 C) Notify the clinic administrator
0 D) Report Physician B to the stale medical board
0 E) Report Physician B's behavior to the clinic's Medical Director

https://t.me/USMLENBME2CK
ts e « t
Previous Next Lab Values Calculator Review Help pause
Exam Section 1: Item 15 of 50 National Board of Medical Examiners Time Remaining.
an Comprehensive Clinical Science Self-Assessment 4 hr 59 min 6 sec

15. A 65-year-old man comes to the office for follow-up 5 days after discharge from the hospital, where he received treatment for an exacerbation of chronic obstructive pulmonary disease. Medical history
otherwise is unremarkable. Medications are fluticasone-salmeterol, tiotropium, albuterol as necessary, and a 5-day prednisone taper; he also has 4 days remaining of a 7-day course of doxycycline. Since
discharge, he has been taking his medications as directed and has noticed improvement in his breathing and decreased coughing. He smoked one pack of cigarettes daily for 50 years but quit before his
hospital admission. He does not drink alcoholic beverages. Vital signs are pulse 90/min, respirations 20/min, and blood pressure 120/80 mm Hg. Pulse oximetry on room air shows an oxygen saturation of
94%. Auscultation of the lungs discloses decreased breath sounds with occasional wheezing and no crackles. Cardiac examination discloses a soft S, and S There is no clubbing of the digits or edema of
the extremities. In considering this patient's risk for readmission, which of the following is the most appropriate management?

0 A) Incentive spirometry
0 B) Long-term prednisone therapy
0 C) Oxygen therapy
0 D) Pulmonary rehabilitation
0 E) Suppressive azithromycin therapy three times weekly

https://t.me/USMLENBME2CK
ts e « t
Previous Next Lab Values Calculator Review Help pause
Exam Section 1: ltem 16 of 50 National Board of Medical Examiners Time Remaining.
an Comprehensive Clinical Science Self-Assessment 4 hr 59 min 4 sec

16. A 32-year-old primigravid woman at 16 weeks' gestation comes to the physician for a prenatal visit. She has bipolar disorder. Her medications are lithium carbonate and a prenatal vitamin. Physical
examination shows a uterus consistent in size with a 16-week gestation. On mental status examination, she is cooperative and fully oriented. She has a euthymic mood and congruent affect. Thought
processes are linear. She reports reading about the possible adverse effects of mood stabilizers on the developing fetus and asks if she should stop taking lithium carbonate. Which of the following is the
most appropriate recommendation regarding this patient's medication regimen during pregnancy?

0 A) Discontinue mood stabilizer pharmacotherapy


0 B) Switch from lithium carbonate to carbamazepine
0 C) Switch from lithium carbonate lo clozapine
0 D) Switch from lithium carbonate to valproic acid
0 E) Continue the current regimen

https://t.me/USMLENBME2CK ti
Previous Next Lab Values Calculator Review Help pause
Exam Section 1: Item 17 of 50 National Board of Medical Examiners Time Remaining.
an Comprehensive Clinical Science Self-Assessment 4 hr 59 min 2 sec

17. A 30-year-old primigravid woman at 10 weeks' gestation comes to the physician for her first prenatal visit. She has polycystic ovarian syndrome. She conceived following clomiphene therapy, and her only
current medication is a prenatal vitamin. She is 165 cm (5 fl 5 in) tall and weighs 82 kg (180 1b); BMI is 30 kg/m?. Examination shows a uterus consistent in size with a 10-week gestation. Which of the
following is the most appropriate recommendation for this patient during her pregnancy?

0 A) Amniocentesis
0 B) Cesarean delivery
0 C) Chorionic villus sampling
0 D) limiting pregnancy weight gain
0 E) Measurement of serum fasting insulin concentration

https://t.me/USMLENBME2CK
ts e « t
Previous Next Lab Values Calculator Review Help pause
Exam Section 1: ltem 18 of 50 National Board of Medical Examiners Time Remaining.
an Comprehensive Clinical Science Self-Assessment 4 hr 59 min 0 sec

18. A previously healthy 57-year-old man comes to the emergency department 1 hour after the sudden onset of severe occipital headache. His temperature is 37.2°C (99°F), pulse is 92/min, respirations are
18/min, and blood pressure is 180/98 mm Hg. He is mildly lethargic but oriented to person, place, and time. Neurologic examination shows no other abnormalities. A CT scan of the head shows no
abnormalities. Which of the following is the most appropriate next step in management?

0 A) Measurement of erythrocyte sedimentation rate


0 B) Measurement of intraocular pressure
0 C) Carotid duplex ultrasonography
0 D) MRI of the brain with contrast
0 E) Intravenous prochlorperazine therapy
0 F) Cerebrospinal fluid analysis

https://t.me/USMLENBME2CK ti e
Previous Next Lab Values Calculator Review Help pause
Exam Section 1: Item 19 of 50 National Board of Medical Examiners Time Remaining.
an Comprehensive Clinical Science Self-Assessment 4 hr 58 min 58 sec

19. A45-year-old woman comes to the clinic because of a 2-month history of increasingly severe epigastric pain, bloating, and intermittent nausea and vomiting. During this time, she has had decreased
appetite resulting in a 3.6-kg (8-lb) weight loss. She has hypertension and type 2 diabetes mellitus. The conditions have been poorly controlled during the past 2 years, because she has not been adherent
to her medication regimen. The patient has no history of operative procedures. Medications are metformin, insulin glargine, atorvastatin, lisinopril, and daily aspirin. She appears well. She is 157 cm (5 ft 2
in) tall and weighs 91 kg (200 lb); BMI is 37 kg/m2. Temperature is 37.0°C (98.6°F), pulse is 78/min, respirations are 12/min, and blood pressure is 124/68 mm Hg. The abdomen is protuberant; there is no
organomegaly or tenderness to palpation. Bowel sounds are decreased. Results of a complete blood count, metabolic profile, and determination of serum lipase activity are within the reference ranges.
Which of the following is the most appropriate next step in diagnosis?

0 A) Endoscopic retrograde cholangiopancreatography


0 B) Esophageal manometry
0 C) Gastric emptying study
0 D) MRI of the abdomen
0 E) Ultrasonography of the abdomen

https://t.me/USMLENBME2CK
ts e « t
Previous Next Lab Values Calculator Review Help pause
Exam Section 1: Item 20 of 50 National Board of Medical Examiners Time Remaining.
an Comprehensive Clinical Science Self-Assessment 4 hr 58 min 56 sec

20. A 54-year-old man comes to the office because of a 3-week history of a dull ache along the back of his left ankle that is worse in the morning when he gets out of bed. The pain also worsens when he plays
tennis on weekends. Acetaminophen once daily has provided moderate relief of the pain. Medical history is unremarkable, and the patient takes no other medications. He is 178 cm (5 fl 10 in) tall and
weighs 99 kg (219 lb); BMI is 31 kg/m2. Vital signs are within normal limits. The patient appears well. There is mild swelling and tenderness to palpation at the proximal aspect of the Achilles tendon. The
remainder of the examination discloses no abnormalities. Thompson test is negative. In addition to recommending rest, which of the following is the most appropriate next step in management?

0 A) Add oxycodone as needed for pain


0 B) Increase the frequency of acetaminophen to four times daily
0 C) Inject cortisone into the calcaneal insertion site of the Achilles tendon
0 D) Recommend physical therapy
0 E) Schedule surgical intervention for the Achilles tendon

https://t.me/USMLENBME2CK
ts e « t
Previous Next Lab Values Calculator Review Help pause
Exam Section 1: Item 21 of 50 National Board of Medical Examiners Time Remaining.
an Comprehensive Clinical Science Self-Assessment 4 hr 58 min 54 sec

21. A 10-year cohort study is conducted lo investigate the effect of cigarette smoking on the incidence of head and neck cancers in adults. A total of 10,000 adult participants with no history of any type of
cancer are enrolled in the study; 2000 participants are current cigarette smokers and 8000 are current nonsmokers and have never smoked cigarettes. During the 10-year study period, 20 smokers and
eight nonsmokers develop head and neck cancer. Based on these data, which of the following best represents the attributable risk from smoking for head and neck cancer among current cigarette
smokers?

O A) 0.001
OB) 0.009
O)0.01
OD) 0.09
OE)0.1

https://t.me/USMLENBME2CK ti
Previous Next Lab Values Calculator Review Help pause
Exam Section 1: Item 22 0f 50 National Board of Medical Examiners@ Time Remaining:
i ark Comprehensive Clinical Science Self-Assessment 4hr58min53 sec

22. A 15-year-old girl is brought to the office for a well-child examination. She has no history of serious illness and takes no medications. Her temperature
is 36.9C(98.4F), pulse is 86/min, respirations are 16/min, and blood pressure is 100/60 mm Hg. A photograph of the back is shown. The remainder
of the examination shows no abnormalities. In addition to routine annual growth and development assessments, which of the following is the most
appropriate recommendation regarding routine screening in this patient?

0 A) Yearly CT scan of the chest and abdomen


0 B) Yearly MRI of the brain
0 C) Yearly nerve conduction studies
0 D) Yearly ophthalmologic examination
0 E) No additional screening is indicated

https://t.me/USMLENBME2CK ts e « t
Previous Next Lab Values Calculator Review Help pause
Exam Section 1: Item 23 of 50 National Board of Medical Examiners Time Remaining.
an Comprehensive Clinical Science Self-Assessment 4 hr 58 min 50 sec

23. A 7-year-old gir1 is brought to the physician by her mother because she is concerned that her daughter has had two episodes of vaginal bleeding during the past 3 months. She also has had breast
development during this time. Development had been appropriate for age. She is at the 48th percentile for height and 47th percentile for weight and BMI. Breast development is sexual maturity rating
(SMR) stage 2, and axillary and pubic hair development is SMR stage 3. Examination shows normal-appearing external genitalia. Pelvic examination is deferred. An MRI of the pelvis shows a normal
vagina and cervix. The uterus is 4 cm long, and a 76-0m left ovarian mass is seen. Which of the following is the most appropriate next step in management?

0 A) Chromosomal analysis
0 B) Measurement of serum CA 125 concentration
0 C) CT scan of the pituitary gland
0 D) Fine-needle aspiration of the mass
0 E) left oophorectomy

https://t.me/USMLENBME2CK ti e
Previous Next Lab Values Calculator Review Help pause
Exam Section 1: Item 24 0f 50 National Board of Medical Examiners Time Remaining.
an Comprehensive Clinical Science Self-Assessment 4 hr 58 min 47 sec

24. A 3-year-old boy is brought to the physician because of a scaling lesion on the scalp for 7 days. The family has no pets; the day-care center that the child attends has a pet turtle. The other children at the
day-care center are asymptomatic. Examination of the scalp shows a thin plaque with alopecia and scaling. Which of the following is the most appropriate pharmacotherapy to prevent spread to the
children in the day-care center?

0 A) Oral griseofulvin for the patient only


0 B) Oral griseofulvin for the patient and his classmates
0 C) Topical hydrocortisone for the patient only
0 D) Topical hydrocortisone for the patient and his classmates
0 E) Topical selenium sulfide for the patient only
0 F) Topical selenium sulfide for the patient and his classmates

https://t.me/USMLENBME2CK ti
Previous Next Lab Values Calculator Review Help pause
Exam Section 1: Item 25 of 50 National Board of Medical Examiners@ Time Remaining:
i ark Comprehensive Clinical Science Self-Assessment 4hr58min45 sec

25. A 5-year-old boy is brought to the physician by his parents as a new patient because they are concerned that he may have an immunodeficiency.
The family recently moved to the area. The parents say their son's previous physician had been examining him for the condition. The patient has
had six episodes of pneumonia and 10 episodes of sinusitis since birth. He underwent uncomplicated surgical drainage of the sinuses at the ages
of 3 and 4 years. He had his first episode of otitis media at the age of 2 months and underwent uncomplicated placement of tympanostomy tubes
at the age of 7 months. His umbilical stump fell off at the age of 2 weeks. The patient's previous leukocyte, lymphocyte, and platelet counts;
serum immunoglobulin and CH50 concentrations; and diphtheria, tetanus, pneumococcal, and Haemophi!us inf!uenzae type b titers are within the
reference ranges. Six previous chest x-rays showed lower lobe consolidations in inconsistent locations; subsequent x-rays showed clearing of the
infiltrates. Today, vital signs are within normal limits. Examination shows no abnormalities. A CT scan of the chest is shown. Which of the following
is the most likely diagnosis?

0 A) Common variable immunodeficiency


0 B) Complement deficiency
0 C) Leukocyte adhesion defect
0
0
D) Primary ciliary dyskinesia
E) Wiskott-Aldrich syndrome
•·

https://t.me/USMLENBME2CK ts e « t
Previous Next Lab Values Calculator Review Help pause
Exam Section 1: Item 26 of 50 National Board of Medical Examiners Time Remaining.
an Comprehensive Clinical Science Self-Assessment 4 hr 58 min 42 sec

26. A physician, who is covering for a colleague who is out of town, receives a telephone call at the office from a dentist regarding whether antibiotic prophylaxis is needed for a scaling procedure for the
colleague's patient. The patient is a 50-year-old woman with mitral valve prolapse diagnosed 15 years ago prior to her transferring to the colleague's care. She has never had endocarditis. Further review
of the patient's medical record indicates no documentation of echocardiography. She also has hypertension, type 2 diabetes mellitus, and an allergy to penicillin. Medications are lisinopril and metformin.
Cardiac examination from the patient's last office visit 6 months ago showed a normal pulse and no murmurs. Which of the following is the most appropriate next step in management of the colleague's
patient?

0 A) Advise the dentist to proceed with the procedure without antibiotic prophylaxis
0 B) Ask the dentist to postpone the procedure, and then schedule outpatient echocardiography
0 C) Ask the dentist to postpone the procedure, and then schedule an examination in the office
0 D) Prescribe amoxicillin to be taken 1 hour before the procedure
0 E) Prescribe clindamycin to be taken 1 hour before the procedure

https://t.me/USMLENBME2CK
ts e « t
Previous Next Lab Values Calculator Review Help pause
Exam Section 1: Item 27 of 50 National Board of Medical Examiners Time Remaining.
an Comprehensive Clinical Science Self-Assessment 4 hr 58 min 40 sec

27. A 52-year-old woman with amyotrophic lateral sclerosis has been receiving mechanical ventilation in the hospital for the past 4 weeks. She is taking no medications. Attempts at discontinuing mechanical
ventilation have been unsuccessful. Her parents visit her often and participate in her health care decisions. The physician tells her and her parents that she could require mechanical ventilation for several
more years and recommends transfer to a facility for ventilator-dependent patients. Although her parents approve of this recommendation, the patient refuses placement and asks to have mechanical
ventilation and all other life-sustaining treatments withheld. She says, "I would rather die now than spend the rest of my life in one of those places." Which of the following is the most appropriate next step
in management?

0 A) Ask the parents privately about the patient's prior beliefs about long-term care
0 B) Assess the patient's mental capacity to make her own health care decisions
0 C) Continue current treatment and start additional treatment as needed
0 D) Continue current treatment but do not start any new life-sustaining treatment
0 E) Seek a court order to appoint a guardian to make her health care decisions

https://t.me/USMLENBME2CK ti
Previous Next Lab Values Calculator Review Help pause
Exam Section 1: Item 28 of 50 National Board of Medical Examiners Time Remaining.
an Comprehensive Clinical Science Self-Assessment 4 hr 58 min 38 sec

28. A 62-year-old man comes to the physician because of a lesion on his right temple for 3 months. He works as a farmer. Examination shows a 1.5-cm ulcerated lesion over the right temple area; the lesion
has a raised margin and a waxy border. If left untreated, which of the following is the most likely sequela of the lesion?

0 A) local spread
0 B) lymphatic invasion
0 C) Regression
0 D) Systemic metastases

https://t.me/USMLENBME2CK ti
Previous Next Lab Values Calculator Review Help pause
Exam Section 1: Item 29 of 50 National Board of Medical Examiners Time Remaining.
an Comprehensive Clinical Science Self-Assessment 4 hr 58 min 36 sec

29. A 57-year-old man comes to the emergency department because of a 6-hour history of fatigue and mild shortness of breath. He has type 2 diabetes mellitus, hypertension, and anuric renal failure requiring
hemodialysis. He reports having missed his regularly scheduled dialysis session 2 days ago. His current medications are enalapril, amlodipine, insulin, calcilriol, and calcium supplementation. His
temperature is 37°C (98.6°F), pulse is 110/min, respirations are 24/min, and blood pressure is 160/100 mm Hg. Examination shows no jugular venous distention. Crackles are heard at both lung bases.
Heart sounds are normal. Serum studies show a potassium concentration of 6.8 mEq/L, urea nitrogen concentration of 120 mg/dL, and creatinine concentration of 9.9 mg/dl. An ECG shows sinus
tachycardia at 115/min, with widening of the PR and ORS intervals and peaked T waves. Intravenous administration of which of the following is the most appropriate next step in treatment?

OA Atenolol
0 B) Calcium chloride
0 C) Furosemide
0 D) Lidocaine
0 E) Magnesium sulfate
0 F) 0.9% Saline

https://t.me/USMLENBME2CK ti
Previous Next Lab Values Calculator Review Help pause
Exam Section 1: Item 30 of 50 National Board of Medical Examiners@ Time Remaining:
i ark Comprehensive Clinical Science Self-Assessment 4hr58min 34 sec

30. A 13-year-old boy is brought to the physician because of intermittent pain in his right thigh during the past 6 weeks. The symptoms are exacerbated
by running, and his mother reports that he limps for 30 minutes after running. He has no history of serious illness and takes no medications. He
recalls no recent trauma to either leg. He has practiced with the school football team three times weekly for the past 2 months. He is at the 75th
percentile for height and 95th percentile for weight and BMI. Examination shows a normal gait. There is no tenderness of the right hip or thigh.
Internal rotation of the right hip is difficult, especially when the knee is flexed. Genital and pubic hair development are sexual maturity rating (SMR)
stage 3. An x-ray of the hips is shown. Which of the following is the most appropriate treatment?

0 A) Physical therapy
0 8) Nonsteroidal anti-inflammatory drug therapy
0 C) Cast immobilization of the right hip and thigh
0 D) Surgical stabilization of the femoral head
0 E) No treatment is indicated

https://t.me/USMLENBME2CK ts e « t
Previous Next Lab Values Calculator Review Help pause
Exam Section 1: ltem 31 of 50 National Board of Medical Examiners Time Remaining.
an Comprehensive Clinical Science Self-Assessment 4 hr 58 min 32 sec

31. An 87-year-old woman with type 2 diabetes mellitus comes to the physician for evaluation of a nonhealing fool ulcer that has increasingly worse tissue necrosis and erythema. She has a2-year history of
mild to moderate dementia, Alzheimer type, with no recent changes in cognitive status. The patient lives alone with the help of various neighbors. Her only family is an out-of-town sister, with whom she last
spoke 1 year ago. Physical examination shows a 2-cm ulcer that exposes the head of the first metatarsal. There is 1 cm of necrosis and 2 cm of erythema around the edge of the ulcer. On mental status
examination, the patient is alert and oriented to person and place but thinks that the year is 1999. She recalls one of three objects after 5 minutes. A below-the-knee amputation is recommended. The risks
and benefits of the operation are discussed with the patient. She understands and repeats the discussion correctly. Attempts to contact her sister are unsuccessful. Which of the following is the most
appropriate next step in management?

0 A) Consult with the hospital attorney


0 B) Delay the amputation until her sister can be contacted
0 C) Obtain a psychiatric assessment of this patient's mental capacity
0 D) Schedule a follow-up examination after 1 month of antibiotic therapy
0 E) Proceed with the amputation

https://t.me/USMLENBME2CK ti
Previous Next Lab Values Calculator Review Help pause
Exam Section 1: ltem 32 of 50 National Board of Medical Examiners Time Remaining.
an Comprehensive Clinical Science Self-Assessment 4 hr 58 min 29 sec

32. A 4 7 -year-old woman comes to the physician because of a 2-month history of substernal chest lightness and shortness of breath associated with exercise and relieved by rest. She previously walked
2 miles daily, but now she is limited to walking one-half block because of chest pain, dyspnea, and light-headedness. She has never smoked and drinks one to two beers daily. Her temperature is 36.8°C
(98.2F), pulse is 98/min, respirations are 20/min, and blood pressure is 110/82 mm Hg. Jugular venous pressure is 8 cm H0. Crackles are heard at both lung bases. The point of maximal impulse is
increased but not displaced; S,is normal, but Sis decreased. There is a grade 3/6, harsh systolic murmur at the cardiac base. Serum studies show a total cholesterol concentration of 210 mg/dL,
HDL-cholesterol concentration of 95 mg/dl, and LDL-cholesterol concentration of 100 mg/dl. Which of the following is the most likely cause of this patient's heart failure?

0 A) Alcohol use disorder


0 B) Amyloidosis
0 C) Anemia
0 D) Aortic stenosis
0 E) Atrial fibrillation
0 F) Car pulmonale
0 G) Hemochromatosis
0 H) lschemic heart disease
0 I ) Mitral regurgitation

https://t.me/USMLENBME2CK ti
Previous Next Lab Values Calculator Review Help pause
Exam Section 1: ltem 33 of 50 National Board of Medical Examiners Time Remaining.
an Comprehensive Clinical Science Self-Assessment 4 hr 58 min 27 sec

33. A 30-year-old man who is in the US Army is brought to the emergency department of a military hospital 40 minutes after being hit by a motor vehicle traveling 25 mph while he was walking. On arrival, he is
unconscious. Temperature is 36.0°C (96.8°F), pulse is 113/min, respirations are 22/min, and blood pressure is 80/55 mm Hg. Pulse oximetry on room air shows an oxygen saturation of 90%. On
examination, the abdomen is rigid. He is resuscitated with 8 units of packed red blood cells, 6 units of fresh frozen plasma, and 2 units of platelets. Emergency laparotomy shows transection of the small
bowel 5 cm proximal to the cecum. Laboratory studies done 45 minutes after arriving in the surgical intensive care unit show an activated partial thromboplastin time of 42 seconds and a prothrombin time
of 25 seconds (INR=3.2). Which of the following is the most likely cause of the coagulation abnormalities in this patient?

0 A) Cirrhosis
0 B) Disseminated intravascular coagulation
0 C) Factor VII deficiency
0 D) Hyperfibrinogenemia
0 E) Thrombocytopenia

https://t.me/USMLENBME2CK ti
Previous Next Lab Values Calculator Review Help pause
Exam Section 1: Item 34 of 50 National Board of Medical Examiners Time Remaining.
an Comprehensive Clinical Science Self-Assessment 4 hr 58 min 25 sec

34. A 25-year-old woman, gravida 1, para 1, comes to the office because of a 3-month history of chronic moderate perianal pain and bleeding that began 3 days after she gave birth via cesarean delivery.
Before the onset of her symptoms, she had constipation associated with the use of postdelivery oxycodone therapy. The constipation resolved after she discontinued the medication, but her bowel
movements have been painful and she has noticed blood on the toilet tissue. One week ago, the patient's obstetrician prescribed topical hydrocortisone and diltiazem, which provide temporary relief but do
not resolve her symptoms. The patient's medical history otherwise is unremarkable and she takes no other medications. Family history is unremarkable. Vital signs are temperature 37.2°C (99.0°F), pulse
80/min, respirations 20/min, and blood pressure 130/80 mm Hg. Examination of the anus shows a deep wound in the posterior mid line with exposure of the internal sphincter muscle and an associated
1.5-cm skin tag. Which of the following is the most appropriate next step in management?

0 A) Biopsy of the wound


0 B) Colonoscopy
0 C) Metronidazole therapy
0 D) Surgical repair

https://t.me/USMLENBME2CK ti
Previous Next Lab Values Calculator Review Help pause
Exam Section 1: ltem 35 of 50 National Board of Medical Examiners Time Remaining.
an Comprehensive Clinical Science Self-Assessment 4 hr 58 min 22 sec

35. A42-year-old man is brought to the emergency department by paramedics 1 hour after his girlfriend found him confused and agitated in his home. The girlfriend says that when she spoke with him on the
telephone last night, she noted no problems, but she became worried when he did not answer his telephone 3 hours ago. He has a 3-year history of chronic back pain for which he takes oxycodone. He
also has major depressive disorder and seasonal allergies treated with citalopram and diphenhydramine, respectively. On arrival, he is confused, screaming loudly, and actively pulling against wrist
restraints. His temperature is 38°C (100.4°F), pulse is 163/min, respirations are 24/min, and blood pressure is 215/105 mm Hg. Physical examination shows diaphoresis and tremulousness. The pupils are
dilated. Which of the following is the most likely cause of this patient's confusion?

0 A) Acetaminophen overdose
0 B) Cannabis withdrawal
0 C) Cocaine intoxication
0 D) Diphenhydramine toxicity
0 E) Opioid withdrawal

https://t.me/USMLENBME2CK ti
Previous Next Lab Values Calculator Review Help pause
Exam Section 1: ltem 36 of 50 National Board of Medical Examiners Time Remaining.
an Comprehensive Clinical Science Self-Assessment 4 hr 58 min 20 sec

36. A21-year-old woman, gravida 2, para 1, at 36 weeks' gestation is admitted to the hospital in labor. She received no prenatal care. She has no history of operative procedures. She reports that she
occasionally used cocaine before she knew she was pregnant but has abstained from illicit drug use during the past 8 months. On admission, the cervix is 5 cm dilated and 100% effaced; the vertex is at
0 station. There is a moderate amount of blood-tinged mucus. The membranes are artificially ruptured, yielding clear fluid. Her blood group is A, Rh-negative; a serum anti-D antibody titer is positive.
During the next 30 minutes, fetal heart monitoring shows regular, sinusoidal wave forms. Which of the following is the most likely cause of this fetal heart rate pattern?

0 A) Abruptio placentae
0 B) Fetal heart defect
0 C) Maternal cocaine use
0 D) Placenta previa
0 E) Rh isoimmunization

https://t.me/USMLENBME2CK ti
Previous Next Lab Values Calculator Review Help pause
Exam Section 1: ltem 37 of 50 National Board of Medical Examiners Time Remaining.
an Comprehensive Clinical Science Self-Assessment 4 hr 58 min 16 sec

37. A22-year-old college student comes to the physician because of a 3-year history of falling asleep during her classes; she worries that she will not be able to continue with her education because of this.
She says that she always has had difficulty staying awake during class, even in high school. She also occasionally has trouble staying awake during movies. She says she has no problems getting to sleep
or waking up. She goes to bed between midnight and 1 AM and gets up at 8 AM. She has tried sleeping for longer durations, even up to 8½ hours, but she still has been unable to stay awake during the
day. She has the greatest difficulty staying awake during her classes after lunch. She thinks that she sleeps very soundly and moves little during sleep because her bedsheets are neat when she awakens
in the morning. She does not remember having dreams. She reports that no one has ever told her that she snores. She is 170 cm (5 fl 5 in) tall and weighs 62 kg (136 lb); BMI is 21 kg/m. Physical
examination shows no abnormalities. On mental status examination, she has a concerned mood and a reactive affect. Which of the following is the most appropriate next step in diagnosis?

0 A) Minnesota Multiphasic Personality Inventory


0 B) Overnight pulse oximetry
0 C) Pulmonary function tests
Oo MRI of the brain
0 E) Multiple sleep latency tests

https://t.me/USMLENBME2CK
ts e « t
Previous Next Lab Values Calculator Review Help pause
Exam Section 1: ltem 38 of 50 National Board of Medical Examiners Time Remaining.
an Comprehensive Clinical Science Self-Assessment 4 hr 58 min 14 sec

38. A 38-year-old woman comes to the office because of a 6-month history of progressive fatigue and muscle weakness. During this time, she has not been able to concentrate at work and has noticed that
her voice has become hoarse. During the past 3 months, she has had a 5.4-kg (12-lb) weight gain despite limiting her food intake. She has bipolar disorder, type 2 diabetes mellitus, and
hypercholesterolemia. Her medications are metformin, lisinopril, lithium, and pravastatin. She does not smoke cigarettes, drink alcohol, or use illicit drugs. Her temperature is 37°C (98.6°F), pulse is
50/min, respirations are 14/min, and blood pressure is 130/90 mm Hg. Examination shows cold skin and coarse hair over the scalp. There is a firm, nontender, symmetrically enlarged thyroid gland. Deep
tendon reflexes are 1+ Her serum thyroid-stimulating hormone concentration is 25 U/mL. Results of serum electrolyte concentrations and renal function tests are within the reference ranges. Antithyroid
peroxidase antibody testing is negative. Which of the following is the most likely cause of these findings?

0 A) Autoimmune thyroiditis
0 B) Granulomatous infiltration
0 C) Iodine deficiency
0 D) Medication adverse effect
0 E) Pituitary dysfunction

https://t.me/USMLENBME2CK ti
Previous Next Lab Values Calculator Review Help pause
Exam Section 1: ltem 39 of 50 National Board of Medical Examiners Time Remaining.
an Comprehensive Clinical Science Self-Assessment 4 hr 58 min 12 sec

39. A 27-year-old man comes to the emergency department because of a progressive rash and wheezing. Yesterday, the patient received the diagnosis of cellulitis of his right leg, and cephalexin therapy was
begun. He has a history of mild, intermittent asthma and eczema. His only other medication is inhaled albuterol as needed. He had a mild allergic reaction to ampicillin as a child. He is in moderate
distress. His temperature is 37°C (98.6F), pulse is 118/min, respirations are 22/min, and blood pressure is 90/60 mm Hg. Examination shows a 3 x 3-cm, erythematous, warm, tender rash on the right leg
that is unchanged from yesterday. There are new, raised, 1 x 2-cm pruritic lesions on the upper and lower extremities and trunk. Diffuse, scattered inspiratory and expiratory wheezes are heard bilaterally.
Which of the following is the most likely diagnosis?

0 A) Anaphylactic reaction
0 B) Rocky Mountain spotted fever
0 C) Staphylococcal scalded skin syndrome
0 0) Toxic shock syndrome
0 E) Urticaria I vasculitis

https://t.me/USMLENBME2CK ti
Previous Next Lab Values Calculator Review Help pause
Exam Section 1: Item 40 of 50 National Board of Medical Examiners Time Remaining.
an Comprehensive Clinical Science Self-Assessment 4 hr 58 min 9 sec

40. A 37-year-old man comes to the physician because he has had two bowel movements containing bright red blood during the past 6 hours. He has a 3-day history of intermittent severe abdominal cramps
and diarrhea. He has had five loose stools daily during this period. He has no history of serious illness and takes no medications. His last visit to a physician was 5 years ago. His most recent travel
experience was a trip to Canada 6 months ago. His temperature is 37"C (98.6°F), pulse is 98/min, and blood pressure is 126/78 mm Hg. Abdominal examination shows diffuse severe tenderness with no
peritoneal signs. Bowel sounds are increased. Rectal examination shows no abnormalities. There is no stool in the rectal vault. His hemoglobin concentration is 13 g/dL, leukocyte count is 12,300/mm3,
and platelet count is 302,000/mm3. Which of the following is the most likely causal organism?

0 A) Clostridium difficile
0 B) Clostridium perfringens
0 C) Cryptosporidium parvum
0 D) Enterotoxigenic Escherichia coli
0 E) Escherichia co/i0157:H7

https://t.me/USMLENBME2CK ti
Previous Next Lab Values Calculator Review Help pause
Exam Section 1: Item 41 of 50 National Board of Medical Examiners Time Remaining.
an Comprehensive Clinical Science Self-Assessment 4 hr 58 min 7 sec

41 . A 50-year-old man with a 30-year history of paranoid schizophrenia is admitted to a psychiatric facility because of an exacerbation of his condition. His history includes multiple admissions to the hospital
for his condition. The physician prescribes oral antipsychotic therapy and the patient agrees to take the medication while in the facility, but he says that he does not need it and he will stop taking the
medication as soon as he is discharged. He states, "I don't trust any of you. You're all out to get me. But I'll play along until my time here is done." Which of the following is the most appropriate next step in
management?

0 A) Administer the antipsychotic therapy until the patient's paranoia completely resolves
(B) Change the oral antipsychotic therapy to a long-acting injectable formulation
0 C) Commit the patient to the facility against his will
0 D) Discharge the patient with support from the local community mental health center, telling him that he can always return to the facility if he needs help in the future
0 E) Maximize the dose of the anti psychotic therapy until the patient gains insight into his illness

https://t.me/USMLENBME2CK ti
Previous Next Lab Values Calculator Review Help pause
Exam Section 1: ltem 42 of 50 National Board of Medical Examiners Time Remaining.
an Comprehensive Clinical Science Self-Assessment 4 hr 58 min 5 sec

42. An 8-year-old boy is brought to the emergency department because of a 2-day history of urinary urgency and frequency and right-sided abdominal cramps. The cramps are intermittent, and he cannot get
comfortable in any position when they occur. He has not had fever, diarrhea, or constipation, but he has vomited twice. He has no history of similar episodes or serious illness. He takes no medications. His
paternal uncle has a history of renal calculi, and his cousin has Crohn disease. The patient has no dietary restrictions. His temperature is 37.5°C (99.5°F). The right flank is tender to palpation. An
abdominal x-ray shows a 3-mm ovoid, opaque mass in the right pelvis. Laboratory studies show:
Serum
Urea nitrogen 20 mg/dl
Creatinine 0.1 mg/dl
Urine
pH 6.5
RBC 30-50/hpf
WBC 3-5/hpf

In addition to administration of morphine, which of the following is the most appropriate next step in management?

0 A) Oral administration of allopurinol


0 B) Oral administration of fluids
0 C) Oral administration of hydrochlorothiazide
0 D) Oral administration of potassium citrate
0 E) Oral administration of pyridoxine
0 F ) Lithotripsy

https://t.me/USMLENBME2CK ti
Previous Next Lab Values Calculator Review Help pause
Exam Section 1: ltem 43 of 50 National Board of Medical Examiners Time Remaining.
an Comprehensive Clinical Science Self-Assessment 4 hr 58 min 2 sec

43. A previously healthy 57-year-old woman comes to the physician because of a 6-month history of muscle weakness and fatigue. The weakness began in the muscles of her face and has progressed to all of
the muscles in her body. Examination shows bilateral ptosis and weakness of the left abducens muscle. An x-ray of the chest shows a mediastinal mass. Which of the following is the most appropriate next
step in diagnosis of this patient's neurologic findings?

0 A) Nerve conduction studies with repetitive stimulation


0 B) MRI of the cervical spine
0 C) Somatosensory evoked potentials
0 D) Biopsy of a peripheral nerve
0 E) Biopsy of a skeletal muscle

https://t.me/USMLENBME2CK ti
Previous Next Lab Values Calculator Review Help pause
Exam Section 1: Item 44 of 50 National Board of Medical Examiners Time Remaining.
an Comprehensive Clinical Science Self-Assessment 4 hr 58 min 0 sec

44. A 29-year-old woman, gravida 1, para 1, comes to the physician 1 week after noticing a mildly tender lump in her left breast. Five months ago, she delivered a healthy newborn at term following an
uncomplicated pregnancy and spontaneous vaginal delivery. She is breast-feeding and says her infant feeds equally from both breasts. The patient has no history of serious illness, and her only
medication is a prenatal vitamin. Her mother was diagnosed with breast cancer at the age of 61 years. The patient's vital signs are within normal limits. Examination shows no axillary or supraclavicular
lymphadenopathy. There is a 3-cm, smooth, mobile, cystic, tender mass in the upper outer quadrant of the left breast. Which of the following is the most appropriate next step in management?

0 A) Bromocriptine therapy
0 8) Dicloxacillin therapy
0 C) Discontinuing breast-feeding
0 D) Fine-needle aspiration of the mass
0 E) Increasing breast-feeding on the left breast

https://t.me/USMLENBME2CK ti
Previous Next Lab Values Calculator Review Help pause
Exam Section 1: Item 45 of 50 National Board of Medical Examiners@ Time Remaining:
i ark Comprehensive Clinical Science Self-Assessment 4hr57 min57 sec

Inspiration Expiration

45. An 18-year-old man is brought to the physician 1 hour after he felt light-headed while playing basketball. He has not had chest pain, palpitations, or shortness of breath. He has no history of serious illness
and takes no medications. His father died in a motor vehicle collision at the age of 35 years. There is no family history of serious illness. The patient's pulse is 80/min, and blood pressure is 115/85 mm Hg.
There is no Jugular venous distention. Pulmonary examination shows no abnormalities. A diagram of cardiac auscultation is shown. The murmur does not change with inspiration, position, or Valsalva
maneuver. An ECG shows no abnormalities. Which of the following is the most appropriate next step in management?

0 A) Aortic valve repair


0 B) Implantation of a cardiac defibrillator
0 C) Recommendation not to play competitive sports
0 D) Warfarin therapy
0 E) No management is indicated at this time

https://t.me/USMLENBME2CK ts e « t
Previous Next Lab Values Calculator Review Help pause
Exam Section 1: Item 46 of 50 National Board of Medical Examiners Time Remaining.
an Comprehensive Clinical Science Self-Assessment 4 hr 57 min 53 sec

46. A 37-year-old woman is brought to the emergency department 30 minutes after an episode of syncope at a shopping mall. She says she had shortness of breath and was sweating before she passed out.
She has a childhood history of asthma. She currently has panic disorder and gastroesophageal reflux disease. Current medications include sertraline, omeprazole, and an oral contraceptive. On arrival,
she is alert and oriented to person, place, and lime. Her temperature is 37.4°C (99.3°F), pulse is 110/min, and blood pressure is 90/60 mm Hg. Pulse oximetry on room air shows an oxygen saturation of
90%. Examination, including neurologic examination, shows no abnormalities. Laboratory studies show:
Hemoglobin 12 g/dl
Hematocrit 36%
Leukocyte count 9000/mm3

Arterial blood gas analysis on room air:


pH 7.46
Pco, 30 mm Hg
Po, 62 mm Hg

Which of the following is the most likely explanation for these findings?

0 A) Asthma
0 B) Congestive heart failure
0 C) Panic episode
0 D) Pneumonia
0 E) Pulmonary embolism

https://t.me/USMLENBME2CK ti
Previous Next Lab Values Calculator Review Help pause
Exam Section 1: Item 47 of 50 National Board of Medical Examiners Time Remaining.
an Comprehensive Clinical Science Self-Assessment 4 hr 57 min 51 sec

47 A 62-year-old woman comes to the physician because of a 2-monlh history of generalized fatigue and mild low back pain. She has not had any
other symptoms. She has no history of serious illness and takes no medications. Her last health maintenance examination was 10 months ago,
and a complete blood count at that lime was within the reference range. Examination today shows no abnormalities. Laboratory studies show:
Hematocrit 30%
Mean corpuscular volume 103 m3
Leukocyte count 8200/mm3
Segmented neutrophils 67%
Bands 4%
Lymphocytes 29%
Platelet count
Serum
165,000/mm?
0
Ca?+ 10.9 mg/dL
Creatinine 1.8 mg/dL

A blood smear is shown. Which of the following is the most appropriate next step to confirm the diagnosis?

0 A) Liver function tests


(B) Measurement of serum vitamin B, (cobalamin) concentration
0 C) Serum iron studies
0 D) Colonoscopy
0 E) Bone marrow biopsy

https://t.me/USMLENBME2CK
ts e « t
Previous Next Lab Values Calculator Review Help pause
Exam Section 1: Item 48 of 50 National Board of Medical Examiners Time Remaining.
an Comprehensive Clinical Science Self-Assessment 4 hr 57 min 49 sec

48. A 37-year-old man comes to the physician for a routine health maintenance examination. He feels well. He has no history of serious illness and takes no medications. He received the hepatitis B vaccine at
the age of 28 years. He does not smoke or drink alcohol. During the past 6 months, he has been sexually active with one male partner, and they do not use condoms. The patient works as an accountant
and has not traveled outside the USA. Examination shows no abnormalities. A blood sample is obtained for serum lipid studies and an HIV antibody test. In addition to counseling this patient about risk
factors for sexually transmitted diseases, which of the following is the most appropriate next step in management?

OA PPD skin test


0 B) Haemophifus influenzae type b vaccine
0 C) Hepatitis A vaccine
0 0) Meningococcal vaccine
0 E) Pneumococcal polysaccharide vaccine, 23-valent

https://t.me/USMLENBME2CK
ts e « t
Previous Next Lab Values Calculator Review Help pause
Exam Section 1: Item 49 of 50 National Board of Medical Examiners Time Remaining.
an Comprehensive Clinical Science Self-Assessment 4 hr 57 min 47 sec

49. A27-year-old man comes to the physician because of a 1-monlh history of sudden episodes of a "scary choking feeling." He has had three episodes during this period; the episodes are accompanied by
nausea, sweating, and "heart pounding" sensations. The symptoms peak within 5 to 10 minutes and disappear about 15 minutes later. The first two episodes "came out of the blue" while he was at work;
the last episode occurred when he was stuck in traffic. He is fearful that he will have another episode and that "someone will notice and think I'm going crazy." He has no history of serious medical illness
and takes no medications. He is a veteran of the U.S. Marine Corps and experienced some intense fighting 5 years ago. He has occasional nightmares involving combat and intermittent sadness when he
thinks about military comrades who were killed. He has occasional difficulty falling asleep. He has had no changes in energy level or appetite. He is 175 cm (5 ft 9 in) tall and weighs 82 kg (180 lb); BMI is
27 kg/m2. Vital signs are within normal limits. Physical examination shows no abnormalities. On mental status examination, he is anxious but relates well with the physician. His serum thyroid-stimulating
hormone concentration is within the reference range. A 12-lead ECG shows a normal sinus rhythm. Which of the following is the most appropriate pharmacotherapy?

0 A) Bupropion
0 B) Buspirone
0 C) Haloperidol
0 D) lithium carbonate
0 E) Paroxetine
OF Risperdal

https://t.me/USMLENBME2CK ti e
Previous Next Lab Values Calculator Review Help pause
Exam Section 1: Item 50 of 50 National Board of Medical Examiners Time Remaining.
an Comprehensive Clinical Science Self-Assessment 4 hr 57 min 44 sec

50. A previously healthy 25-year-old man is brought to the emergency department 2 hours after being involved in a motor vehicle collision. He was the restrained driver. On arrival, he is alert and oriented to
person, place, and time. He says he has severe lower abdominal pain. His temperature is 37.5€ (99.5°F), pulse is 105/min, respirations are 18/min, and blood pressure is 120/70 mm Hg. Abdominal
examination shows ecchymoses over the lower quadrants and an abrasion where the seat belt was fastened. There is suprapubic tenderness. Palpation of the iliac crests produces pain. There is blood at
the urethral meatus. His hematocrit is 30%. X-rays of the pelvis show a fracture of the superior and inferior right pubic rami and the right sacroiliac joint. Which of the following is the most appropriate next
step in management?

0 A) Cystography
0 B) Retrograde urethrography
0 C) Placement of a suprapubic catheter
0 D) Intravenous pyelography
0 E) Bladder irrigation

https://t.me/USMLENBME2CK ti
Previous Next Lab Values Calculator Review Help pause
Exam Section 2: Item 1 of 50 National Board of Medical Examiners Time Remaining.
an Comprehensive Clinical Science Self-Assessment 4 hr 59 min 50 sec

1. A47-year-old woman comes to the physician because of a 3-week history of moderate pain in her arms and legs. She states that her "bones ache" in all joints both at rest and with movement. She has
hypertension, hyperlipidemia, and type 2 diabetes mellitus well controlled with diet. Current medications include hydrochlorothiazide and simvastatin. Her sister has SJ6gren syndrome, and her mother has
coronary artery disease. The patient does not smoke and drinks one glass of wine daily. Her blood pressure is 144/80 mm Hg. Cardiopulmonary examination shows no abnormalities. There is tenderness to
palpation over the wrist, metacarpophalangeal, knee, and ankle joints. Clubbing of the fingers and toes is noted. A chest x-ray shows a left upper lobe mass with perihilar lymphadenopathy. X-rays of the
tibia and fibula show periostosis. Which of the following is the most likely diagnosis?

0 A) Bone metastases
0 8) Hypertrophic pulmonary osteoarthropathy
( C) Neuropathic arthropathy (Charcot joint)
0 D) Sarcoidosis
0 E) Systemic sclerosis (scleroderma)

https://t.me/USMLENBME2CK ti
Next Lab Values Calculator Review Help Pause
Exam Section 2: Item 2 of 50 National Board of Medical Examiners Time Remaining.
an Comprehensive Clinical Science Self-Assessment 4 hr 59 min 48 sec

2. A healthy 16-year-old girt comes to the physician for a school physical examination. She is sexually active with one partner and does not use contraception. She has smoked one pack of cigarettes daily for
2 years. She is 163 cm (5 ft 4 in) tall and weighs 70 kg (156 lb); BMI is 27 kg/m2. Physical examination shows no abnormalities. Which of the following behavioral modifications is most likely to have the
greatest impact on mortality in this patient during the next 10 years?

0 A) Condom use
0 B) Monthly breast self-examination
0 C) Seat belt use
0 D) Smoking cessation
0 E) Weight loss

https://t.me/USMLENBME2CK ti e
Previous Next Lab Values Calculator Review Help pause
Exam Section 2: ltem 3 of 50 National Board of Medical Examiners Time Remaining.
an Comprehensive Clinical Science Self-Assessment 4 hr 59 min 46 sec

3. An 82-year-old woman who resides in a skilled nursing care facility is transferred to the hospital because of a 2-day history of fever and cellulilis of the left side of her face. She is hemiparetic, aphasic, and
unable to swallow. She is fed through a gastrostomy tube. She has congestive heart failure, type 2 diabetes mellitus, hypertension, hyperlipidemia, major depressive disorder, and osteoporosis. Her
medications are furosemide, metoprolol, digoxin, glyburide, lisinopril, carvedilol, atorvastatin, amitriptyline, alendronate, and 81-mg aspirin. She appears ill and moans frequently. Her temperature is 39.1C
(102.4F), pulse is 84/min, respirations are 24/min, and blood pressure is 112/72 mm Hg. A firm, exquisitely tender mass is palpated from the left cheek to the angle of the mandible; this area is also
erythematous and edematous. The left pinna appears normal, and the otic canal is not inflamed. The mouth is dry. Bilateral basilar crackles are heard. Cardiac examination shows no abnormalities. The
abdomen is soft and nontender. The gastrostomy tube is in place with no inflammation. Her leukocyte count is 28,000/mm3 (78% segmented neutrophils, 18% bands, and 4% lymphocytes), and serum
glucose concentration is 168 mg/dl. A chest x-ray shows cardiomegaly; there are bilateral basilar increased markings and atelectasis in the left lung base but no focal infiltrates. Which of the following is the
most likely cause of this patient's fever?

0 A) Acute suppurative parotitis


0 B) Aspiration pneumonia
0 C) Mandibular osteomyelitis
0 D) Osteonecrosis of the jaw
0 E) Urinary tract infection

https://t.me/USMLENBME2CK ti
Previous Next Lab Values Calculator Review Help pause
Exam Section 2: ltem 4 of 50 National Board of Medical Examiners Time Remaining.
an Comprehensive Clinical Science Self-Assessment 4 hr 59 min 44 sec

4. A hospitalized 42-year-old man has chills and a temperature of 38.3C (101F) 1 hour after transfusion of packed red blood cells for treatment of anemia secondary to a bleeding duodenal ulcer. He has
never received a blood transfusion in the past. He has no known allergies. He is pale and flushed. He is 178 cm (5 ft 10 in) tall and weighs 79 kg (175 lb); BMI is 25 kg/m2. His pulse is 98/min and regular,
and blood pressure is 128/82 mm Hg. Examination shows no abnormalities. The most appropriate next step in management is administration of which of the following?

0 A) Oral acetaminophen
0 B) Oral ibuprofen
0 C) Subcutaneous epinephrine
0 D) Intravenous diphenhydramine
0 E) Intravenous methylprednisolone

https://t.me/USMLENBME2CK
ts e « t
Previous Next Lab Values Calculator Review Help pause
Exam Section 2: Item 5 of 50 National Board of Medical Examiners Time Remaining.
an Comprehensive Clinical Science Self-Assessment 4 hr 59 min 42 sec

5. A 77-year-old woman comes to the office because of a 1-year history of frequent urinary incontinence during the day and at night. The incontinence is not associated with coughing or sneezing, and she
does not have urinary urgency before the episodes. She has not had pain with urination or vaginal discharge. She has a 10-year history of type 2 diabetes mellitus well controlled with metformin. Her other
medications are atorvastatin and lisinopril. Examination shows some wetting of an absorbent pad that she is wearing. Results of laboratory studies are shown:
Hemoglobin A, 6.2%
Serum
Na 138 mEq/L
K+ 3.7 mEq/L
Cl 98 mEq/L
HCO.-3 26 mEq/L
Urea nitrogen 12 mg/dL
Glucose 180 mg/dL
Creatinine 1.1 mg/dL
Urine
pH normal
Glucose 1+
Protein 2+
RBC none
WBC 3/hpf
Casts none

Which of the following is the most likely cause of this patient's urinary incontinence?

0 A) Hyperglycemia
0 B) Interstitial cystitis
0 C) Medication adverse effect
0 D) Neurogenic bladder
0 E) Stress incontinence

https://t.me/USMLENBME2CK ti
Previous Next Lab Values Calculator Review Help pause
Exam Section 2: Item 6 of 50 National Board of Medical Examiners@ Time Remaining:
i ark Comprehensive Clinical Science Self-Assessment 4hr59min41 sec

6. A71-year-old man is brought lo the emergency department because of a 1-week history of increasingly severe pain in his left third toe. During the past 36 hours, he has not slept because of the pain. He
has coronary artery disease, hypertension, and hyperlipidemia. Six years ago, he had a myocardial infarction. He has never undergone an operation. His medications are ramipril, labetalol, and aspirin.
Acetaminophen and ibuprofen have provided no pain relief. He smoked one pack of cigarettes daily for 40 years but quit 6 years ago. He appears uncomfortable. His pulse is 84/min, and blood pressure is
170/90 mm Hg. A 4-cm, pulsatile, nontender mass is palpated in the left inguinal region. The left third toe is necrotic and exquisitely tender to palpation; there is no erythema or drainage. Pedal pulses are 2+
on the left. The remainder of the examination shows no abnormalities. A CT scan of the abdomen and pelvis is shown. In addition to intravenous analgesic therapy, which of the following is the most
appropriate next step in management?

0 A) Piperacillin-tazobactam prescription
0 B) Placement of an Unna boot
0 C) Placement of a vena cava filter
0 D) Rivaroxaban therapy
0 E) Surgical arterial repair

https://t.me/USMLENBME2CK ts e « t
Previous Next Lab Values Calculator Review Help pause
Exam Section 2: Item 7 of 50 National Board of Medical Examiners Time Remaining.
an Comprehensive Clinical Science Self-Assessment 4 hr 59 min 39 sec

7. A37-year-old woman comes to the physician requesting assistance in applying for disability income. She fell at home 12 months ago and left her job at a day-care center because of subsequent low back
pain. She received the diagnosis of major depressive disorder 8 months ago; her sleep, appetite, and mood have improved with citalopram therapy. Her only other medication is ibuprofen. She asks the
physician to document a permanent and total disability on an application form for disability income so that she will have the financial resources to start college education on a full-time basis. She explains
that she will become depressed again if she resumes her stressful job as a child-care provider. She does not appear to be in distress. She is 157 cm (5 ft 2 in) tall and weighs 68 kg (150 lb); BMI is
27 kg/m?. Her temperature is 37°C (98.6F), pulse is 64/min, and blood pressure is 108/60 mm Hg. Examination of the back shows no tenderness; range of motion of the lumbar spine is full. Neurologic
examination shows no abnormalities. The physician will not fill out the form. In addition to informing the patient, which of the following is the most appropriate next step in management?

0 A) Reassure the patient that she will be all right


0 B) Confront the patient about requesting participation in fraud
0 C) Discuss options for the patient to work while pursuing an education
0 D) Explain that the patient is unlikely to have another depressive episode if she follows her medication regimen
0 E) Obtain an x-ray of the lumbosacral spine

https://t.me/USMLENBME2CK ti e
Previous Next Lab Values Calculator Review Help pause
Exam Section 2: Item 8 of 50 National Board of Medical Examiners Time Remaining.
an Comprehensive Clinical Science Self-Assessment 4 hr 59 min 37 sec

8. A 67-year-old woman comes to the physician because of a 1-year history of a nonproductive cough and progressive shortness of breath with exertion. Her symptoms were not a problem at first, but she now
has trouble walking her dog because of shortness of breath after walking one-half block. She has not had fever or any other symptoms. She has no history of serious illness and takes no medications. She
does not smoke. She retired from her position as an executive secretary 5 years ago. Vital signs are within normal limits. Pulse oximetry on room air shows an oxygen saturation of 92%. Examination shows
no cyanosis. Fine, dry crackles are heard at both lung bases. Examination of the hands shows clubbing. There is no peripheral edema. A chest x-ray shows fine linear opacities in the lower third of the lungs
bilaterally. A CT scan of the chest confirms an interstitial pattern. Pulmonary function tests show an FVC of 48% of predicted and a diffusion capacity of the lung for carbon monoxide of 55% of predicted.
Which of the following is the most appropriate next step in diagnosis?

0 A) Measurement of serum ACE activity


()B) Measurement of serum a,-antitrypsin concentration
0 C) Serum antinuclear antibody assay
0 D) Bronchoalveolar lavage
0 E) lung biopsy

https://t.me/USMLENBME2CK ti
Previous Next Lab Values Calculator Review Help pause
Exam Section 2: Item 9 of 50 National Board of Medical Examiners Time Remaining.
an Comprehensive Clinical Science Self-Assessment 4 hr 59 min 35 sec

9. A 2-year-old boy is brought to the physician for a follow-up examination 2 weeks after being treated for an ear infection. He has been treated for six episodes of acute otitis media and one episode of
pneumonia since birth. Three months ago, he underwent placement of tympanostomy tubes. He currently takes no medications. He appears well. He is at the 75th percentile for length and 25th percentile
for weight. Examination shows several 0.5- to 1-cm, firm, nontender, mobile lymph nodes in the anterior cervical chain. Tympanostomy tubes are in place bilaterally without discharge. Laboratory studies
show:
Hemoglobin 12.5 g/dL
Hematocrit 37%
Leukocyte count 8100/mm3
Platelet count 257,000/mm3
Serum
lgA <7 mg/dL
lgG 720 mg/dL
lgM 75 mg/dL

Which of the following is the most appropriate next step in management?

0 A) Oral antibiotic therapy


0 B) Oral prednisone therapy
(_ C) Intramuscular injection of immune globulin
0 D) Intravenous antithymocyte globulin
0 E) Intravenous immune globulin infusion
0 F ) Bone marrow transplant
0 G) No intervention is indicated at this time

https://t.me/USMLENBME2CK
ts e « t
Previous Next Lab Values Calculator Review Help pause
Exam Section 2: Item 10 of 50 National Board of Medical Examiners8 Time Remaining:
i ark Comprehensive Clinical Science Self-Assessment 4hr59min33 sec

10. A42-year-old woman comes to the physician because of nausea and vomiting and abdominal cramping and swelling over the past 12 hours. She takes an oral
contraceptive. She underwent an appendectomy 5 years ago. Abdominal examination shows distention and moderate tenderness without guarding. An x-ray of the
abdomen is shown. Which of the following is the most likely diagnosis?

0 A) Adynamic ileus
0 B) Mechanical obstruction of the small intestine
0 C) Pelvic abscess
0 D) Perforated abdominal viscus
0 E) Regional enteritis

https://t.me/USMLENBME2CK ts e «
Previous Next Lab Values Calculator Review Help pause
Exam Section 2: Item 11 of 50 National Board of Medical Examiners Time Remaining.
an Comprehensive Clinical Science Self-Assessment 4 hr 59 min 31 sec

11. A 22-year-old woman comes to the physician because of a 2-day history of severe shortness of breath. Over the past week, she has been laking ibuprofen for fever, chills, and muscle pain. She has no
history of serious illness and takes no other medications. Her temperature is 38.4°C (101.1°F), pulse is 105/min and irregular, respirations are 28/min, and blood pressure is 70/50 mm Hg. Examination
shows jugular venous distention. Crackles are heard throughout both lung fields. An S 3 gallop is heard. Pulses are decreased in the upper and lower extremities. An x-ray of the chest shows cardiomegaly
and perihilar fluid. An ECG shows frequent premature ventricular contractions. Echocardiography shows cardiomegaly with normal valve function and an ejection fraction of 18%; her estimated cardiac
output is 1.4 L/min (N=4-5). Which of the following is the most likely diagnosis?

0 A) Acute myocarditis
0 B) Acute pericarditis
0 C) Constrictive pericarditis
0 D) lschemic cardiomyopathy
0 E) Pericardia! tamponade
0 F) Superior vena cava syndrome

https://t.me/USMLENBME2CK ti
Previous Next Lab Values Calculator Review Help pause
Exam Section 2: Item 12 of 50 National Board of Medical Examiners Time Remaining.
an Comprehensive Clinical Science Self-Assessment 4 hr 59 min 30 sec

12. A study is conducted to determine if a hemoglobin A,, greater than 15% predicts diabetic retinopathy. Results show that 88% of all patients without diabetic retinopathy had a hemoglobin A, below 15%.
Which of the following is the most accurate interpretation of these results?

0 A) False-negative rate is 88%


0 B) False-positive rate is 88%
0 C) Negative predictive value is 88%
0 D) Positive predictive value is 88%
0 E) Power of the study design is 88%
0 F) Sensitivity is 88%
0 G) Specificity is 88%

https://t.me/USMLENBME2CK ti
Previous Next Lab Values Calculator Review Help pause
Exam Section 2: Item 13 of 50 National Board of Medical Examiners Time Remaining.
an Comprehensive Clinical Science Self-Assessment 4 hr 59 min 28 sec

13. A 13-year-old girl is brought to the physician by her mother because of a 3-day history of pain and pressure in her face, itchy eyes, and decreased hearing in both ears. She has not had fever, chills,
muscle pain, or nausea. Acetaminophen has not relieved her symptoms. She takes no other medications. One year ago, she had a similar episode and was successfully treated with azithromycin therapy
at an urgent care center, and her mother requests another prescription for azithromycin. The patient's two brothers have well-controlled asthma. Her vital signs are within normal limits. Examination shows
a supple neck with a few shotty lymph nodes in the upper cervical chain. The conjunctivae have a cobblestone appearance. The inferior nasal turbinates are edematous. The tympanic membranes are
injected bilaterally. There is moderate mucus in the posterior pharynx. The tonsils are not enlarged. The lungs are clear to auscultation. Which of the following is the most appropriate next step in
management?

0 A) CT scan of the head


0 B) MRI of the sinuses
0 C) Ceftriaxone therapy
0 D) loratadine therapy
0 E) Prednisone therapy

https://t.me/USMLENBME2CK ti
Previous Next Lab Values Calculator Review Help pause
Exam Section 2: Item 14 of 50 National Board of Medical Examiners Time Remaining.
an Comprehensive Clinical Science Self-Assessment 4 hr 59 min 25 sec

14. A 76-year-old woman is scheduled lo undergo right knee arthroscopy al an ambulatory surgery center. Before the procedure begins, the surgical team confirms the identity of the patient, the procedure
being performed, the side and site of the procedure, the patient's position, and the availability of needed equipment. The procedure is then completed without complications, and the patient is discharged
home 4 hours later. Which of the following is most likely increased at this center as a result of the pre-procedure actions of the surgical team?

0 A) Efficiency
0 8) Patient satisfaction
0 C) Patient-centered care
Oo Reliability

https://t.me/USMLENBME2CK ti
Previous Next Lab Values Calculator Review Help pause
Exam Section 2: Item 15 of 50 National Board of Medical Examiners Time Remaining.
an Comprehensive Clinical Science Self-Assessment 4 hr 59 min 23 sec

15. Immediately after extubation following a total thyroidectomy, a 72-year-old woman has stridor and shortness of breath. She has a history of transient ischemic attacks and coronary artery disease. Her only
medication is aspirin. She appears lo be in distress and is gasping for air. Her pulse is 100/min, respirations are 35/min, and blood pressure is 170/100 mm Hg. Examination shows an intact surgical
incision and no ecchymosis. Breath sounds are decreased bilaterally. Arterial blood gas analysis on 40% oxygen by face mask shows:
pH 7.3
Pco, 60 mm Hg
Po, 200 mm Hg
O, saturation 99%

Which of the following is the most likely diagnosis?

0 A) Cerebral infarction
0 B) Extrinsic tracheal compression
0 C) laryngeal nerve paralysis
0 D) Pulmonary embolism
0 E) Tension pneumothorax

https://t.me/USMLENBME2CK ti
Previous Next Lab Values Calculator Review Help pause
Exam Section 2: Item 16 of 50 National Board of Medical Examiners Time Remaining.
an Comprehensive Clinical Science Self-Assessment 4 hr 59 min 22 sec

16. A46-year-old man with hypertension comes to the physician for a routine examination. When the physician enters the room, the patient says the office staff were talking about him in the waiting room. The
patient has declined antihypertensive therapy in the past; today, he declines it again, saying "it does not matter because the world is going to end soon.� He does not drink alcohol or use illicit drugs. He
lives alone and has minimal contact with his family. He says he has one friend and spends his free time playing video games. He works at home for an accounting firm. He appears fearful and older than
his stated age. He wears an overcoat and woolen hat although the outside temperature is 80°F. His pulse is 80/min and regular, and blood pressure is 140/98 mm Hg. Limited physical examination shows
no abnormalities. On mental status examination, he has a euthymic mood and odd affect. He makes minimal eye contact with the physician and appears anxious when questioned. He says he has not had
anxiety or changes in mood. Cognition is intact, and answers to questions are appropriate. His thought process is linear. He reports no hallucinations. Results of laboratory studies are within the reference
ranges. Which of the following is the most likely diagnosis?

0 A) Autism spectrum disorder


0 B) Delusional disorder
0 C) Oppositional defiant disorder
0 D) Schizophrenia
0 E) Schizotypal personality disorder

https://t.me/USMLENBME2CK ti
Previous Next Lab Values Calculator Review Help pause
Exam Section 2: Item 17 of 50 National Board of Medical Examiners Time Remaining.
an Comprehensive Clinical Science Self-Assessment 4 hr 59 min 20 sec

17. A4-year-old girl is brought to the office by her mother because of a 1-week history of vaginal itching. The mother says her daughter is scratching her vaginal area frequently and the area appears irritated.
The patient has not had fever, pain with urination, or urinary frequency. She has been toilet-trained for 1 year. The mother is not concerned about sexual abuse. When questioned alone, the patient says
that no one has touched her inappropriately. Examination shows erythema of the vaginal area. There is no vaginal discharge, foul odor, bleeding, tears, or ecchymoses. Which of the following is the most
appropriate next step in management?

0 A) Begin antibiotic therapy


0 B) Begin anlifungal therapy
0 C) Order evaluation by child protective services
0 D) Recommend that the patient switch to cotton underwear and avoid bubble baths
0 E) Schedule gynecologic examination

https://t.me/USMLENBME2CK ti
Previous Next Lab Values Calculator Review Help pause
Exam Section 2: Item 18 of 50 National Board of Medical Examiners Time Remaining.
an Comprehensive Clinical Science Self-Assessment 4 hr 59 min 18 sec

18. An 87-year-old man comes to the physician because of a 2-day history of visual changes in his right eye. He notices that the lines of his daily crossword puzzle look curved, and the blinds in his apartment
appear wavy. He wears magnifying lenses for reading. There is no pain in his eye, no photo phobia, and no history of trauma to the area. He had cataract removal from the right eye 5 years ago. There is
no history of serious illness. On examination, visual acuity is 20/200 in his right eye and 20/100 in his left eye. The right pupil is irregular, but it is reactive to light. The left pupil is round and reactive. This
examination finding is unchanged since his cataract operation. Palpation of both globes through closed eyelids shows no abnormalities. His lenses appear clear. Which of the following is the most
appropriate next step in diagnosis?

0 A) Dilated funduscopic examination


0 B) Slit-lamp examination
0 C) Visual field testing
0 D) CT scan of the head
0 E) Temporal artery biopsy

https://t.me/USMLENBME2CK ti
Previous Next Lab Values Calculator Review Help pause
Exam Section 2: Item 19 of 50 National Board of Medical Examiners Time Remaining.
an Comprehensive Clinical Science Self-Assessment 4 hr 59 min 16 sec

19. A 27-year-old woman comes to the office for a health maintenance examination. She says she feels well. She has no history of serious illness and takes no medications. Vital signs are within normal limits.
Examination shows no lymphadenopathy. A 2-cm nodule is palpated in the left lobe of the thyroid gland. The remainder of the examination shows no abnormalities. Results of laboratory studies, including
measurement of serum thyroid-stimulating hormone concentration, are within the reference ranges. Which of the following is the most appropriate next step in management?

QA) "3l scintigraphy


0 B) levothyroxine therapy and reexamination in 6 weeks
0 C) Reexamination of the thyroid gland in 6 months
0 D) Repeat measurement of serum thyroid-stimulating hormone concentration in 6 weeks
0 E) Ultrasonography-guided fine-needle aspiration biopsy

https://t.me/USMLENBME2CK ti
Previous Next Lab Values Calculator Review Help pause
Exam Section 2: Item 20 of 50 National Board of Medical Examiners@ Time Remaining:
i ark Comprehensive Clinical Science Self-Assessment 4hr59min 14 sec

20. A17-year-old boy is brought to the emergency department by his mother because of a 20-minute episode of severe chest pain and sweating that
started 1 hour ago while he was watching television. On arrival, he is alert and repeatedly expresses concern that something is "very wrong." He I aVR
reports that during the episode, his heart was "pounding," he felt short of breath, and he was sure he was having a heart attack and was going to
die. One month ago, he had a similar episode that occurred in the evening 30 minutes after his high school football practice. At that time, results of
laboratory studies and an ECG obtained in the emergency department showed no abnormalities. After the initial episode, he quit the football team,
and he has continued to limit his physical activity because he is afraid of bringing on another episode. He reports no insomnia or changes in
appetite. His mother states that he was diagnosed with an innocent heart murmur at the age of 6 months, but it has not been noted on 11 aVL
examinations since the age of 1 year. His paternal grandfather died of a myocardial infarction 3 years ago at the age of 68 years. The patient takes
no medications. He does not smoke cigarettes, drink alcohol, or use illicit drugs. He performs well in school. He is 180 cm (5 ft 11 in) tall and
weighs 73 kg (161 lb); BMI is 23 kg/m2. His temperature is 37.0°C (98.6F), and blood pressure is 130/83 mm Hg. No murmurs are heard on
cardiac examination. The remainder of the examination shows no abnormalities. ECG is shown. Which of the following is the most likely
diagnosis?
111 aVF
0 A) Acute stress disorder
0 B) Delusional disorder
0 C) Generalized anxiety disorder
0 D) Hypertrophic cardiomyopathy
0 E) Idiopathic recurrent pericarditis
0 F ) Panic disorder
0 G) Paroxysmal supraventricular tachycardia

https://t.me/USMLENBME2CK ts e « t
Previous Next Lab Values Calculator Review Help pause
Exam Section 2: Item 21 of 50 National Board of Medical Examiners Time Remaining.
an Comprehensive Clinical Science Self-Assessment 4 hr 59 min 12 sec

21. A72-year-old man comes to the physician for a routine health maintenance examination. Abdominal examination shows a nontender, pulsatile mass in the epigastrium. Abdominal ultrasonography shows a
7.2-cm aneurysm. The physician reviews several studies of patients with abdominal aneurysms that are greater than 7 cm in size. The rupture rate is estimated to be 20% per year. Following rupture of an
abdominal aneurysm, one half of patients die before reaching a hospital. For patients who do reach a hospital, the perioperative mortality rate is approximately 50%. The mortality rate for patients who
undergo elective repair of an aneurysm is 5%. After discussing the risks and benefits of surgical repair of the aneurysm, the patient decides not to undergo an operation. Which of the following best
represents this patient's risk for mortality from rupture of the aneurysm during the next year?

OA) 5%
OB) 10%
Oc) 15%
Oo 20%
OE 50%

https://t.me/USMLENBME2CK ti
Previous Next Lab Values Calculator Review Help pause
Exam Section 2: Item 22 of 50 National Board of Medical Examiners Time Remaining:
an Comprehensive Clinical Science Self-Assessment 4 hr 59 min 11 sec

22. A 58-year-old man with an 8-year history of schizoaffective disorder, manic type, is brought to the clinic for a follow-up examination. Two weeks ago, he was hospitalized for a manic episode. Risperidone
was added to his medication regimen; the dosage was increased to the maximum recommended prior to discharge. Al that lime, neurologic examination showed mild cogwheel rigidity. There was no
evidence of tremors. Muscle strength was 5/5. He has no other history of serious medical or psychiatric illness. His other medications are valproic acid and clonazepam. Today, examination shows drooling,
masked facies, and decreased blinking. He has a slow, shuffling gait with decreased arm swing. The remainder of the examination shows no abnormalities. Mental status examination shows normal
speech and a mildly anxious mood. There is no evidence of psychosis. Which of the following is the most appropriate next step in management?

0 A) Addition of diphenhydramine to the medication regimen


0 B) Addition of lithium carbonate to the medication regimen
0 C) Cognitive behavioral therapy
0 D) Decreasing the dosage of risperidone
0 E) Increasing the dosage of clonazepam
0 F) Psychotherapy

https://t.me/USMLENBME2CK
ts e « t
Previous Next Lab Values Calculator Review Help pause
Exam Section 2: Item 23 of 50 National Board of Medical Examiners Time Remaining.
an Comprehensive Clinical Science Self-Assessment 4 hr 59 min 8 sec

23. A67-year-old man with metastatic lung cancer comes lo the physician because of malaise and lethargy during the past 3 days. Medications are ibuprofen and lisinopril. He appears cachectic but is
oriented to person, place, and time. His temperature is 37C (98.6F), pulse is 102/min, respirations are 13/min, and blood pressure is 105/75 mm Hg. Examination shows decreased skin turgor. The
remainder of the examination shows no abnormalities. Laboratory studies show:
Hemoglobin 10.6 g/dL
Hematocrit 36%
Leukocyte count 11,100/mm3
Platelet count 226,000/mm3
Serum
Na 136 mEq/L
K+ 3.6 mEq/L
Cl- 96 mEq/L
HCO. 3 28 mEq/L
Ca2+ 11.8 mg/dL
Urea nitrogen 22 mg/dL
Glucose 98 mg/dL
Creatinine 0.9 mg/dL
Phosphorus 2.5 mg/dL
Albumin 2.9 g/dL

Which of the following is the most appropriate initial pharmacotherapy?

0 A) Calcitonin
0 B) Hydrocortisone
0 C) Plicamycin
0 D) 0.9% Saline
0 E) Zoledronic acid

https://t.me/USMLENBME2CK ti e
Previous Next Lab Values Calculator Review Help pause
Exam Section 2: Item 24 0f 50 National Board of Medical Examiners Time Remaining.
an Comprehensive Clinical Science Self-Assessment 4 hr 59 min 6 sec

24. A 4 7-year-old man is brought to the emergency department because of severe left groin pain since he fell 2 hours ago. He is unable to walk because of the pain. He does not have pain in other joints. He
has a 20-year history of ulcerative colitis. His only medication is daily prednisone. On examination, any attempts at range of motion of the left hip elicit pain. Muscle strength is intact distally and in the right
lower extremity. Distal pulses are 2+. An x-ray of the pelvis and left lower hip shows moderate loss of joint space and diffuse osteopenia. Which of the following is the most appropriate next step in
management?

0 A) Observation only
0 B) DEXA scan
Oo MRI of the hip
0 0) Nonsteroidal anti-inflammatory drug therapy
( E) Aspiration of the hip joint

0 F) Total hip arthroplasty

https://t.me/USMLENBME2CK ti e
Previous Next Lab Values Calculator Review Help pause
Exam Section 2: Item 25 of 50 National Board of Medical Examiners Time Remaining.
an Comprehensive Clinical Science Self-Assessment 4 hr 59 min 5 sec

25. A 74-year-old woman is brought to the clinic by her family because of a 7-month history of progressive confusion and urinary incontinence. During the past year, she has had multiple falls; she sustained
wrist fractures 1 year ago and a left tibia fracture 3 months ago. She has hypertension, osteoporosis, and depression. She is evasive when asked how much alcohol she drinks. She appears frail. Vital
signs are within normal limits. Physical examination shows a wide-based gait and marked ataxia. She is unable to attempt the Romberg test. Cranial nerve examination shows nystagmus and an inability to
move her eyes horizontally. On cognitive testing, she recalls zero of three objects after 5 minutes. Laboratory studies show:
Hemoglobin 10 g/dl
Mean corpuscular volume 102 m?
Serum
Vitamin B, (cobalamin) 980 pg/mL (N=160--950)
Folate 3.1 ng/mL (N=2.7-17)

Which of the following is the most likely diagnosis?

0 A) Felic acid deficiency


0 B) Lewy body dementia
0 C) Normal-pressure hydrocephalus
0 D) Progressive supranuclear palsy
0 E) Wernicke-Korsakoff syndrome

https://t.me/USMLENBME2CK
ts e « t
Previous Next Lab Values Calculator Review Help pause
Exam Section 2: ltem 26 of 50 National Board of Medical Examiners Time Remaining.
an Comprehensive Clinical Science Self-Assessment 4 hr 59 min 3 sec

26. Eight days after he was involved in a motor vehicle collision in which he was the restrained driver, a hospitalized 28-year-old man has hypertension. He underwent extensive reconstructive orthopaedic
operations for injuries to his left clavicle, shoulder, and femur and exploratory laparotomy for a ruptured spleen. Urine toxicology screening on admission was positive for A-tetrahydrocannabinol, cocaine,
and opiates. He was intubated and mechanically ventilated during the procedures and was extubated on hospital day 4. He receives oral acetaminophen-oxycodone as needed. His blood pressure has
been 180/110 mm Hg; ii decreases to 150/80 mm Hg when he receives his pain medication. An abdominal bandage covers the surgical incision. The nurse tells the physician that the patient rates his pain
as a 10on a 10-point scale and is constantly asking for pain medications. Which of the following is the most appropriate next step in pharmacotherapy for this patient's hypertension?

0 A) Continued administration of acetaminophen-oxycodone every 4 hours


0 B) Continuous intravenous labetalol infusion with titration
0 C) Intravenous nitroprusside therapy
0 D) Patient-controlled morphine
0 E) Switching from acetaminophen-oxycodone to tramadol

https://t.me/USMLENBME2CK ti
Previous Next Lab Values Calculator Review Help pause
Exam Section 2: ltem 27 of 50 National Board of Medical Examiners Time Remaining.
an Comprehensive Clinical Science Self-Assessment 4 hr 59 min 1 sec

27. A 37-year-old primigravid woman at 16 weeks' gestation comes to the physician to discuss the results of amniocentesis. She has an IQ of 70. She is employed as a housekeeping assistant in a hotel and
has been married to a coworker for 6 months. She has no family history of serious illness. Karyotype analysis of the fetus showed trisomy 21. Her pregnancy has been otherwise uncomplicated. She is
informed of the abnormalities associated with the fetal genotype and still wants to continue the pregnancy. Which of the following is the most appropriate next step in management?

0 A) Schedule the patient for ongoing prenatal care


0 B) Discuss the situation with the patient's husband
0 C) Try to convince the patient to change her decision
0 D) Seek a court-appointed guardian
0 E) Refer the patient for psychiatric evaluation to determine competence

https://t.me/USMLENBME2CK ti
Previous Next Lab Values Calculator Review Help pause
Exam Section 2: Item 28 of 50 National Board of Medical Examiners Time Remaining.
an Comprehensive Clinical Science Self-Assessment 4 hr 58 min 58 sec

28. A 29-year-old male US Navy veteran comes to the local Veterans Affairs clinic because of a 2-week history of purulent drainage from a small opening at the site of amputation of his left arm. For the past
10 days, he has had increasing pain, tenderness, and redness at the wound site. Thirteen months ago, the patient sustained a traumatic amputation of the left arm below the elbow when an improvised
explosive device detonated while he was on a combat mission in the Middle East. Management at that time consisted of aggressive surgical debridement of the residual limb, which has healed slowly; a
small part of the wound appeared to heal incompletely. The patient appears well. Vital signs are within normal limits. Examination of the left upper extremity shows a 3-mm opening in the scar with
surrounding erythema; the site is draining purulent material. The remainder of the examination shows no abnormalities. Which of the following studies is most likely to confirm the diagnosis?

0 A) Bone biopsy of the wound site


0 8) Complete blood count with differential
0 C) Erythrocyte sedimentation rate and serum C-reactive protein concentration
0 D) X-ray of the left upper extremity residual limb

https://t.me/USMLENBME2CK ti e
Previous Next Lab Values Calculator Review Help pause
Exam Section 2: Item 29 of 50 National Board of Medical Examiners Time Remaining.
an Comprehensive Clinical Science Self-Assessment 4 hr 58 min 56 sec

29. A 2-year-old girl is brought to the physician because of a 2-week history of difficulty walking and intermittent crossing of the eyes. She also has had three to four episodes of watery diarrhea daily during
this time. She has not had fever or vomiting. She has no history of serious illness and receives no medications. She is alert but irritable and tearful; she clings to her mother during the examination. She is
at the 50th percentile for length, 25th percentile for weight, and 50th percentile for head circumference. Her pulse is 100/min, respirations are 20/min, and blood pressure is 120/80 mm Hg. The abdomen is
soft and nontender; a smooth, firm mass is palpated over the right upper quadrant. Neurologic examination shows rapid, chaotic, multidirectional saccades of the eyes. There are Jerking motions of the
extremities, which are exacerbated by intentional movement. There is truncal ataxia, and the patient is unable to sit or walk without assistance. Which of the following is the most likely underlying cause of
this patient's symptoms?

0 A) Hepatoblastoma
0 B) Medulloblastoma
0 C) Nephroblastoma (Wilms tumor)
0 D) Neuroblastoma
0 E) Non-Hodgkin lymphoma
0 F ) Rhabdomyosarcoma

https://t.me/USMLENBME2CK ti e
Previous Next Lab Values Calculator Review Help pause
Exam Section 2: Item 30 of 50 National Board of Medical Examiners Time Remaining.
an Comprehensive Clinical Science Self-Assessment 4 hr 58 min 54 sec

30. Three days after admission to the hospital for acute bacterial endocarditis, a 62-year-old man has shortness of breath that began 5 minutes ago while walking for the first time since admission. On
admission, he had a 10-day history of fever; examination showed splinter hemorrhages and normal heart sounds without murmurs. His current pulse is 110/min, respirations are 24/min, and blood pressure
is 100/50 mm Hg. Crackles are heard over both lung bases. S,and S,are soft; there is an S A new grade 2/6, early diastolic murmur is heard at the apex and the left sternal border, and a grade 4/6,
systolic ejection murmur is heard best in the carotids and at the left sternal border. Which of the following is the most appropriate next step in management?

0 A) Echocardiography
0 B) Ventilation-perfusion lung scans
0 C) Intravenous furosemide therapy
0 D) Intravenous heparin therapy
0 E) Sublingual nitroglycerin therapy

https://t.me/USMLENBME2CK ti
Previous Next Lab Values Calculator Review Help pause
Exam Section 2: Item 31 of 50 National Board of Medical Examiners Time Remaining.
an Comprehensive Clinical Science Self-Assessment 4 hr 58 min 52 sec

31. A study is proposed to investigate the effectiveness of a third-generation cephalosporin for the treatment of streptococcal pharyngitis. The study will include children between the ages of 5 and 10 years.
They will be randomly assigned to receive treatment with a cephalosporin, penicillin, or a placebo. Informed consent will be obtained from one parent. The principal investigator offers a $50 finder's fee to
any medical resident who identifies a possible study participant. Which of the following features of this study is of greatest potential concern?

0 A) Consent only obtained from one parent


0 B) Use of finder's fees to recruit subjects
0 C) Use of minors as subjects
0 D) Use of placebo as treatment

https://t.me/USMLENBME2CK ti
Previous Next Lab Values Calculator Review Help pause
Exam Section 2: ltem 32 of 50 National Board of Medical Examiners Time Remaining.
an Comprehensive Clinical Science Self-Assessment 4 hr 58 min 51 sec

32. A 27-year-old woman comes to the physician for a follow-up examination. She has a 5-month history of temperatures to 38.3°C (101F) and a cough productive of phlegm and mucous plugs. She has
severe asthma treated with high doses of prednisone. She has had numerous visits to the emergency department since childhood because of uncontrolled symptoms. Expiratory wheezes are heard
bilaterally. Laboratory studies show:
Hemoglobin 14.6 g/dL
Leukocyte count 8700/mm3
Segmented neutrophils 60%
Eosinophils 18%
Lymphocytes 22%
Serum lgE 23,000 IU/mL

An x-ray of the chest shows patchy bilateral upper lobe infiltrates that have shifted in location since an x-ray was taken 4 months ago. A CT scan of the chest shows central bronchiectasis without
lymphadenopathy. This patient is most likely to have an immediate response to which of the following on skin testing?

0 A) Animal dander
0 B) Aspergillus fumigatus
0 C) Bacillus subtilis
0 D) Cladosporium species
0 E) Isocyanate
0 F) Thermophilic actinomycetes

https://t.me/USMLENBME2CK
ts e « t
Previous Next Lab Values Calculator Review Help pause
Exam Section 2: ltem 33 of 50 National Board of Medical Examiners Time Remaining.
an Comprehensive Clinical Science Self-Assessment 4 hr 58 min 49 sec

33. A4-year-old boy undergoes splenectomy lo control bleeding from injuries sustained in a motor vehicle collision. His postoperative course is uncomplicated, and he receives pneumococcal and
meningococcal vaccinations in preparation for his discharge home. After discharge, which of the following is the most appropriate follow-up intervention to decrease this patient's risk for subsequent
infection?

0 A) Avoidance of influenza vaccinations


0 B) Ciprofloxacin prophylaxis
0 C) Folic acid supplementation
0 D) Penicillin prophylaxis
0 E) No further intervention is indicated

https://t.me/USMLENBME2CK ti
Previous Next Lab Values Calculator Review Help pause
Exam Section 2: Item 34 of 50 National Board of Medical Examiners Time Remaining.
an Comprehensive Clinical Science Self-Assessment 4 hr 58 min 47 sec

34. A 22-year-old nulligravid woman with cystic fibrosis comes to the physician because she would like to conceive within the next year. She has not been hospitalized in the past 3 years. Her disease is well
controlled with antibiotics, vigorous bronchial drainage, bronchodilators, and nutritional support including pancreatic enzyme replacement. She is 155 cm (5 ft 1 in) tall and weighs 38 kg (83 lb); BMI is 16
kg/m2. Her pulse is 96/min, and blood pressure is 138/86 mm Hg. Examination shows no other abnormalities except for a prolonged expiratory phase. Her husband is healthy and has no family history of
cystic fibrosis. She should be told that she will most likely require which of the following during pregnancy?

0 A) Additional antibiotic therapy


0 B) Insulin therapy
0 C) Nutritional supplementation
0 D) Oxygen therapy
0 E) Multiple hospital admissions

https://t.me/USMLENBME2CK
ts e « t
Previous Next Lab Values Calculator Review Help pause
Exam Section 2: ltem 35 of 50 National Board of Medical Examiners Time Remaining.
an Comprehensive Clinical Science Self-Assessment 4 hr 58 min 45 sec

35. A 6-month-old girl is brought to the emergency department by her parents because of a 2-day history of temperatures to 39.4°C (103.0F), irritability, decreased oral intake, and vomiting. She has no
history of serious illness and receives no medications. She has not received any vaccinations. She appears acutely ill and is lethargic. Temperature is 38.9C(102.0F), pulse is 170/min, respirations are
60/min, and blood pressure is 70/45 mm Hg. Pulse oximetry on room air shows an oxygen saturation of 95%. Examination discloses a bulging fontanel and decreased responsiveness. Lumbar puncture is
performed. Results of cerebrospinal fluid analysis are shown:
RBC 2/mm3
WBC 800/mm3
Neutrophils 88%
Monocytes 12%
Glucose 15 mg/dL
Protein 150 mg/dL

Broad-spectrum antibiotic and corticosteroid therapy is initiated. Which of the following is the most likely long-term complication of this patient's condition?

0 A) Epidural abscess
0 B) Hearing loss
0 C) Obstructive hydrocephalus
0 D) Seizure disorder
0 E) Subdural effusion

https://t.me/USMLENBME2CK
ts e « t
Previous Next Lab Values Calculator Review Help pause
Exam Section 2: Item 36 of 50 National Board of Medical Examiners Time Remaining.
an Comprehensive Clinical Science Self-Assessment 4 hr 58 min 43 sec

36. A 16-year-old girl is brought to the physician by her mother because she has never had a menstrual period. She is 147 cm (4 ft 10 in) tall and weighs 52 kg (115 lb); BMI is 24 kg/m2. Her temperature is
37.1C(98.8°F), pulse is 70/min, respirations are 14/min, and blood pressure is 110/60 mm Hg. Breast and pubic hair development is sexual maturity rating stage 1, and there is scant axillary hair. Pelvic
examination shows no abnormalities. Her serum follicle-stimulating hormone concentration is 62 mlU/mL. Which of the following is the most appropriate next step in management?

0 A) Observation only
0 B) Karyotype analysis
0 C) Hysterosalpingography
0 D) MRI of the sella turcica
0 E) Gonadotropin-releasing hormone stimulation test

https://t.me/USMLENBME2CK
ts e « t
Previous Next Lab Values Calculator Review Help pause
Exam Section 2: Item 37 of 50 National Board of Medical Examiners@ Time Remaining:
i ark Comprehensive Clinical Science Self-Assessment 4hr58min42 sec

37. A 7-year-old boy is brought to the physician because of a 4-day history of a dry, mildly itchy rash. The rash began as a small, pink, oval area on
his chest and progressed to scaly, red patches over his arms, legs, and body. His mother applied calamine lotion and a topical corticosteroid
cream without relief. He is currently taking amoxicillin for streptococcal pharyngitis diagnosed 4 days ago. He has no other history of rash or
serious illness. He is alert and cooperative and in no acute distress. The pharynx is pink, and the tonsils are slightly enlarged with no exudate.
The neck is supple with no adenopathy. A photograph of the rash over the trunk is shown. Which of the following is the most likely diagnosis?

0 A) Drug allergy
0 B ) Pityriasis rosea
QC) Scarlet fever
0 D) Seborrheic dermatitis
0 E) Tinea corporis

https://t.me/USMLENBME2CK ts e « t
Previous Next Lab Values Calculator Review Help pause
Exam Section 2: Item 38 of 50 National Board of Medical Examiners Time Remaining.
an Comprehensive Clinical Science Self-Assessment 4 hr 58 min 40 sec

38. A 3-month-old girl is brought to the physician because of constipation. She was born at home at term following an uncomplicated pregnancy and weighed 3118 g (6 lb 14 oz). She has not had routine well-
child examinations. She is unable to hold her head up, does not roll over, and does not smile. She is at the 10th percentile for length, 5th percentile for weight, and 75th percentile for head circumference.
The posterior fontanel is 2 cm in diameter. The tongue is large and protuberant. Which of the following is most likely to confirm the diagnosis?

OA Diel history
0 B) Thyroid function tests
0 C) Upper gastrointestinal series
0 D) Cranial ultrasonography
0 E) Rectal biopsy

https://t.me/USMLENBME2CK ti
Previous Next Lab Values Calculator Review Help pause
Exam Section 2: ltem 39 of 50 National Board of Medical Examiners Time Remaining.
an Comprehensive Clinical Science Self-Assessment 4 hr 58 min 38 sec

39. A 32-year-old man comes to the office for a routine examination. He was diagnosed with HIV infection 7 years ago and began antiretroviral therapy 6 years ago. He received the appropriate immunizations
at the time of diagnosis. He has tolerated his medication regimen well. He used illicit intravenous drugs for 5 years but stopped 7 years ago. He is in a monogamous relationship with one female partner;
they use condoms consistently. Examination shows no abnormalities. His CD4+ T-lymphocyte count is 645/mm3 (Norma1>500), and plasma HIV viral load is 1000 copies/ml. Which of the following
immunizations is most appropriate to administer at this time?

0 A) Hepatitis A
0 B) Hepatitis B
0 C) Human papillomavirus
0 D) Meningococcal
0 E) Pneumococcal

https://t.me/USMLENBME2CK ti e
Previous Next Lab Values Calculator Review Help pause
Exam Section 2: Item 40 of 50 National Board of Medical Examiners Time Remaining.
an Comprehensive Clinical Science Self-Assessment 4 hr 58 min 36 sec

40. A24-year-old man with protein C deficiency comes to the physician because of a 1-week history of yellowing of his skin. Three months ago, he had a mesenteric infarction and required resection of all but
45 cm of the proximal small bowel. The remainder of the jejunum was anastomosed to the colon. He receives total parenteral nutrition at home. He takes no medications. Examination today shows
generalized jaundice. Laboratory studies show:
Leukocyte count 7000/mm3
Serum
Bilirubin, total 3.6 mg/dL
Direct 2.6 mg/dL
Alkaline phosphatase 150 U/L
AST 20 U/L
ALT 17 U/L

Which of the following is the most likely cause of this patient's hyperbilirubinemia?

0 A) Biliary stricture
0 B) Cholestatic hepatic disease
0 C) Hemolysis
0 D) Hepatitis A
0 E) Hepatitis C

https://t.me/USMLENBME2CK ti
Previous Next Lab Values Calculator Review Help pause
Exam Section 2: Item 41 of 50 National Board of Medical Examiners Time Remaining.
an Comprehensive Clinical Science Self-Assessment 4 hr 58 min 34 sec

41. Eight weeks after an uncomplicated vaginal delivery of a healthy full-term newborn, a 27-year-old woman, gravida 1, para 1, is brought to the physician by her husband because of a change in personality
and depressed mood over the past 10 days. She has not had headaches. Her menses have not resumed despite the discontinuation of breast-feeding 5 weeks ago. Her husband says that she has no
interest in sexual intercourse. She does not use illicit drugs. She has had a 2.7-kg (6-lb) weight gain since delivery. She is now 175 cm (5 ft 9 in) tall and weighs 74 kg (164 lb); BMI is 24 kg/m. Physical
and neurologic examinations show no abnormalities. On mental status examination, she has a flat affect and is slow to respond to questioning. She spontaneously cries during the interview and
examination. She has no suicidal ideation. Which of the following is the most appropriate next step in management?

OA MRI of the brain


0 B) Antipsychotic therapy
0 C) Cyclic progestin therapy
0 D) Estrogen therapy
0 E) Selective serotonin reuptake inhibitor therapy

https://t.me/USMLENBME2CK ti
Previous Next Lab Values Calculator Review Help pause
Exam Section 2: Item 42 of 50 National Board of Medical Examiners Time Remaining.
an Comprehensive Clinical Science Self-Assessment 4 hr 58 min 32 sec

42. A 32-year-old woman, gravida 3, para 2, at 37 weeks' gestation is admitted to the hospital in labor. Contractions occur every 3 minutes. She has gained 2.3 kg (5 lb) during the past week and has had
occasional blurred vision. She has been otherwise healthy. Her temperature is 37"C (98.6°F), pulse is 80/min, respirations are 20/min, and blood pressure is 150/110 mm Hg. Physical examination shows
retinal arteriolar spasm and pedal edema. The fundus is midway between the umbilicus and xiphoid. The cervix is 3 cm dilated and 100% effaced; the vertex is at O station. laboratory studies show:
Hemoglobin 12.5 g/dL
leukocyte count 8000/mm3
Platelet count 200,000/mm3
Serum creatinine 1.2 mg/dl
Urine protein 2+

Which of the following is the most likely complication during labor?

0 A) Abruptio placentae
0 B) Amniotic fluid embolism
0 C) Arrest of labor
0 D) Maternal death
0 E) Uterine rupture

https://t.me/USMLENBME2CK ti
Previous Next Lab Values Calculator Review Help pause
Exam Section 2: Item 43 of 50 National Board of Medical Examiners Time Remaining.
an Comprehensive Clinical Science Self-Assessment 4 hr 58 min 30 sec

43. A 32-year-old man is brought to the physician by his wife because of a gradual change in personality during the past 2 years. She describes him as less motivated and more self-centered. During the past
year, he has had several angry outbursts that were triggered by minor events. The patient says that he does not believe his personality has changed and that he does not recall these events. He is alert
and attentive. On examination, eye movements are full, but saccadic eye movements are noticeably slow. Smooth pursuit and vestibular ocular movements are normal. He has occasional involuntary
rapid, Jerk-like movements of the trunk and extremities, but he is unaware of these movements. The remainder of the examination shows no abnormalities. Which of the following is the most appropriate
next step in diagnosis?

OA) EEG
0 B) Follow-up examination in 1 month
0 C) Genetic testing
0 0) lumbar puncture
0 E) Measurement of serum ceruloplasmin concentration
0 F) Psychiatric assessment

https://t.me/USMLENBME2CK ti
Previous Next Lab Values Calculator Review Help pause
Exam Section 2: Item 44 of 50 National Board of Medical Examiners Time Remaining.
an Comprehensive Clinical Science Self-Assessment 4 hr 58 min 28 sec

44. A 12-year-old girl is brought to the physician by her mother because of a 2-monlh history of increasing, constant, throbbing pain and pressure in the pelvic area. She has no history of serious illness and
takes no medications. She has never had a menstrual period. She follows a normal diet and takes gymnastics lessons weekly. She is at the 50th percentile for height, weight, and BMI. Vital signs are
within normal limits. Breast and pubic hair development are sexual maturity rating (SMR) stage 3. Examination of the external genitalia shows a bluish bulge of tissue between a normal-appearing urethra
and anus. Serum studies show:
Follicle-stimulating hormone 12 mlU/mL
Luteinizing hormone 15 mlU/mL
Estradiol 50 pg/mL (N=30-400)
Prolactin 10 ng/mL

Which of the following is the most likely diagnosis?

0 A) Bartholin duct cyst


0 B) Cervical stenosis
0 C) lmperforate hymen
0 D) Ovarian cancer
0 E) Sarcoma botryoides

https://t.me/USMLENBME2CK
ts e « t
Previous Next Lab Values Calculator Review Help pause
Exam Section 2: Item 45 of 50 National Board of Medical Examiners Time Remaining.
an Comprehensive Clinical Science Self-Assessment 4 hr 58 min 26 sec

45. A 9-year-old girl is brought to the physician by her father for a well-child examination. He is concerned that she is beginning to develop breasts at an early age. Menarche has not occurred. Her mother's
menarche began at the age of 12 years. The patient is at the 75th percentile for height and 50th percentile for weight. Breast development is sexual maturity rating (SMR) stage 2, and pubic hair
development is SMR stage 1. The remainder of the examination shows no abnormalities. Which of the following is the most appropriate next step in management?

0 A) Reassurance
0 8) Measurement of serum estrogen concentration
0 C) Measurement of serum follicle-stimulating hormone concentration
0 D) X-ray of the wrist to assess bone age
0 E) Transvaginal ultrasonography
0 F) Pelvic examination

https://t.me/USMLENBME2CK ti
Previous Next Lab Values Calculator Review Help pause
Exam Section 2: Item 46 of 50 National Board of Medical Examiners Time Remaining.
an Comprehensive Clinical Science Self-Assessment 4 hr 58 min 24 sec

46. A57-year-old man comes to the physician because of a 1-week history of shortness of breath and swelling of his legs. He says his shortness of breath occurs most frequently when he lies down. During
the past month, he has had the sensation of abdominal fullness and pulling and has noticed a progressive increase in his belt size. Five years ago, he received the diagnosis of bleeding esophageal
varices and alcoholic cirrhosis. He underwent sclerosis of the varices and has abstained from alcohol since then. His temperature is 36.9°C (98.4°F). The abdomen is moderately tense and nontender with
shifting dullness. There is 3+ pitting edema of the lower extremities. Paracentesis is performed. Laboratory studies show:
Serum
Protein, total 2.8 g/dL
Albumin 1.8 g/dL
Peritoneal fluid
Leukocyte count 900/mm3
Segmented neutrophils 67%
Lymphocytes 28%
Protein, total 1.8 g/dL
Albumin 1 g/dl

Which of the following is the most appropriate next step in management?

0 A) Echocardiography
0 B) Ultrasonography of the hepatic veins
0 C) Antibiotic therapy
0 D) Laparoscopy
0 E) Liver biopsy
0 F) CT angiography of the liver

https://t.me/USMLENBME2CK ti
Previous Next Lab Values Calculator Review Help pause
Exam Section 2: Item 47 of 50 National Board of Medical Examiners Time Remaining.
an Comprehensive Clinical Science Self-Assessment 4 hr 58 min 22 sec

47. An asymptomatic 62-year-old man comes for a routine health maintenance examination. Physical examination shows splenomegaly. Laboratory studies show:
Hematocrit 42%
Leukocyte count 46,000/mm3
Segmented neutrophils 35%
Bands 15%
Basophils 1%
Lymphocytes 16%
Monocytes 5%
Myeloblasts 1%
Promyelocytes 2%
Metamyelocytes 13%
Myelocytes 12%
Platelet count 650,000/mm3

Leukocyte alkaline phosphatase activity is decreased. Examination of bone marrow shows many immature white blood cell precursors. Which of the following is the most likely diagnosis?

0 A) Agnogenic myeloid metaplasia


0 B) Chronic myelogenous leukemia
0 C) Chronic myelomonocytic leukemia
0 D) Erythroleukemia
0 E) Essential thrombocytosis
( F) Myelodysplasia
( G) Polycythemia vera

https://t.me/USMLENBME2CK
ts e « t
Previous Next Lab Values Calculator Review Help pause
Exam Section 2: Item 48 of 50 National Board of Medical Examiners Time Remaining.
an Comprehensive Clinical Science Self-Assessment 4 hr 58 min 20 sec

48. A 5-year-old boy is brought to the office for a follow-up examination 2 days after he was discharged from the hospital following an episode of pyelonephritis. Medications are cefixime and acetaminophen.
He is at the 50th percentile for height and 25th percentile for weight; BMI is at the 10th percentile. Temperature is 37.6°C (99.6°F), pulse is 120/min, respirations are 22/min, and blood pressure is 90/60
mm Hg. Physical examination shows no other abnormalities. Results of laboratory studies from his hospital admission showed a leukocyte count within the reference range and urine positive for leukocyte
esterase; urine culture grew Klebsiel/a pneumoniae susceptible to cephalosporins. Kidney ultrasonography showed mild hydronephrosis. Which of the following is the most appropriate next step in
management?

0 A) Continuous antibiotic prophylaxis


0 B) Kidney and bladder ultrasonography
0 C) Renal scintigraphy using dimercaptosuccinic acid (DMSA)
0 D) Repeat urine culture
0 E) Voiding cystourethrography

https://t.me/USMLENBME2CK
ts e « t
Previous Next Lab Values Calculator Review Help pause
Exam Section 2: Item 49 of 50 National Board of Medical Examiners@ Time Remaining:
i ark Comprehensive Clinical Science Self-Assessment 4hr58min 18 sec

49. A 16-year-old girl is brought to the emergency department immediately after she collapsed on her way to the bathroom at home. She has anorexia nervosa and has had an 18-kg (40-lb) weight loss during
the past 16 months. Menarche was at the age of 11 years, but her last menstrual period was 14 months ago. She does not drink alcohol or use illicit drugs. On arrival, she appears cachectic. She is
165 cm (5 fl 5 in) tall and weighs 36 kg (80 lb); BMI is 13 kg/m2. Her temperature is 36.4"C (97.6°F), pulse is 40/min, respirations are 18/min, and blood pressure is 85/55 mm Hg. Physical examination
shows fine hair over the skin. An ECG shows ST-segment depression and an increased U wave. This patient is most likely to have which of the following sets of serum electrolyte findings?

Na° K" Cl HCO-3


(mEq/L) (mEq/L) (mEq/L) (mEq/L)
OA 125 3.2 102 25
OB 125 6.8 102 27
Oo» 140 2.3 94 28
OD 140 6.8 94 28
OE 155 3.2 102 26

https://t.me/USMLENBME2CK ts e « t
Previous Next Lab Values Calculator Review Help pause
Exam Section 2: Item 50 of 50 National Board of Medical Examiners Time Remaining.
an Comprehensive Clinical Science Self-Assessment 4 hr 58 min 16 sec

50. A 20-hour-old female newborn was noted to have jaundice 1 hour ago. She was born at term to a 29-year-old woman, gravida 2, para 2, following an uncomplicated pregnancy and spontaneous vaginal
delivery. The mother emigrated from Ethiopia 2 years ago. She received routine prenatal care. Examination of the patient shows jaundice to the level of the umbilicus. There is conjunctival icterus. No other
abnormalities are noted. The maternal blood group is 0, Rh-positive. Results of laboratory studies, including the patient's blood group, are pending. Which of the following laboratory findings is most likely
to be decreased in this patient?

0 A) Hematocrit
0 B) leukocyte count
0 C) Platelet count
0 D) Serum direct bilirubin concentration
0 E) Serum indirect bilirubin concentration

https://t.me/USMLENBME2CK ti
Previous Next Lab Values Calculator Review Help pause
Exam Section 3: Item 1 of 50 National Board of Medical Examiners Time Remaining.
an Comprehensive Clinical Science Self-Assessment 4 hr 59 min 52 sec

1. A 48-year-old man is admitted to the hospital for management of acute respiratory failure. He has a 4-day history of fever, an increase in thick, yellow-green sputum production, and diffuse muscular aches.
During the past 2 years, he has produced 2 to 3 tablespoons of yellow sputum daily. He has smoked two packs of cigarettes daily for 30 years. He is 183 cm (6 ft) tall and weighs 134 kg (295 lb); BMI is
40 kg/m?. On admission, his temperature is 38.6°C (101.5°F), pulse is 120/min, respirations are 30/min, and blood pressure is 170/100 mm Hg. Pulse oximetry on room air shows an oxygen saturation of
60%. On admission, an AP portable x-ray shows bilateral diffuse interstitial infiltrates without consolidation. He does not respond to high-flow oxygen therapy. He is intubated and mechanically ventilated.
Examination shows hyperresonance to percussion and wheezes bilaterally. Ceftriaxone and levofloxacin are administered, and his symptoms improve. On hospital day 4, he is extubated. His temperature is
37.6°C (99.7°F), pulse is 100/min, respirations are 22/min, and blood pressure is 150/96 mm Hg. Pulse oximetry while sitting shows an oxygen saturation greater than 95%. Six hours later, the patient's
oxygen saturation is 86%. The patient is lying in bed and says he feels fine. Physical examination shows no wheezing, crackles, or rhonchi. There is no peripheral edema. A repeat chest x-ray shows
improvement. Which of the following is the most appropriate next step in management to improve this patient's oxygen saturation?

0 A) Increase the oxygen concentration of inspired air


0 B) Increase the positive end-expiratory pressure
0 C) Place the patient in Trendelenburg position
0 D) Place the patient's head on three pillows
0 E) Return the patient to a sitting position

https://t.me/USMLENBME2CK ti
Next Lab Values Calculator Review Help Pause
Exam Section 3: Item 2 of 50 National Board of Medical Examiners Time Remaining.
an Comprehensive Clinical Science Self-Assessment 4 hr 59 min 51 sec

2. A 24-year-old female medical student sustains a deep laceration of her left index finger while assisting the surgeon during an operation on a patient with cervical cancer. The surgical patient has a history of
multiple sexual partners and intravenous drug use; it is not yet known if she is HIV positive. Examination of the medical student shows a 2-cm, bleeding laceration; the bleeding stops with compression.
Which of the following is the most appropriate next step in management pending results of HIV testing on the surgical patient?

0 A) Acyclovir therapy
0 B) Antiretroviral therapy
0 C) Corticosteroid therapy
0 D) Nafcillin therapy
0 E) No treatment is indicated

https://t.me/USMLENBME2CK ti
Previous Next Lab Values Calculator Review Help pause
Exam Section 3: ltem 3 of 50 National Board of Medical Examiners Time Remaining.
an Comprehensive Clinical Science Self-Assessment 4 hr 59 min 49 sec

3. A 10-month-old female infant is brought to the office by her parents for a follow-up examination 3 weeks after undergoing staged repair of a left complete unilateral cleft lip and palate. The procedure and
recovery were uncomplicated, and the infant was discharged home 2 days later. The parents report she is feeding well with a traditional bottle nipple and now also consumes some soft baby foods. She has
not had fever, pain, or nasal regurgitation. She receives no medications. Today, she is at the 25th percentile for length and weight. Vital signs are normal. Physical examination discloses a well-healing
surgical incision on the left upper vermilion and philtrum; the palate also appears to be healing well without wound dehiscence or palatal holes. This patient's parents should be informed that she is at
increased risk for developing which of the following?

0 A) Chronic colonization with group A streptococcus


0 B) Dyslexia
0 C) Peritonsillar abscess
0 D) Severe myopia
0 E) Velopharyngeal insufficiency

https://t.me/USMLENBME2CK ti
Previous Next Lab Values Calculator Review Help pause
Exam Section 3: Item 4 of 50 National Board of Medical Examiners Time Remaining.
an Comprehensive Clinical Science Self-Assessment 4 hr 59 min 48 sec

4. A 22-year-old man is brought to the physician by his parents because he has become newly preoccupied with religion during the past year. He lives with his parents and attends a community college.
Two months ago, he stopped attending his classes. His parents report that he spends most of the day in his room alone reading religious scripture. He shows no interest in activities he once enjoyed, no
longer spends time with his friends, masturbates frequently and obviously, and spends most of the night walking around his room laughing and talking to himself. He has no history of serious illness and
takes no medications. Vital signs are within normal limits. Physical examination shows no abnormalities. On mental status examination, he has a depressed mood and animated affect. His speech is
pressured, and responses to questions are tangential. He reports hearing a voice during the past 2 months that he believes is "the voice of God." He says that he feels confused and has thoughts about
ending his life because he cannot understand if God truly wants him to become the next Messiah. Which of the following is the most likely diagnosis?

0 A) Delusional disorder
0 B) Obsessive-compulsive disorder
0 C) Schizoid personality disorder
0 D) Schizophrenia
0 E) Schizotypal personality disorder

https://t.me/USMLENBME2CK
ts e « t
Previous Next Lab Values Calculator Review Help pause
Exam Section 3: Item 5 of 50 National Board of Medical Examiners Time Remaining.
an Comprehensive Clinical Science Self-Assessment 4 hr 59 min 46 sec

5. A 26-year-old primigravid woman has spontaneous rupture of the membranes at 30 weeks' gestation. She is afebrile and has no contractions. A sterile speculum examination shows a closed cervix and a
pool of clear fluid in the vagina. Which of the following is the most likely complication?

0 A) Amniotic band syndrome


0 B) Fetal deformation
0 C) Fetal hypoplastic lungs
0 D) Preterm delivery
0 E) Umbilical cord occlusion

https://t.me/USMLENBME2CK ti
Previous Next Lab Values Calculator Review Help pause
Exam Section 3: Item 6 of 50 National Board of Medical Examiners@ Time Remaining:
i ark Comprehensive Clinical Science Self-Assessment 4hr59min44 sec

6. An 8-year-old African American girl with sickle cell disease is brought to the physician by her mother because of a 12-hour history of cough and difficulty breathing. Her medications are penicillin and folic
acid. Her temperature is 39C (102.2F), pulse is 130/min, respirations are 40/min, and blood pressure is 105/60 mm Hg. Pulse oximetry on room air shows an oxygen saturation of 94%. On pulmonary
examination, crackles are heard in the upper lobes. Breath sounds are decreased at the lung bases bilaterally. The remainder of the examination shows no abnormalities. An x-ray of the chest is shown.
Which of the following is the most appropriate next step in diagnosis?

0 A) Anti-Mycop/asma pneumoniae lgG titer


0 B) Rapid antigen detection test for respiratory syncytial virus
0 C) Blood culture
0 D) Spiral CT scan of the chest
0 E) Thoracentesis

https://t.me/USMLENBME2CK ts e « t
Previous Next Lab Values Calculator Review Help pause
Exam Section 3: Item 7 of 50 National Board of Medical Examiners Time Remaining.
an Comprehensive Clinical Science Self-Assessment 4 hr 59 min 42 sec

7. A 58-year-old man is admitted to the hospital for treatment of anemia. Two days ago, he had excessive blood loss following colon polyp removal via colonoscopy. Today, his hemoglobin concentration is 6.5
g/dl and hematocrit is 19.5%. Two months ago, in order to decrease transfusion errors, a blood lock system was implemented in which the patient ID must be entered into the lock to open the blood
container; the container cannot be opened without the correct ID. Requiring staff to enter the patient's ID to open the blood lock is best described as which of the following?

0 A) An affordance
0 B) A forcing function
0 C) An interruption in care
0 D) Mapping of the design of the blood container
0 E) A policy leading to delays in care

https://t.me/USMLENBME2CK ti
Previous Next Lab Values Calculator Review Help pause
Exam Section 3: Item 8 of 50 National Board of Medical Examiners Time Remaining.
an Comprehensive Clinical Science Self-Assessment 4 hr 59 min 40 sec

8. Twenty-four hours after admission to the hospital for treatment of temperatures to 38.5°C (101.3°F) and abdominal pain, a 22-year-old man has a decrease in his temperature. Two weeks ago, he went on a
5-day trip to South America and had diarrhea for 7 days. He has no previous history of serious illness. On admission, a CT scan of the abdomen showed a 5-cm mass in the right lobe of the liver; the mass
was filled with fluid. He has received metronidazole, ampicillin, and gentamicin since admission. His current temperature is 37.4°C (99.3°F), and blood pressure is 120/75 mm Hg. The abdomen is flat with
moderate right upper quadrant tenderness and guarding. There are no peritoneal signs. A serum titer for antibodies to Entamoeba histolytica is positive at 1:128. Which of the following is the most
appropriate next step in management?

0 A) Continue the current antibiotic regimen for 10 days


0 B) Discontinue ampicillin and gentamicin and continue metronidazole
0 C) Perform examination of the stool for ova and parasites
0 D) Perform CT scan-guided aspiration of the mass
0 E) Perform surgical drainage of the mass

https://t.me/USMLENBME2CK ti
Previous Next Lab Values Calculator Review Help pause
Exam Section 3: Item 9 of 50 National Board of Medical Examiners@ Time Remaining:
i ark Comprehensive Clinical Science Self-Assessment 4hr59min 38 sec

9. A previously healthy 23-year-old woman is brought to the emergency department 20 minutes after the sudden onset of shortness of breath, moderate
abdominal pain, nausea, and an itchy rash over the abdomen. Her symptoms began after she ate a sugar cookie given to her by a friend. The patient
had a peanut allergy as a child and before eating the cookie, she asked her friend to taste it. Neither she nor her friend tasted or smelled peanuts. The
patient's only medication is an oral contraceptive. Her last menstrual period was 1 week ago. She has smoked one pack of cigarettes daily for 5 years.
She drinks three 12-oz beers on weekends. She has been staying at different friends' homes during the past 2 months while searching for an
apartment and has been using unfamiliar laundry detergents. She is in mild respiratory distress but is able to speak in full sentences. Her pulse is
110/min, respirations are 28/min, and blood pressure is 100/60 mm Hg. Pulse oximetry on room air shows an oxygen saturation of 94%. A photograph
of the rash is shown. Scattered wheezes are heard bilaterally. Cardiac examination shows no abnormalities. Abdominal examination shows mild,
diffuse tenderness to palpation; there is no rebound or guarding. Which of the following is the most appropriate initial pharmacotherapy?

OA Albuterol
0 B) Diphenhydramine
0 C) Epinephrine
0 D) Furosemide
0 E) Nitroglycerin
0 F) Prednisone

https://t.me/USMLENBME2CK ts e « t
Previous Next Lab Values Calculator Review Help pause
Exam Section 3: Item 10 of 50 National Board of Medical Examiners Time Remaining.
an Comprehensive Clinical Science Self-Assessment 4 hr 59 min 36 sec

10. A67-year-old man comes to the physician because he has noticed blood in his urine for the past 2 weeks. He has no prior history of blood in his urine. He has not had pain or any other symptoms. He has
hypertension controlled with a diuretic and hypercholesterolemia controlled with a lipid-lowering agent. His blood pressure is 120/76 mm Hg. Examination shows no abnormalities. Urinalysis shows RBCs
that are too numerous to count. Which of the following is the most appropriate next step in diagnosis?

0 A) Renal scintigraphy
0 B) Retrograde urethrography
Oo MRI of the pelvis
0 D) Cystoscopy
0 E) Renal biopsy

https://t.me/USMLENBME2CK ti
Previous Next Lab Values Calculator Review Help pause
Exam Section 3: Item 11 of 50 National Board of Medical Examiners@ Time Remaining:
i ark Comprehensive Clinical Science Self-Assessment 4hr59min 35 sec

11. A 57-year-old man comes to the emergency department because of a 3-week history of headaches that have been increasing in severity and frequency. He also has a 2-week history of progressive blurred
vision in both eyes. He has not had pain in the eyes. His last visit to a physician was 10 years ago because of pain in his fingers; at that visit, he was told he had high blood pressure. His temperature is
37C (98.6F), pulse is 90/min, respirations are 12/min, and blood pressure is 200/110 mm Hg. Repeat blood pressure measurements are 210/112 mm Hg and 214/114 mm Hg. Findings on funduscopic
examination are shown. Examination shows ulnar deviation of both hands and several swollen interphalangeal joints. His fasting serum glucose concentration is 160 mg/dl. Which of the following is the
most likely underlying cause of this patient's symptoms?

0 A) Diabetes mellitus
0 B) Glioblastoma multiforme
0 C) Hypertension
0 D) Idiopathic intracranial hypertension
0 E) Rheumatoid arthritis

https://t.me/USMLENBME2CK ts e « t
Previous Next Lab Values Calculator Review Help pause
Exam Section 3: Item 12 of 50 National Board of Medical Examiners Time Remaining.
an Comprehensive Clinical Science Self-Assessment 4 hr 59 min 32 sec

12. A case-control study is conducted to assess the effect of dietary fat intake on the occurrence of sudden cardiac death. One hundred fifty cases and controls are matched by age, sex assigned at birth, race,
and history of hypertension, hypercholesterolemia, and cigarette smoking. Dietary intake history of the cases is obtained from the next of kin. Results show an odds ratio of 1.6 with a 95% confidence
interval of 1.2 to 1.9. The authors conclude that there is a relationship between dietary fat intake and sudden cardiac death. Which of the following raises the most concern about the authors' conclusion?

0 A) Differential bias in assessing dietary fat


0 B) Inadequate sample size
Oo lack of blinding
0 D) Misclassification of cases and controls
0 E) Weak statistical association

https://t.me/USMLENBME2CK ti
Previous Next Lab Values Calculator Review Help pause
Exam Section 3: ltem 13 of 50 National Board of Medical Examiners Time Remaining.
an Comprehensive Clinical Science Self-Assessment 4 hr 59 min 30 sec

13. A67-year-old woman comes to the physician because of a 2-month history of hoarseness. She has not had any other symptoms. She has no history of serious illness and takes no medications. She has
smoked one pack of cigarettes daily for 45 years. Examination, including examination of the oropharynx, shows no abnormalities. Which of the following is the most appropriate next step in management?

0 A) Follow-up examination in 6 weeks


0 B) X-rays of the maxillary sinuses
0 C) CT scan of the chest
0 D) Bronchodilator therapy
0 E) Indirect laryngoscopy

https://t.me/USMLENBME2CK ti
Previous Next Lab Values Calculator Review Help pause
Exam Section 3: Item 14 of 50 National Board of Medical Examiners Time Remaining.
an Comprehensive Clinical Science Self-Assessment 4 hr 59 min 28 sec

14. A57-year-old man is admitted lo the hospital because of fever, shortness of breath, and cough productive of yellow sputum. He has prostate cancer metastatic lo the bone; his only symptom from the
cancer is occasional back pain well controlled with oral acetaminophen and hydrocodone. At his last appointment 6 months ago, his serum prostate-specific antigen assay was stable, and his disease had
not progressed. He plays golf several times each week and enjoys traveling with his wife. His advance directive states that he does not want intubation or mechanical ventilation unless he has an acute
illness with a reasonable chance of recovery to a good quality of life. He is lethargic and unable to communicate. His temperature is 38.9C (102F), pulse is 100/min, respirations are 30/min, and blood
pressure is 148/92 mm Hg. Pulmonary examination shows shallow, rapid breathing. He is using accessory muscles of respiration. lnspiratory crackles are heard at the right upper and lower lung fields.
Arterial blood gas analysis on 100% oxygen shows:
pH 7.36
Pco, 45 mm Hg
Po, 42 mm Hg

An x-ray of the chest shows diffuse right-sided infiltrates. Which of the following is the most appropriate next step in management?

0 A) Consult the hospital ethics committee


0 B) Begin noninvasive positive pressure ventilation by face mask
0 C) Continue oxygen by face mask and administer intravenous morphine for comfort
0 D) Intubate and begin mechanical ventilation

https://t.me/USMLENBME2CK ti
Previous Next Lab Values Calculator Review Help pause
Exam Section 3: Item 15 of 50 National Board of Medical Examiners Time Remaining.
an Comprehensive Clinical Science Self-Assessment 4 hr 59 min 26 sec

15. A 77 -year-old man comes to the physician because of a 3-day history of passing air and fecal material during urination. He has a long-standing history of intermittent constipation. His last examination was
20 years ago. He has smoked two packs of cigarettes daily for 55 years. He is 183 cm (6 ft) tall and weighs 70 kg (155 lb); BMI is 21 kg/m2. Vital signs are within normal limits. Abdominal examination
shows mild distention. Bowel sounds are decreased. Rectal examination shows a diffusely enlarged prostate. There is no stool in the rectal vault. Laboratory studies show:
Hemoglobin 14 gldl
Leukocyte count 9600/mm3
Platelet count 444,000/mm3
Urine
WBC >100/hpf
Flora mixed

Which of the following is the most likely cause of this patient's symptoms?

0 A) Appendiceal abscess
0 B) Diverticular disease
0 C) Inflammatory bowel disease
0 D) Prostate cancer
0 E) Transitional cell carcinoma of the bladder

https://t.me/USMLENBME2CK ti
Previous Next Lab Values Calculator Review Help pause
Exam Section 3: Item 16 of 50 National Board of Medical Examiners Time Remaining.
an Comprehensive Clinical Science Self-Assessment 4 hr 59 min 23 sec

16. A 25-year-old man comes to the physician 2 hours after the sudden onset of sharp right buttock pain that radiates to the back of his knee. The pain worsens when he sits for a prolonged period or drives.
He is able to walk. Use of over-the-counter ibuprofen has provided mild relief. He has no history of serious illness and takes no other medications. On examination, ipsilateral and conlralateral straight-leg
raise testing is positive. Muscle strength is normal throughout. Sensation is intact throughout. Which of the following is the most appropriate next step in diagnosis?

0 A) Electromyography and nerve conduction studies


0 B) MRI of the lumbosacral spine
0 C) Myelography
0 D) X-ray of the lumbosacral spine
0 E) No further testing is indicated

https://t.me/USMLENBME2CK ti
Previous Next Lab Values Calculator Review Help pause
Exam Section 3: Item 17 of 50 National Board of Medical Examiners Time Remaining.
an Comprehensive Clinical Science Self-Assessment 4 hr 59 min 21 sec

17. A 16-year-old girl is brought to the physician by her mother because of an 8-month history of irregular menstrual periods, acne, and facial hair. Menarche was at the age of 12 years. Menses occur at
irregular 35- to 42-day intervals; they last 1 week, and the flow is heavy. Moderate cramps on the first 2 days are relieved with ibuprofen therapy. The patient had recurrent urinary tract infections until the
age of 10 years. She has no other history of serious illness and takes no other medications. She is 163 cm (5 ft 4 in) tall and weighs 77 kg (170 lb); BMI is 29 kg/m2. Her temperature is 36.9C(98.4F),
pulse is 78/min, respirations are 20/min, and blood pressure is 128/84 mm Hg. Examination shows scattered papules, pustules, and comedones over the face. There is sparse, coarse hair over the chin.
The neck is supple; there is no thyromegaly. Breast examination shows no masses. Breast and pubic hair development are sexual maturity rating (SMR) stage 5. Serum studies are most likely to show an
increased concentration of which of the following?

0 A) Follicle-stimulating hormone
0 B) 11a-Hydroxyprogesterone
0 C) 21a-Hydroxyprogesterone
0 D) luteinizing hormone
0 E) Thyroid-stimulating hormone

https://t.me/USMLENBME2CK
ts e « t
Previous Next Lab Values Calculator Review Help pause
Exam Section 3: Item 18 of 50 National Board of Medical Examiners Time Remaining.
an Comprehensive Clinical Science Self-Assessment 4 hr 59 min 19 sec

18. A49-year-old woman comes to the physician for preoperative examination before she undergoes cholecystectomy. During the past 3 years, she has had recurrent cholecystitis and multiple gallstones. She
has not had shortness of breath, difficulty swallowing, or hoarseness. She has rheumatoid arthritis with morning stiffness and widespread Joint swelling. Her medications are methotrexate, folic acid, and
prednisone. Vital signs are within normal limits. Examination shows nodules over the extensor aspect of the upper extremities. There is swelling of the wrists, metacarpophalangeal joints of the hands, and
knees; there are swan neck deformities of the fingers. The remainder of the examination shows no abnormalities. Which of the following is the most appropriate next step in management to decrease this
patient's risk for operative complications?

0 A) Discontinuation of methotrexate
0 B) Discontinuation of prednisone
0 C) Indirect laryngoscopy
0 D) Pulmonary function testing
0 E) X-rays of the cervical spine

https://t.me/USMLENBME2CK ti
Previous Next Lab Values Calculator Review Help pause
Exam Section 3: Item 19 of 50 National Board of Medical Examiners Time Remaining.
an Comprehensive Clinical Science Self-Assessment 4 hr 59 min 17 sec

19. A 32-year-old woman, gravida 1, para 1, comes to the physician because of spontaneous discharge from her nipples during the past 3 months. She discontinued breast-feeding her child 1 year ago. She
takes no medications. Menses occur at regular 28-day intervals. On physical examination, a milky white discharge can be expressed from both nipples. Examination shows no other abnormalities. Her
fasting serum prolactin concentration is 40 ng/mL. The most appropriate next step in diagnosis is measurement of which of the following serum concentrations?

0 A) Cortisol
0 B) Dopamine
0 C) Estrogen
0 D) Gonadotropin-releasing hormone
0 E) Thyroid-stimulating hormone

https://t.me/USMLENBME2CK ti e
Previous Next Lab Values Calculator Review Help pause
Exam Section 3: Item 20 of 50 National Board of Medical Examiners Time Remaining.
an Comprehensive Clinical Science Self-Assessment 4 hr 59 min 15 sec

20. A 57-year-old man comes to the physician for a follow-up examination. One year ago, he sustained a myocardial infarction. He has coronary artery disease, hypertension, and type 2 diabetes mellitus. His
medications are atorvastatin, lisinopril, metformin, metoprolol, and aspirin. He has smoked one-half pack of cigarettes daily for 25 years. He is 178 cm (5 ft 10 in) tall and weighs 97 kg (213 lb); BMI is 31
kg/m2. He feels well. His pulse is 62/min, and blood pressure is 145/78 mm Hg. Examination shows no other abnormalities. Laboratory studies show a hemoglobin A,, of 6.5% and serum LDL-cholesterol
concentration of 110 mg/dl. Serum electrolyte, urea nitrogen, glucose, and creatinine concentrations are within the reference ranges. The physician recommends smoking cessation. Addition of which of
the following is the most appropriate next step in pharmacotherapy?

0 A) Clonidine
0 8) Exenatide
0 C) Hydrochlorothiazide
0 D) Rosiglitazone
OE Sitagliptin
0 F) Spironolactone

https://t.me/USMLENBME2CK
ts e « t
Previous Next Lab Values Calculator Review Help pause
Exam Section 3: Item 21 of 50 National Board of Medical Examiners Time Remaining.
an Comprehensive Clinical Science Self-Assessment 4 hr 59 min 13 sec

21 . A 60-year-old woman with osteoarthritis of the right knee comes to the office to discuss treatment options, including total knee replacement. Medications are naproxen and 81-mg aspirin. Vital signs are
within normal limits. Examination shows edema of the right knee; range of motion of the right knee is decreased. Results of laboratory studies are within the reference ranges. The patient asks the
physician about the success and complication rates associated with total knee replacement at the local university hospital. Which of the following types of studies is most appropriate for the physician to
reference to answer this patient's question?

0 A) Case-control study
0 B) Cross-sectional study
0 C) Institutional case series
0 D) Randomized controlled trial

https://t.me/USMLENBME2CK
ts e « t
Previous Next Lab Values Calculator Review Help pause
Exam Section 3: Item 22 of 50 National Board of Medical Examiners Time Remaining.
an Comprehensive Clinical Science Self-Assessment 4 hr 59 min 12 sec

22. A 72-year-old woman comes to the office with her son because of a "strange mole" on her back that she first noticed 6 months ago. The patient's son says he thinks it has enlarged during the past month.
The patient says the mole is mildly itchy but not painful, and she has not had bleeding from the area. She has not had any weight loss or changes in appetite. Medical history is unremarkable, and she
takes no medications. The patient has spent a lot of time outside by her son's new swimming pool during the past summer months. Vital signs are within normal limits. A photograph of the lesion is shown.
The remainder of the examination shows no abnormalities. Which of the following is the most appropriate diagnostic study at this time?

0 A) Culture of the lesion


0 B) Full-thickness biopsy of the lesion
0 C) KOH testing of a scraping of the lesion
0 D) Shave biopsy of the lesion
0 E) No diagnostic study is indicated

https://t.me/USMLENBME2CK ti e
Previous Next Lab Values Calculator Review Help pause
Exam Section 3: Item 23 of 50 National Board of Medical Examiners Time Remaining.
an Comprehensive Clinical Science Self-Assessment 4 hr 59 min 10 sec

23. A 27-year-old woman is brought to the emergency department because of a 2-day history of fever, pain with urination, and progressive generalized weakness. She has systemic lupus erythematosus
complicated by nephritis. Current medications include prednisone and cyclophosphamide. Her temperature is 39.4 °C ( 1 03°F), pulse is 122/min, respirations are 24/min, and blood pressure is
88/62 mm Hg. Examination shows warm, flushed skin. There is moderate suprapubic and right flank tenderness. Her leukocyte count is 18,000/mm3 (80% segmented neutrophils, 5% bands,
1 % eosinophils, 1% basophils, 10% lymphocytes, and 3% monocytes). Urinalysis shows 100 WBC/hpf. A Gram stain shows numerous gram-negative rods. Intravenous administration of piperacillin and
tazobactam and 2 liters of 0.9% saline is started. Thirty minutes later, her pulse is 124/min, and blood pressure is 92/68 mm Hg. Which of the following is the most appropriate next step in management?

0 A) Administration of ciprofloxacin
0 B) Administration of gentamicin
0 C) Administration of hydrocortisone
0 D) Administration of metoprolol
0 E) Cardioversion

https://t.me/USMLENBME2CK ti
Previous Next Lab Values Calculator Review Help pause
Exam Section 3: Item 24 0f 50 National Board of Medical Examiners Time Remaining.
an Comprehensive Clinical Science Self-Assessment 4 hr 59 min 8 sec

24. A72-year-old man comes to the physician because of a 3-monlh history of progressive shortness of breath, nonproductive cough, and fatigue. During this period, he has had early satiety resulting in a 2.3-
kg (5-lb) weight loss. He has no known sick contacts. He has a 20-year history of chronic obstructive pulmonary disease and a 10-year history of hypertension. Current medications are an inhaled
bronchodilator and an oral thiazide diuretic. He has smoked one pack of cigarettes daily for 50 years. He is in mild respiratory distress. He is 165 cm (5 ft 5 in) tall and weighs 52 kg (115 lb); BMI is 19
kg/m. His temperature is 37°C (98.6F), pulse is 103/min, respirations are 20/min, and blood pressure is 130/90 mm Hg. Pulse oximetry on room air shows an oxygen saturation of 87%. Examination
shows no scleral icterus. There is jugular venous distention. Breath sounds are distant. There is a right ventricular heave and a loud S A grade 2/6 systolic murmur is heard best over the left sternal
border. The liver is tender, smooth, and pulsatile. There is no peripheral edema. His hematocrit is 50%, leukocyte count is 4500/mm3, and platelet count is 200,000/mm3. Which of the following is the most
appropriate next step in management?

0 A) Digoxin therapy
0 8) Furosemide therapy
0 C) Incentive spirometry
0 D) Continuous positive airway pressure
0 E) Home oxygen therapy

https://t.me/USMLENBME2CK
ts e « t
Previous Next Lab Values Calculator Review Help pause
Exam Section 3: Item 25 of 50 National Board of Medical Examiners@ Time Remaining:
i ark Comprehensive Clinical Science Self-Assessment 4hr58min58 sec

IIII II II III II III I III W' III III IIIII WII III III II INII
[y(I[ IMIINII I[III(III [II III[III[ [III[NII [III1[ (I[1(/1Ill [/I[]
III III III /III II II III III III IIIIIIIIIII III III IL III
II IA III]! I! ,INIA!IWI.I!UNI,I!! IIIIIIMI
iilllllllllllllllllllllllllllllllllillllllllllllllllllllllllllllllllillllllllllllllllllllllllll
25. Four days after admission to the hospital for treatment of a myocardial infarction, a 72-year-old woman has the sudden onset of shortness of breath and confusion. She has atrial fibrillation, type 2
diabetes mellitus, essential hypertension, and hypercholesterolemia. Her medications are diltiazem, quinapril, metformin, and simvastatin. She has smoked one pack of cigarettes daily for 50 years and
has drunk one glass of wine daily for 52 years. She is anxious and oriented to person but not to place or time. Her temperature is 35°C (95°F), pulse is 116/min and irregularly irregular, respirations are
30/min, and blood pressure is 92/58 mm Hg. Pulse oximetry on 100% oxygen by face mask shows an oxygen saturation 0f 91%. The skin is cool to the touch. Examination shows jugular venous
distention. Decreased breath sounds are heard in the lung bases bilaterally. Scattered expiratory wheezes and crackles are heard bilaterally. A rub is heard in the lower lobe of the left lung. There is
2+ pretibial and pedal edema bilaterally. Laboratory studies show:
Hemoglobin 11.6 g/dL
Leukocyte count 11,000/mm3
Segmented neutrophils 65%
Bands 3%
Lymphocytes 30%
Monocytes 2%
Serum
Na 136 mEq/L
K 3. 7 mEq/L
Cl- 105 mEq/L
HCO.3 14 mEq/L
Glucose 86 mg/dL
Creatine kinase 68 U/L
Lactic acid 3.6 mEq/L (N=0.5-2)

Arterial blood gas analysis shows:


pH 7.29
Pco, 30 mm Hg
Po, 64 mm Hg

An ECG is shown. A chest x-ray shows cardiomegaly and pulmonary infiltrates, and pleural effusions bilaterally. Which of the following is the most appropriate next step in diagnosis?

0 A) Cardiac catheterization

https://t.me/USMLENBME2CK ts e « t
Previous Next Lab Values Calculator Review Help pause
Exam Section 3: Item 25 of 50 National Board of Medical Examiners Time Remaining.
an .
Comprehensive Clinical Science Self-Assessment
..
4 hr 58 min 55 sec

25. Four days after admission to the hospital for treatment of a myocardial infarction, a 72-year-old woman has the sudden onset of shortness of breath and confusion. She has atrial fibrillation, type 2
diabetes mellitus, essential hypertension, and hypercholesterolemia. Her medications are diltiazem, quinapril, metformin, and simvastatin. She has smoked one pack of cigarettes daily for 50 years and
has drunk one glass of wine daily for 52 years. She is anxious and oriented to person but not to place or time. Her temperature is 35°C (95°F), pulse is 116/min and irregularly irregular, respirations are
30/min, and blood pressure is 92/58 mm Hg. Pulse oximetry on 100% oxygen by face mask shows an oxygen saturation 0f 91%. The skin is cool to the touch. Examination shows jugular venous
distention. Decreased breath sounds are heard in the lung bases bilaterally. Scattered expiratory wheezes and crackles are heard bilaterally. A rub is heard in the lower lobe of the left lung. There is
2+ pretibial and pedal edema bilaterally. Laboratory studies show:
Hemoglobin 11.6 g/dL
Leukocyte count 11,000/mm3
Segmented neutrophils 65%
Bands 3%
Lymphocytes 30%
Monocytes 2%
Serum
Na 136 mEq/L
K 3.7 mEq/L
Cl- 105 mEq/L
HCO.3 14 mEq/L
Glucose 86 mg/dL
Creatine kinase 68 U/L
Lactic acid 3.6 mEq/L (N=0.5-2)

Arterial blood gas analysis shows:


pH 7.29
Pco, 30 mm Hg
Po, 64 mm Hg

An ECG is shown. A chest x-ray shows cardiomegaly and pulmonary infiltrates, and pleural effusions bilaterally. Which of the following is the most appropriate next step in diagnosis?

0 A) Cardiac catheterization
0 8) CT angiography
0 C) Echocardiography
0 D) Measurement of serum brain natriuretic peptide concentration
0 E) Radionuclide myocardial scan

https://t.me/USMLENBME2CK ti
Previous Next Lab Values Calculator Review Help pause
Exam Section 3: Item 26 of 50 National Board of Medical Examiners Time Remaining.
an Comprehensive Clinical Science Self-Assessment 4 hr 58 min 52 sec

26. A 2-month-old girl is brought to the clinic by her mother for a well-child examination. The mother says her daughter is exclusively breast-fed; it takes 1 hour to complete a feeding, and her daughter sweats
during feedings. The patient is at the 10th percentile for weight; she was at the 50th percentile at her last examination 6 weeks ago. Other growth parameters have remained stable at the 50th percentile.
Vital signs are within normal limits. The lungs are clear to auscultation. On cardiac examination, a grade 3/6 systolic murmur is heard best at the lower left sternal border. The liver edge is palpated 5 cm
below the right costal margin. Which of the following is the most likely cause of this patient's failure to thrive?

0 A) Atrial septal defect


0 B) Congenital hypothyroidism
0 C) Fabry disease
0 D) Glycogen storage disease, type II (Pompe disease)
0 E) Inadequate breast milk supply
0 F) Inadequate breast-feeding technique
0 G) Ventricular septal defect

https://t.me/USMLENBME2CK ti
Previous Next Lab Values Calculator Review Help pause
Exam Section 3: Item 27 of 50 National Board of Medical Examiners Time Remaining.
an Comprehensive Clinical Science Self-Assessment 4 hr 58 min 50 sec

27. A67-year-old man comes to the emergency department because of a 12-hour history of increasingly severe generalized, cramping pain that began in his left groin and spread to his abdomen. The patient
rates the pain as a 10 on a 10-poinl scale. He previously has noted a hernia in the left groin area. He says, "Normally I can push ii back in, but this lime I couldn't. I was going to wail and make an
appointment with my primary care doctor, but I couldn't tolerate the pain." He has hypertension treated with verapamil and hydrochlorothiazide. The patient appears uncomfortable. Temperature is 37.4°C
(99.3°F), pulse is 96/min, respirations are 20/min, and blood pressure is 140/95 mm Hg. Cardiac examination discloses an S,gallop. Bowel sounds are increased, and the abdomen is soft with moderate,
diffuse tenderness. A tender mass is present in the left groin extending into the scrotal sac, and the skin overlying the groin is erythematous. Results of laboratory studies are shown:
Blood Arterial blood gas analysis
Hematocrit 45% Po 75 mm Hg
Hemoglobin 15 g/dL Pco, 34 mm Hg
WBC 18,000/mm3 pH 7.35
Neutrophils, segmented 70%
Neutrophils, bands 20%
lymphocytes 8%
Monocytes 2%

The physician contacts the on-call surgeon to see the patient. The surgeon instructs the physician to attempt reduction of the hernia and to call back if unsuccessful. Which of the following is the most
appropriate next step in management?

0 A) Administer hydromorphone and attempt manual reduction of the hernia


0 B) Contact the hospital administrator and report an Emergency Medical Treatment and Active labor Act violation
0 C) Inform the surgeon that the patient needs to be evaluated
0 D) Obtain CT scan of the abdomen and call the surgeon back if there are signs of bowel obstruction

https://t.me/USMLENBME2CK
ts e « t
Previous Next Lab Values Calculator Review Help pause
Exam Section 3: Item 28 of 50 National Board of Medical Examiners Time Remaining.
an Comprehensive Clinical Science Self-Assessment 4 hr 58 min 49 sec

28. A 55-year-old man comes to the office because of a 2-month history of moderate right shoulder pain. The patient says the pain occurs whenever he raises his right arm straight up in front of him or up from
his side. He also reports decreased strength in his right shoulder. He says he injured his right shoulder 35 years ago during a college football game when he was hit by another player and landed shoulder-
first on the ground. After the injury, he had pain in the shoulder that resolved in 3 days without treatment. He has not had any subsequent injuries to the shoulder. Medical history otherwise is unremarkable,
and he takes no routine medications. Vital signs are normal. Physical examination discloses inability to resist downward pressure on the right arm when it is held straight out in front with the thumb pointing
downward. Which of the following imaging studies of the right shoulder is most likely to confirm the diagnosis?

0 A) AP and lateral x-rays


0 B) Arthrography
OC) MRI
0 D) Ultrasonography
0 E) No imaging studies are necessary to confirm the diagnosis

https://t.me/USMLENBME2CK ti e
Previous Next Lab Values Calculator Review Help pause
Exam Section 3: Item 29 of 50 National Board of Medical Examiners Time Remaining.
an Comprehensive Clinical Science Self-Assessment 4 hr 58 min 47 sec

29. A 25-year-old woman is brought to the emergency department 45 minutes after the onset of dizziness. Her symptom began after she consumed 1800 ml of water during a contest in which participants
tried to drink the most water in 15 minutes. She has no history of serious illness and takes no medications. On arrival, she is alert and fully oriented. Her pulse is 88/min and regular, respirations are
16/min, and blood pressure is 110/80 mm Hg. Examination, including neurologic examination, shows no abnormalities. Laboratory studies show:
Serum
Na' 128 mEq/L
K" 3.8 mEq/L
Urea nitrogen 5 mg/dl
Glucose 80 mg/dl
Creatinine 0.7 mg/dl
Urine specific gravity 1.005

Administration of which of the following is the most appropriate next step in management?

0 A) ADH (vasopressin)
0 B) Furosemide
0 C) 0.9% Saline
0 D) 3% Saline
0 E) No pharmacotherapy is indicated

https://t.me/USMLENBME2CK ti e
Previous Next Lab Values Calculator Review Help pause
Exam Section 3: Item 30 of 50 National Board of Medical Examiners@ Time Remaining:
i ark Comprehensive Clinical Science Self-Assessment 4hr58min45 sec

30. A previously healthy 57-year-old man is brought to the emergency department immediately after being involved in a head-on motor vehicle
collision. He was the driver and was wearing a seat belt with a shoulder strap. On arrival, he is alert and oriented to person, place, and time. He
says that he does not think he had toss of consciousness but feels sore all over his body. His temperature is 36.7C (98F), pulse is 100/min,
respirations are 14/min, and blood pressure is 140/70 mm Hg. Pulse oximetry on 2 L/min of oxygen via nasal cannula shows an oxygen
saturation of 99%. Physical examination and an ECG show no abnormalities. A chest x-ray is shown. Which of the following is the most likely
diagnosis?

0 A) Aortic disruption
0 B) Aspiration pneumonia
0 C) Perforation of the esophagus
0 D) Perforation of an intra-abdominal viscus
0 E) Pericardia! effusion
0 F ) Pneumothorax

https://t.me/USMLENBME2CK ts e « t
Previous Next Lab Values Calculator Review Help pause
Exam Section 3: Item 31 of 50 National Board of Medical Examiners Time Remaining.
an Comprehensive Clinical Science Self-Assessment 4 hr 58 min 43 sec

31. A 77-year-old man comes to the physician because of a 2-week history of increasing back pain. Eight years ago, he was diagnosed with prostate cancer. Gonadotropin inhibitor therapy was begun, and he
was asymptomatic until 2 months ago; metastatic lesions at L3 and TB were confirmed at that lime. Ibuprofen therapy sufficiently controlled the pain for 1 month, after which oxycodone (5 mg every 4
hours) and laxatives were added to the medication regimen. At his last visit 1 month ago, he reported his pain as between O and 2 on a 10-point scale. He now describes his pain as 4 to 5 on a 10-point
scale. Which of the following is the most appropriate next step in management?

0 A) Massage therapy and reevaluation in 1 week


0 B) Add oral acetaminophen therapy to the regimen
0 C) Begin intravenous morphine therapy
0 D) Begin oral propoxyphene therapy
0 E) Increase the oxycodone dosage by 25% to 50%
0 F) Switch from codeine to meperidine therapy

https://t.me/USMLENBME2CK ti
Previous Next Lab Values Calculator Review Help pause
Exam Section 3: ltem 32 of 50 National Board of Medical Examiners Time Remaining.
an Comprehensive Clinical Science Self-Assessment 4 hr 58 min 41 sec

32. An 18-year-old woman comes lo the physician for a follow-up examination. She has a5-year history of asthma treated with inhaled albuterol as needed and inhaled beclomethasone daily. She has never
required treatment in the hospital for an acute episode. She says that her asthma is worse after she spends prolonged periods of time indoors and after she visits homes of friends who own cats. Her
house has hardwood floors, which she and her family clean every week. She sleeps with her windows closed and changes her bedding regularly. She does not smoke or drink alcohol. Examination shows
no abnormalities. The lungs are clear to auscultation. Skin testing shows allergies to dust mites and cat dander. The physician advises the patient to avoid direct contact with cats. Which of the following
environmental changes is likely to be most helpful in preventing exacerbations of this patient's asthma?

0 A) Humidifier use
0 B) Placement of carpet in her bedroom
0 C) Sleeping with the windows open
0 D) Use of allergen-impermeable mattress and pillow covers
0 E) Vacuuming the floors of her house daily

https://t.me/USMLENBME2CK ti
Previous Next Lab Values Calculator Review Help pause
Exam Section 3: ltem 33 of 50 National Board of Medical Examiners Time Remaining.
an Comprehensive Clinical Science Self-Assessment 4 hr 58 min 39 sec

33. A44-year-old man comes to the office because of a 6-month history of intermittent moderate headache. He has had one to two headaches weekly since residents of the neighboring apartment got a dog
that barks a lot. He says he has had recurrent thoughts about killing the dog during the past 6 months, but he has no intent or plan to harm the animal. He worries that his headaches are punishment for
these thoughts about killing; he says he realizes this does not make sense but remains concerned. He vacuums 2 hours daily because he is worried that dog hair is coming into his apartment. He has no
history of serious illness. His only medication is acetaminophen, which resolves his headaches. He drinks two to three 12-oz beers on weekends. His pulse is 72/min, respirations are 16/min, and blood
pressure is 150/105 mm Hg. Physical examination shows no other abnormalities. Mental status examination shows an anxious affect. Which of the following is the most likely diagnosis?

0 A) Delusional disorder
0 B) Generalized anxiety disorder
0 C) Obsessive-compulsive disorder
0 D) Paranoid personality disorder
0 E) Somatic symptom disorder

https://t.me/USMLENBME2CK
ts e « t
Previous Next Lab Values Calculator Review Help pause
Exam Section 3: Item 34 of 50 National Board of Medical Examiners Time Remaining.
an Comprehensive Clinical Science Self-Assessment 4 hr 58 min 37 sec

34. A 27-year-old primigravid woman at 32 weeks' gestation comes to the emergency department because of a 2-day history of progressive drooping of the right side of her face, an inability to close her right
eye, and right-sided facial pain. She has not had loss of hearing or balance. Her pregnancy had been uncomplicated. She has no personal or family history of serious illness. Her temperature is 37C
(98.6F), pulse is 88/min, respirations are 16/min, and blood pressure is 140/82 mm Hg. Examination of the face shows paralysis on the right side and an inability to close the right eye. The patient moves
her head stiffly; there is no nuchal rigidity. Heart sounds are normal. A grade 2/6 holosystolic murmur is heard best at the apex. The abdomen is soft. The fundal height is 32 cm. There is mild edema of the
lower extremities. Muscle strength is normal. Deep tendon reflexes are 2+. Which of the following is the most appropriate next step in management?

0 A) Carotid ultrasonography
0 B) CT scan of the head
0 C) Corticosteroid therapy
0 0) Angiography of the brain
0 E) Nerve conduction studies

https://t.me/USMLENBME2CK ti e
Previous Next Lab Values Calculator Review Help pause
Exam Section 3: Item 35 0f 50 National Board of Medical Examiners@ Time Remaining:
i ark Comprehensive Clinical Science Self-Assessment 4hr58min35 sec

35. A 73-year-old woman is brought to the clinic by her husband because of a 6-month history of progressive memory difficulty. During this time, she has misplaced objects more frequently than usual and
has had trouble following recipes. Six days ago, she became lost while walking their dog in their neighborhood. Today, she forgot that their daughter had called the previous day. The patient has type 2
diabetes mellitus and hypertension. Her medications are metformin and lisinopril. She does not smoke cigarettes or drink alcohol. Vital signs are within normal limits. Physical examination shows no
abnormalities. On mental status examination, she is oriented to person but not to place or time. She recalls zero of three objects after 5 minutes. She makes four errors when performing serial sevens.
She can name the US President and Vice President. She can name eight animals in 1 minute. The remainder of the examination shows no abnormalities. Results of a complete blood count, her
erythrocyte sedimentation rate, and results of liver and thyroid function tests are within the reference ranges. VDRL testing for syphilis is negative. MRls of the brain are shown. Which of the following is
the most appropriate next step in management?

0 A) Deep brain stimulation


0 B) Placement of a ventriculoperitoneal shunt
0 C) Transcranial magnetic stimulation
0 D) Treatment with an acetylcholine precursor

https://t.me/USMLENBME2CK ts e « t
Previous Next Lab Values Calculator Review Help pause
Exam Section 3: Item 35 of 50 National Board of Medical Examiners" Time Remaining:
an Comprehensive Clinical Science Self-Assessment 4 hr 58 min 32 sec

35. A 73-year-old woman is brought to the clinic by her husband because of a 6-month history of progressive memory difficulty. During this time, she has misplaced objects more frequently than usual and
has had trouble following recipes. Six days ago, she became lost while walking their dog in their neighborhood. Today, she forgot that their daughter had called the previous day. The patient has type 2
diabetes mellitus and hypertension. Her medications are metformin and lisinopril. She does not smoke cigarettes or drink alcohol. Vital signs are within normal limits. Physical examination shows no
abnormalities. On mental status examination, she is oriented to person but not to place or time. She recalls zero of three objects after 5 minutes. She makes four errors when performing serial sevens.
She can name the US President and Vice President. She can name eight animals in 1 minute. The remainder of the examination shows no abnormalities. Results of a complete blood count, her
erythrocyte sedimentation rate, and results of liver and thyroid function tests are within the reference ranges. VDRL testing for syphilis is negative. MRls of the brain are shown. Which of the following is
the most appropriate next step in management?

0 A) Deep brain stimulation


0 B) Placement of a ventriculoperitoneal shunt
0 C) Transcranial magnetic stimulation
0 D) Treatment with an acetylcholine precursor
0 E) Treatment with a cholinesterase inhibitor
0 F) Treatment with a postsynaptic choline receptor agonist

https://t.me/USMLENBME2CK ts e « t
Previous Next Lab Values Calculator Review Help pause
Exam Section 3: Item 36 of 50 National Board of Medical Examiners Time Remaining.
an Comprehensive Clinical Science Self-Assessment 4 hr 58 min 28 sec

36. A previously healthy 77-year-old man comes to the emergency department because of a 6-hour history of increasingly severe, nonradiating lower abdominal pain. He also has had urinary urgency during
this time but has not been able to void for 8 hours. He has not had fever, chills, diarrhea, nausea, or vomiting. Vital signs are within normal limits. On examination, there is fullness in the midline from the
umbilicus to the pubic symphysis; the area is tender to palpation, but there are no peritoneal signs. Rectal examination shows an enlarged prostate; sphincter tone is normal, and no masses are palpable.
Which of the following is the most likely diagnosis?

0 A) Bacterial prostalilis
0 B) Benign prostatic hyperplasia
0 C) Cecal volvulus
0 D) Prostatic adenocarcinoma
0 E) Sigmoid volvulus
0 F) Urinary tract infection

https://t.me/USMLENBME2CK ti
Previous Next Lab Values Calculator Review Help pause
Exam Section 3: Item 37 of 50 National Board of Medical Examiners8 Time Remaining:
i ark Comprehensive Clinical Science Self-Assessment 4hr58min27 sec

37. A 6-year-old boy is brought to the emergency department by his parents because of a 2-hour history of severe right leg pain and refusal to walk. They say he has
intermittently reported pain in his leg during the past 2 weeks. He has no history of trauma or serious illness and receives no medications. Growth and development are
appropriate for age. His temperature is 38.1C (100.5F), pulse is 100/min, and blood pressure is 100/80 mm Hg. There is warmth and tenderness to palpation of the
proximal aspect of the right tibia with an associated firm mass. Distal pulses are intact. Range of motion of the right knee is limited by pain. Muscle strength in the right lower
extremity is 4/5 because of pain and 5/5 elsewhere. Sensation is intact. Laboratory studies show a hemoglobin concentration of 10 g/dl, leukocyte count of 20,000/mm?
and erythrocyte sedimentation rate of 10 mm/h. X-rays show a 6-cm, poorly defined lytic lesion in the proximal tibial metadiaphysis with adjacent periosteal reaction. The
results of a biopsy are shown. Which of the following is the most likely diagnosis?

0 A) Acute lymphoblastic leukemia


0 B) Eosinophilic granuloma
0 C) Ewing sarcoma
0 D) Osteoid osteoma
0 E ) Osteomyelitis

https://t.me/USMLENBME2CK ts e « t
Previous Next Lab Values Calculator Review Help pause
Exam Section 3: Item 38 of 50 National Board of Medical Examiners Time Remaining.
an Comprehensive Clinical Science Self-Assessment 4 hr 58 min 24 sec

38. A 39-year-old man comes to the office because of a 1-month history of intermittent regurgitation and constipation. He has regurgitated undigested food four times weekly. He previously had daily bowel
movements; during the past month, he has had a bowel movement every 3 to 4 days. He also has a 1-year history of progressive difficulty swallowing. He says initially he had difficulty swallowing solids.
He now also has difficulty swallowing liquids. When he eats, he occasionally has moderate pain in the lower sternal area that radiates upward. He emigrated from Brazil 5 years ago. Vital signs are within
normal limits. Examination shows no abnormalities. A chest x-ray shows a dilated esophagus with an air-fluid level. Which of the following is the most likely diagnosis?

0 A) Barrett esophagus
0 B) Nutcracker esophagus
0 C) Schatzki ring
0 D) South American trypanosomiasis
0 E) Systemic sclerosis (scleroderma)

https://t.me/USMLENBME2CK ti
Previous Next Lab Values Calculator Review Help pause
Exam Section 3: Item 39 of 50 National Board of Medical Examiners Time Remaining.
an Comprehensive Clinical Science Self-Assessment 4 hr 58 min 21 sec

39. A hospitalized 27-year-old woman with sickle cell crisis has shortness of breath and nonproductive cough 4 hours after transfusion of 1 unit of packed red blood cells. Her only medication is morphine. Her
temperature is 38°C (100.4°F), pulse is 126/min, respirations are 28/min, and blood pressure is 98/58 mm Hg. Pulse oximetry on room air shows an oxygen saturation of 88%. Examination shows central
cyanosis. Diffuse crackles are heard bilaterally. Heart sounds are normal. Arterial blood gas analysis on room air shows:
pH 7.47
Pco, 38 mm Hg
Po, 55mm Hg

A chest x-ray shows bilateral pulmonary infiltrates. Which of the following is the most likely cause of these findings?

OA ABO incompatibility
0 B) Anaphylactic transfusion reaction
0 C) Fluid overload
0 D) Hemolytic transfusion reaction
0 E) Transfusion-related acute lung injury

https://t.me/USMLENBME2CK ti
Previous Next Lab Values Calculator Review Help pause
Exam Section 3: Item 40 of 50 National Board of Medical Examiners Time Remaining.
an Comprehensive Clinical Science Self-Assessment 4 hr 58 min 19 sec

40. A22-year-old man comes to the physician because of a severe nosebleed and a generalized rash for the past 24 hours. He has a 1-month history of fatigue and generalized weakness. His temperature is
36. 7"C (98"F), pulse is 100/min, respirations are 18/min, and blood pressure is 142/72 mm Hg. Examination of the skin shows diffuse petechiae over the entire body. There is diffuse lymphadenopathy and
mild hepatosplenomegaly. The nares are packed with gauze, and there is no active bleeding. laboratory studies show:
Hematocrit 28%
leukocyte count 36,000/mm3
Segmented neutrophils 10%
lymphocytes 25%
lymphoblasts and other immature forms 65%
Platelet count 15,000/mm3
Serum
Urea nitrogen 20 mg/dl
Creatinine 1.2 mg/dL
Uric acid 15.2 mg/dL

A biopsy specimen of bone marrow shows acute lymphoblastic leukemia. In addition to vigorous hydration and alkalinization of the urine, which of the following is most appropriate to administer to prevent
acute renal failure during chemotherapy in this patient?

0 A) Acetylcysteine
0 B) Allopurinol
0 C) lisinopril
0 D) low-dose dopamine
0 E) Mesna

https://t.me/USMLENBME2CK ti
Previous Next Lab Values Calculator Review Help pause
Exam Section 3: Item 41 of 50 National Board of Medical Examiners Time Remaining.
an Comprehensive Clinical Science Self-Assessment 4 hr 58 min 14 sec

41. A25-year-old man is brought to the emergency department by ambulance 25 minutes after his wife found him unconscious on the floor at home this morning. He was discharged from the military 3 months
ago after he sustained a traumatic brain injury and underwent right below-the-knee amputation. Since his return, his family says he has had mood swings, aggressive behavior, and anxiety, which have
prevented him from interacting with people. He has major depressive disorder and severe limb pain. His medications are amitriptyline, sertraline, oxycodone, and acetaminophen. Two days ago, he began
a short-term course of alprazolam for persistent anxiety. His family says he generally has been adherent to his medication and cognitive behavioral therapy regimens. He does not smoke cigarettes. He
drank four 12-oz beers weekly as a teenager; now he drinks eight 12-oz beers weekly. On arrival, he is somnolent and responds to painful stimuli only. Temperature is 36.7C (98.0°F), pulse is 110/min,
respirations are 10/min and shallow, and blood pressure is 85/60 mm Hg. Pulse oximetry on 4 Umin of oxygen by nasal cannula shows an oxygen saturation of 92%. Examination shows warm, dry, flushed
skin. The pupils are mydriatic with nystagmus. Air movement is decreased in the lungs. Cardiac examination shows no abnormalities. The abdomen is soft. Bowel sounds are decreased. Patellar deep
tendon reflexes are 3+ bilaterally. An overdose of which of the following is the most likely explanation for these findings?

0 A) Acetaminophen
Q B) Alcohol
0 C) Alprazolam
0 D) Amitriptyline
0 E) Oxycodone

https://t.me/USMLENBME2CK ti
Previous Next Lab Values Calculator Review Help pause
Exam Section 3: Item 42 of 50 National Board of Medical Examiners Time Remaining:
an Comprehensive Clinical Science Self-Assessment 4 hr 58 min 11 sec

42. A 29-year-old woman comes to the physician for a routine health maintenance examination. She has no history of serious illness and takes no medications. Menses occur at regular 28-day intervals, last
5 days, and are accompanied by mild cramps. Her last menstrual period was 3 weeks ago. She is sexually active and monogamous with one male partner. She has a copper IUD in place. Her blood
pressure is 118/72 mm Hg. Physical examination shows no abnormalities. Pelvic examination shows an IUD string at the cervical os and a mildly tender right adnexal mass. Transvaginal ultrasonography
shows a normal uterus and left adnexa and a 5-cm hemorrhagic cyst in the right adnexa. Which of the following is the most appropriate next step in management?

0 A) Ceftriaxone and doxycycline therapy


0 B) Oral contraceptive therapy
0 C) Ovarian cystectomy
0 D) Removal of the IUD
0 E) Repeat transvaginal ultrasonography in 6 weeks

https://t.me/USMLENBME2CK ti
Previous Next Lab Values Calculator Review Help pause
Exam Section 3: ltem 43 of 50 National Board of Medical Examiners Time Remaining.
an Comprehensive Clinical Science Self-Assessment 4 hr 58 min 8 sec

43. A 20-year-old primigravid woman at 28 weeks' gestation is brought to the emergency department after she was involved in a high-speed motor vehicle collision. She was the restrained front seat
passenger. She has no history of serious illness. Her pregnancy had been uncomplicated. She has moderate neck and back pain and abdominal tenderness. She appears confused. Her temperature is
37°C (98.6°F), pulse is 120/min, respirations are 24/min, and blood pressure is 100/58 mm Hg. The fetal heart rate is 140--150/min. Examination shows multiple facial contusions. laboratory studies show
a hematocrit of 32%, leukocyte count of 9000/mm3, and platelet count of 210,000/mm3. In addition to continuous fetal monitoring, which of the following is the most appropriate next step in management?

0 A) CT scan of the head and neck


( B) Administration of Rh, (D) immune globulin
0 C) Terbutaline tocolysis
0 D) Cesarean delivery
0 E) Exploratory laparotomy

https://t.me/USMLENBME2CK ti
Previous Next Lab Values Calculator Review Help pause
Exam Section 3: Item 44 of 50 National Board of Medical Examiners Time Remaining.
an Comprehensive Clinical Science Self-Assessment 4 hr 58 min 6 sec

44. A 15-year-old boy is brought to the physician because of fatigue since starting his freshman year of high school 3 months ago. He often falls asleep during class. He urinates four to five times nightly and
often has difficulty falling asleep again. He has no history of serious illness and takes no medications. He is at the 20th percentile for height and above the 95th percentile for weight and BMI. Vital signs
are within normal limits. Examination shows a velvety, hyperpigmented macular rash over the neck and axillae. The remainder of the examination shows no abnormalities. Laboratory studies show:
Hematocrit 39%
Leukocyte count 5900/mm3
Platelet count 401,000/mm?
Serum
Na 133 mEq/L
K+ 4.3 mEq/L
Cl- 98 mEq/l
HCO.3 22 mEq/l
Urea nitrogen 14 mg/dL
Glucose 148 mg/dL
Creatinine 0.6 mg/dL
Urine
pH 5
Specific gravity 1.035
Glucose 2+
Ketones negative

This patient is at increased risk for which of the following complications of his current condition during the next 15 years?

0 A) Adenocarcinoma of the bowel


0 B) Adrenal insufficiency
0 C) Coronary artery disease
0 D) Diabetes insipidus
0 E) Ruptured cervical disc

https://t.me/USMLENBME2CK ti
Previous Next Lab Values Calculator Review Help pause
Exam Section 3: Item 45 of 50 National Board of Medical Examiners Time Remaining.
an Comprehensive Clinical Science Self-Assessment 4 hr 58 min 4 sec

45. A previously healthy 3-year-old boy who is scheduled for an elective operation begins lo cry and cling tightly lo his mother on arrival at the hospital. Which of the following is the most likely cause of this
behavior?

0 A) Fear of mutilation
0 B) Fear of pain
0 C) Fear of separation from parent
0 D) Panic disorder
0 E) Stranger anxiety

https://t.me/USMLENBME2CK ti
Previous Next Lab Values Calculator Review Help pause
Exam Section 3: Item 46 of 50 National Board of Medical Examiners Time Remaining.
an Comprehensive Clinical Science Self-Assessment 4 hr 58 min 1 sec

46. A 19-year-old woman is brought to the emergency department by her mother 30 minutes after ingesting an unknown liquid she found in her parents' utility room following an argument with her boyfriend.
Immediately after swallowing the liquid, she telephoned her mother at work, who arrived at home 20 minutes later and found her daughter drooling and reporting severe upper chest pain. On arrival at the
emergency department, the patient is in obvious distress and is drooling and unable to swallow. Temperature is 37.0°C (98.6°F), pulse is 120/min and regular, respirations are 20/min, and blood pressure is
148/90 mm Hg. Physical examination shows profuse clear drainage from the mouth and erythema and edema of the posterior oropharynx. Cardiopulmonary examination shows no abnormalities. Two
large-bore intravenous catheters are inserted for fluid administration and a dose of intravenous hydromorphone is administered. The patient begins to feel more comfortable. Results of laboratory studies
are within the reference ranges. Chest x-ray shows no abnormalities. Which of the following is the most appropriate management?

0 A) Administration of activated charcoal


0 B) Administration of oral bicarbonate
0 C) Esophagogastroduodenoscopy
0 D) Induction of vomiting
0 E) Nasogastric tube placement

https://t.me/USMLENBME2CK ti
Previous Next Lab Values Calculator Review Help pause
Exam Section 3: Item 47 of 50 National Board of Medical Examiners Time Remaining.
an Comprehensive Clinical Science Self-Assessment 4 hr 57 min 58 sec

4 7. A 23-year-old male college student comes to the clinic for a follow-up visit 14 days after undergoing emergent splenectomy following a motor vehicle collision. The patient has resumed his regular diet and
is ambulating normally. Medical history otherwise is unremarkable, and his only medication is acetaminophen as needed. The patient completed his Haemophilus influenzae type b vaccinations by age 18
months and his varicella, polio, and measles-mumps-rubella vaccinations by age 5 years; he received a human papillomavirus vaccination at age 11 years. Temperature is 37.0C (98.6°F), pulse is 82/min,
and blood pressure is 116/54 mm Hg. Abdominal examination shows a well-healing midline incision. Which of the following vaccinations is most appropriate to administer to this patient today?

0 A) Haemophilus influenzae type b


0 B) Human papillomavirus
0 C) Inactivated poliovirus
0 D) Measles, mumps, rubella
0 E) Varicella
0 F) No vaccine is indicated at this lime

https://t.me/USMLENBME2CK
ts e « t
Previous Next Lab Values Calculator Review Help pause
Exam Section 3: Item 48 of 50 National Board of Medical Examiners Time Remaining.
an Comprehensive Clinical Science Self-Assessment 4 hr 57 min 56 sec

48. A previously healthy 19-year-old college student comes to the emergency department because of a 3-day history of lower abdominal pain and vaginal spotting. Menses usually occur at regular 28-day
intervals, but she cannot recall the date of her last menstrual period. She takes no medications. She is sexually active and uses condoms inconsistently. She appears pale but is alert and answers
questions appropriately. Her pulse is 110/min, respirations are 22/min, and blood pressure is 80/50 mm Hg. Abdominal examination shows lower quadrant tenderness with marked guarding. Pelvic
examination shows a soft uterus and adnexal tenderness greater on the right than on the left. Her hemoglobin concentration is 6.2 g/dL, hematocrit is 18%, and serum 13-hCG concentration is
6645 mlU/mL. Urinalysis shows no abnormalities. Which of the following is the most appropriate next step in management?

0 A) Observation only
0 B) CT scan of the abdomen
0 C) Intravenous antibiotic therapy
0 D) Culdocentesis
0 E) Exploratory laparotomy

https://t.me/USMLENBME2CK
ts e « t
Previous Next Lab Values Calculator Review Help pause
Exam Section 3: Item 49 of 50 National Board of Medical Examiners Time Remaining.
an Comprehensive Clinical Science Self-Assessment 4 hr 57 min 54 sec

49. A17-year-old boy is brought to the office by his parents because of changes in his behavior during the past 6 months. They say he has become withdrawn and moody and that his grades have declined.
He has returned home from parties intoxicated three times during the past month. During a wedding reception 3 weeks ago, he was delirious, incoherent, and imbalanced. The patient says he drinks
alcohol but that his drinking is not a problem and he can stop drinking whenever he wants. Vital signs are within normal limits. Physical examination shows no abnormalities. On mental status examination,
he is calm and cooperative. He reports no insomnia, hypersomnia, or suicidal ideation. Which of the following is the most likely diagnosis?

0 A) Alcohol use disorder


0 B) Conduct disorder
0 C) Major depressive disorder
0 D) Normal adolescent behavior
0 E) Recreational alcohol use

https://t.me/USMLENBME2CK ti
Previous Next Lab Values Calculator Review Help pause
Exam Section 3: Item 50 of 50 National Board of Medical Examiners Time Remaining.
an Comprehensive Clinical Science Self-Assessment 4 hr 57 min 53 sec

50. A 4 7 -year-old woman comes to the physician because of a 9-month history of hot flashes, night sweats, and pain during sexual intercourse. Her last menstrual period was 1 year ago. Physical examination
shows no abnormalities. Pelvic examination shows vaginal atrophy and no cervical mucus. A Pap smear and mammography show no abnormalities. Which of the following is the most appropriate next step
in management?

0 A) Pregnancy test
0 B) Measurement of serum luteinizing hormone concentration
0 C) Hormone therapy
0 D) levothyroxine therapy
0 E) Vaginal corticosteroid therapy

https://t.me/USMLENBME2CK ti
Previous Next Lab Values Calculator Review Help pause
Exam Section 4: ltem 1 of 50 National Board of Medical Examiners Time Remaining.
an Comprehensive Clinical Science Self-Assessment 4 hr 59 min 53 sec

1. A 67-year-old woman comes to the physician because of a 2-month history of perirectal pain that worsens with bowel movements and a thick, yellow rectal discharge. She has hypertension and psoriasis.
Her medications are metoprolol and topical fluocinolone cream. There is no family history of serious illness. She does not smoke cigarettes, drink alcohol, or use illicit drugs. Her temperature is 37C
(98.6F), pulse is 80/min, respirations are 14/min, and blood pressure is 136/70 mm Hg. The abdomen is soft, nontender, and nondistended. Rectal examination shows brown stool. The skin of the posterior
perinea I wall is erythematous with a draining sinus tract. Test of the stool for occult blood is positive. Which of the following is the most likely diagnosis?

0 A) Clostridium difficile colitis


0 B) Colon cancer
0 C) Crohn disease
0 D) Diverticular abscess
0 E) Ulcerative colitis

https://t.me/USMLENBME2CK ti
Next Lab Values Calculator Review Help Pause
Exam Section 4: Item 2 of 50 National Board of Medical Examiners Time Remaining.
an Comprehensive Clinical Science Self-Assessment 4 hr 59 min 50 sec

2. Twelve hours after discharge from the hospital, a 2-day-old male newborn is brought to the emergency department after having a seizure. His parents report that he stiffened, and his eyes rolled back for 3
or 4 minutes; then he went limp. He was delivered vaginally at term to a 20-year-old woman, gravida 1, para 1, following an uncomplicated pregnancy; his weight on delivery was 3062 g (6 lb 12 oz). The
patient's mother used to drink two glasses of wine each evening with dinner but stopped drinking alcohol when she learned she was pregnant at 4 weeks' gestation. She has never smoked cigarettes or
used illicit drugs. On arrival, the infant is awake but sluggish. Examination shows hypertelorism and low-set ears. The lungs are clear to auscultation. There are no murmurs. A chest x-ray shows a narrow
mediastinum and a right-sided aortic arch. Which of the following is the most likely diagnosis?

0 A) Down syndrome
0 B) Severe combined immunodeficiency
0 C) Thymic-parathyroid dysplasia
0 D) Trisomy 13 syndrome
0 E) Wiskott-Aldrich syndrome

https://t.me/USMLENBME2CK ti
Previous Next Lab Values Calculator Review Help pause
Exam Section 4: ltem 3 of 50 National Board of Medical Examiners Time Remaining.
an Comprehensive Clinical Science Self-Assessment 4 hr 59 min 49 sec

3. A 2-year-old girl is brought to the physician for a well-child examination. She has had two urinary tract infections since birth. She has not had pain with urination or urinary frequency or urgency. Examination
shows no abnormalities. Renal ultrasonography shows no obstruction. Voiding cystourethrography shows grade II vesicoureteral reflux. Which of the following is the most appropriate initial step to prevent
future urinary tract infections in this patient?

0 A) Monitor temperature curves daily


0 B) Urine cultures weekly
0 C) Oral antibiotic prophylaxis
0 D) 2-Week course of intravenous antibiotic therapy
0 E) Vesicoureteral reimplantation

https://t.me/USMLENBME2CK ti
Previous Next Lab Values Calculator Review Help pause
Exam Section 4: Item 4 of 50 National Board of Medical Examiners Time Remaining.
an Comprehensive Clinical Science Self-Assessment 4 hr 59 min 47 sec

4. A4-year-old boy is brought to the physician by his mother because she is concerned that he is having difficulty making friends. The mother reports that he attends preschool but does not socialize with the
other children and spends most of the time playing by himself. At school, he is only interested in blocks, reports how tall he can build towers, and resists other activities. At home, he spends hours building
towers out of blocks in his room and becomes very upset if he is interrupted. He does not talk about any of his classmates; when his mother invites another child to the house, he has little interest in
socializing. His mother reports that he understands directions and communicates without difficulty but has never been very affectionate. He is generally well behaved but will have a temper tantrum if a daily
routine is altered or if furniture is rearranged. Pregnancy, labor, and delivery were uncomplicated, and he has reached all appropriate motor milestones for his age. His mother does not recall if language
emerged somewhat later in him than ii did for her daughter but notes that he makes his needs and wants known without any problem. He is neatly dressed. Physical examination shows no abnormalities.
On mental status examination, he makes no eye contact with the physician and perseverates about the towers that he built in the waiting room. Which of the following is the most likely diagnosis?

0 A) Attention-deficit/hyperactivity disorder
0 B) Expressive language disorder
0 C) Obsessive-compulsive disorder
0 D) Pervasive developmental disorder
0 E) Social anxiety disorder (social phobia)

https://t.me/USMLENBME2CK ti
Previous Next Lab Values Calculator Review Help pause
Exam Section 4: Item 5 of 50 National Board of Medical Examiners Time Remaining.
an Comprehensive Clinical Science Self-Assessment 4 hr 59 min 45 sec

5. Two days after undergoing urgent cesarean delivery because of a nonreassuring fetal heart tracing, a37-year-old woman, gravida 1, para 1, has the onset of dysphoria, insomnia, and easy distractibility.
Pregnancy was uncomplicated. She has no history of serious illness, and her only medication is a prenatal vitamin. Her paternal cousin had major depressive disorder, postpartum onset. The patient's vital
signs are within normal limits. Pulse oximetry on room air shows an oxygen saturation of 97%. Physical examination shows no abnormalities. On mental status examination, she has an irritable mood and
tired, concerned affect. Without treatment, which of the following is most likely to develop in this patient?

0 A) Alopecia
0 B) Dissociative amnesia with dissociative fugue
0 C) loss of maternal bond with newborn
0 D) Resolution of symptoms within 2 weeks
OE Suicidality

https://t.me/USMLENBME2CK ti
Previous Next Lab Values Calculator Review Help pause
Exam Section 4: Item 6 of 50 National Board of Medical Examiners Time Remaining.
an Comprehensive Clinical Science Self-Assessment 4 hr 59 min 44 sec

6. A 32-year-old woman with asthma comes to the physician because of a 3-month history of progressive wheezing and a cough that is occasionally productive of plugs of brownish sputum. She received the
diagnosis of asthma 15 years ago. She has been taking an inhaled long-acting 13-adrenergic agonist and inhaled glucocorticoid daily for 8 years and had good control of her symptoms until 3 months ago.
She has no pets. She has no recent history of travel abroad or new occupational exposure. She appears well and is not in respiratory distress. Vital signs are within normal limits. Pulse oximetry on room air
shows an oxygen saturation of 96%. Examination shows no cyanosis. Scattered inspiratory and expiratory wheezes are heard. There is no clubbing or peripheral edema. Her leukocyte count is 20,000/mm3
(45% segmented neutrophils, 40% eosinophils, 12% lymphocytes, and 3% monocytes). Her serum lgE concentration is 2800 IU/ml. A chest x-ray shows alveolar infiltrates in the upper lungs bilaterally.
Which of the following is the most likely diagnosis?

0 A) Allergic bronchopulmonary aspergillosis


0 B) Eosinophilic leukemia
0 C) Hypereosinophilic syndrome
0 D) Polyarteritis nodosa
0 E) Strongy/oides stercora/is infection

https://t.me/USMLENBME2CK ti
Previous Next Lab Values Calculator Review Help pause
Exam Section 4: Item 7 of 50 National Board of Medical Examiners Time Remaining.
an Comprehensive Clinical Science Self-Assessment 4 hr 59 min 42 sec

7. A 24-year-old woman comes to the office for an annual employment physical examination. She feels well. Medical history is unremarkable and she takes no medications. Vital signs are within normal limits.
Physical examination discloses no abnormalities. As the physician is preparing to leave the examination room the patient asks, "Do you think having a romantic relationship with my gynecologist is all right?"
Which of the following is the most appropriate response?

0 A) "I am under no obligation to report any romantic relationship to a disciplinary board."


0 B) "If you switch gynecologists ii would be acceptable for your former gynecologist to begin a romantic relationship with you immediately."
0 C) "A mutually consensual romantic relationship between patient and physician can sometimes be Justified."
0 D) "A romantic relationship in this setting is always unethical."
0 E) ''Would you mind telling me who your gynecologist is?"

https://t.me/USMLENBME2CK
ts e « t
Previous Next Lab Values Calculator Review Help pause
Exam Section 4: Item 8 of 50 National Board of Medical Examiners Time Remaining.
an Comprehensive Clinical Science Self-Assessment 4 hr 59 min 41 sec

8. A healthy 52-year-old man comes lo the physician for a routine total colonoscopy. He has no history of blood in his stool or rectal bleeding. There is no family history of colorectal polyps or cancer. He takes
no medications and has never smoked cigarettes. Physical examination shows no abnormalities. Colonoscopy shows a 3-mm pedunculated polyp in the cecum. No other abnormalities are noted. The polyp
is completely excised. Histopathologic evaluation shows dysplastic-appearing columnar epithelial cells confined by the basement membrane, lining a fibrovascular stalk. Which of the following is the most
appropriate recommendation for future screening for colon cancer in this patient?

0 A) Colonoscopy in 1 year
0 B) Colonoscopy in 3 years
0 C) Colonoscopy in 10 years
0 D) Flexible sigmoidoscopy in 1 year
0 E) No further screening is indicated unless annual test of the stool for occult blood is positive

https://t.me/USMLENBME2CK ti e
Previous Next Lab Values Calculator Review Help pause
Exam Section 4: Item 9 of 50 National Board of Medical Examiners Time Remaining.
an Comprehensive Clinical Science Self-Assessment 4 hr 59 min 39 sec

9. A healthy 18-month-old boy is brought for a well-child examination. He was born at 37 weeks' gestation, needed no resuscitation, and had no neonatal problems. He was fed formula for the first 2 months of
life, and then his mother substituted cow's milk because of the expense of the formula. He has received no supplements or medications. Examination shows no abnormalities. This child is at greatest risk for
which of the following?

0 A) Iron deficiency anemia


0 B) Pellagra
0 C) Protein-calorie malnutrition
0 D) Vitamin 8 12 (cobalamin) deficiency
0 E) Vitamin D deficiency

https://t.me/USMLENBME2CK ti
Previous Next Lab Values Calculator Review Help pause
Exam Section 4: ltem 10 of 50 National Board of Medical Examiners Time Remaining.
an Comprehensive Clinical Science Self-Assessment 4 hr 59 min 35 sec

10. A5-year-old girl is brought to the physician as a new patient for an examination prior lo starting kindergarten. She was noted to have high blood pressure by a previous physician 3 years ago. Her blood
pressure 2 years ago was within normal limits. She has not been seen by a physician since that lime because her mother did not have adequate health insurance. The patient is at the 50th percentile for
height and 15th percentile for weight. Her temperature is 36.7C (98F), pulse is 80/min, respirations are 25/min, and blood pressure is 140/90 mm Hg. A bruit is heard over the right aspect of the
midabdomen. No other abnormalities are noted. Renal arteriography shows stenosis of the right renal artery. Which of the following is the most appropriate next step in management?

0 A) Weight loss program


0 B) Warfarin therapy
0 C) Angioplasty
0 D) Renal transplant
0 E) No treatment is indicated

https://t.me/USMLENBME2CK ti e
Previous Next Lab Values Calculator Review Help pause
Exam Section 4: Item 11 of 50 National Board of Medical Examiners Time Remaining.
an Comprehensive Clinical Science Self-Assessment 4 hr 59 min 33 sec

11. A 42-year-old woman, gravida 1, para 1, comes lo the office for a routine examination. She is concerned about her risk for coronary artery disease. She has irritable bowel syndrome and migraines.
Medications are sumatriptan and ibuprofen as needed. Her father has hypertension and hyperlipidemia. Her paternal uncle was diagnosed with coronary artery disease at the age of 50 years. The patient
smoked one pack of cigarettes daily for 10 years but quit 7 years ago. She drinks two glasses of wine weekly. She does not use illicit drugs. She runs 3 miles one to three limes weekly. She is 168 cm
(5 fl 6 in) tall and weighs 77 kg (170 lb); BMI is 27 kg/m2. Blood pressure is 118/70 mm Hg. Examination shows no other abnormalities. Results of fasting serum studies are shown:
Glucose 82 mg/dl
Cholesterol, total 255 mg/dl
HDL-cholesterol 85 mg/dl
LDL-cholesterol 155 mg/dL
Triglycerides 70 mg/dl

The risk assessment score for her 10-year probability of coronary artery disease is 0.4%. The patient is counseled about maintaining a diet low in saturated fat and continuing her exercise regimen. Which
of the following is the most appropriate next step in management?

0 A) Cardiac stress scintigraphy


0 B) Daily aspirin therapy
0 C) Echocardiography
0 D) HMG-CoA reductase inhibitor therapy
0 E) No intervention is indicated at this time

https://t.me/USMLENBME2CK
ts e « t
Previous Next Lab Values Calculator Review Help pause
Exam Section 4: ltem 12 of 50 National Board of Medical Examiners Time Remaining.
an Comprehensive Clinical Science Self-Assessment 4 hr 59 min 31 sec

12. An investigator would like lo determine if a new medication for chronic obstructive pulmonary disease (COPD) decreases hospitalization rates in adults who come to the emergency department (ED)
because of an exacerbation. All patients with COPD who come to the ED because of an exacerbation will be invited lo participate in the study. Patients will be randomly assigned to receive the new
medication plus standard therapy or standard therapy alone during the hours that a study nurse is present (7 AM lo 7 PM daily). Patients who come to the ED outside of these hours will not be enrolled in
the study. Physicians treating the patients will be blinded to the group assignments. Outcomes will be assessed by a physician not involved in the care of the patients. Which of the following is the greatest
limitation of this study design?

0 A) Generalizability
0 B) Inadequate blinding of outcome assessors
0 C) lack of a placebo group
0 D) lead-time bias

https://t.me/USMLENBME2CK ti
Previous Next Lab Values Calculator Review Help pause
Exam Section 4: ltem 13 of 50 National Board of Medical Examiners Time Remaining.
an Comprehensive Clinical Science Self-Assessment 4 hr 59 min 30 sec

13. A57-year-old man is brought to the emergency department by his neighbors, who found him lying unconscious in a puddle of vomit in his apartment. He has a 20-year history of alcohol use disorder. On
arrival, he is lethargic and reports shortness of breath. He is unable to provide a medical history. His temperature is 39°C (102.2°F), pulse is 98/min, respirations are 28/min, and blood pressure is
132/76 mm Hg. Diffuse crackles are heard bilaterally. Arterial blood gas analysis on room air shows:
pH 7.44
Pco, 30 mm Hg
Po, 48 mm Hg

An x-ray of the chest shows pulmonary infiltrates bilaterally. Which of the following is the most appropriate next step in management?

0 A) Hyperbaric oxygen therapy


0 B) Intravenous administration of dobutamine
0 C) Intravenous administration of methylprednisolone
0 0) Bronchoscopy with lavage
0 E) Intubation and mechanical ventilation

https://t.me/USMLENBME2CK ti
Previous Next Lab Values Calculator Review Help pause
Exam Section 4: Item 14 of 50 National Board of Medical Examiners Time Remaining.
an Comprehensive Clinical Science Self-Assessment 4 hr 59 min 28 sec

14. A 37-year-old woman is scheduled for arthroscopic repair of a right medial meniscus tear. Her procedure is the surgeon's last scheduled operation of the day. All previous procedures this day involved the
left knee. In the operating room, the resident prepares the left knee for the operation. The operation is discontinued after the surgeon discovers no meniscus tear, and the patient is informed about the
error. Which of the following is the best approach to prevent future errors of this type?

0 A) Asking the surgeon to resign from the medical staff


0 B) Having the surgeon and resident present the case at the next morbidity and mortality conference
0 C) Having the surgeon mark the surgical site with input from the patient prior to induction of anesthesia
0 D) Having the surgeon review the chart just prior lo the procedure
0 E) Placing an incident report in the files of the resident and surgeon

https://t.me/USMLENBME2CK ti
Previous Next Lab Values Calculator Review Help pause
Exam Section 4: ltem 15 of 50 National Board of Medical Examiners Time Remaining.
an Comprehensive Clinical Science Self-Assessment 4 hr 59 min 26 sec

15. A 52-year-old woman comes to the office because of a 6-week history of moderate epigastric pain and an intermittent mild, midsternal burning sensation when she eats. She has had a 4.5-kg (10-lb)
weight loss during the past 6 months despite no change in appetite. During this time, she also occasionally has had dark stools. She has type 2 diabetes mellitus, hypertension, and osteoarthritis. Her
medications are metformin, hydrochlorothiazide, and ibuprofen. She has smoked one pack of cigarettes daily for 36 years. She appears malnourished. She is 163 cm (5 ft 4 in) tall and weighs 54 kg
(120 lb); BMI is 21 kg/m?. Her pulse is 86/min, and blood pressure is 110/65 mm Hg. Examination shows mild epigastric tenderness. There is no hepatosplenomegaly. The remainder of the examination
shows no abnormalities. Her hematocrit is 30%, leukocyte count is 6200/mm3, and platelet count is 530,000/mm3. Serum electrolyte concentrations are within the reference ranges. In addition to
discontinuing ibuprofen therapy, which of the following is the most appropriate next step in management?

0 A) Celecoxib therapy
0 B) Esophagogastroduodenoscopy
0 C) Follow-up examination in 6 weeks
0 D) Pantoprazole therapy and follow-up examination in 6 weeks
0 E) Serologic testing for He/icobacter pylori

https://t.me/USMLENBME2CK ti
Previous Next Lab Values Calculator Review Help pause
Exam Section 4: ltem 16 of 50 National Board of Medical Examiners Time Remaining.
an Comprehensive Clinical Science Self-Assessment 4 hr 59 min 23 sec

16. A67-year-old man comes to the physician because of a 6-monlh history of increasing difficulty walking. He has not had pain or numbness. He has no history of serious illness or recent infection.
Examination of the lower extremities shows atrophy; muscle strength is 3/5 bilaterally. Deep tendon reflexes are increased in the lower extremities. Sensation is intact. Which of the following is the most
likely diagnosis?

0 A) Amyotrophic lateral sclerosis


0 B) Multiple sclerosis
0 C) Myasthenia gravis
0 D) Polymyositis
0 E) Transverse myelitis

https://t.me/USMLENBME2CK ti
Previous Next Lab Values Calculator Review Help pause
Exam Section 4: ltem 17 of 50 National Board of Medical Examiners Time Remaining.
an Comprehensive Clinical Science Self-Assessment 4 hr 59 min 21 sec

17. A 14-year-old girl is brought to the physician because of an 18-hour history of persistent right-sided lower abdominal pain. She has not had fever, vomiting, or diarrhea. She has no history of serious illness
or operative procedures. Menarche was at the age of 12 years, and menses occur at regular 28-day intervals. Her last menstrual period was 2 weeks ago. She is not sexually active. Her temperature is
37°C (98.6°F), pulse is 72/min, respirations are 14/min, and blood pressure is 120/80 mm Hg. Abdominal examination shows mild tenderness with guarding over the right lower quadrant. There is no
distention, and bowel sounds are normal. The remainder of the examination shows no abnormalities. Her hemoglobin concentration is 13.9 g/dl, and leukocyte count is 8000/mm3with a normal differential.
Urinalysis shows no abnormalities. Abdominal ultrasonography shows a small amount of fluid in the pelvis. Which of the following is the most likely diagnosis?

0 A) Appendicitis
0 B) Ectopic pregnancy
0 C) Mittelschmerz
0 D) Ovarian torsion
0 E) Pelvic inflammatory disease

https://t.me/USMLENBME2CK ti
Previous Next Lab Values Calculator Review Help pause
Exam Section 4: ltem 18 of 50 National Board of Medical Examiners Time Remaining.
an Comprehensive Clinical Science Self-Assessment 4 hr 59 min 19 sec

18. A 62-year-old man, who underwent right below-the-knee amputation 36 years ago, comes lo the emergency department because of a 1-month history of an ulcer on the end of the residual limb. His
amputation was the result of injuries he sustained while serving in the military. He says his prosthetic socket has "felt loose and has been rubbing recently." During the past 6 months, he has had
progressive difficulty walking because of sharp pain that radiates up his leg. The pain has been occurring earlier during his walks; it slowly improves with rest. He previously walked 1 mile daily for exercise
and could walk several miles without stopping. He has not had vomiting, fever, chills, or malaise. He has type 2 diabetes mellilus treated with diet. One month ago, his hemoglobin A,, was 6%. Vital signs
are within normal limits. Examination of the right lower extremity shows mild erythema and a1x1-cm ulcer, with a depth of 3 mm, over the anteromedial aspect of the limb. There is viable soft tissue, and
margins are clear. The limb is warm. The popliteal pulse is 1 +. In addition to a recommendation for non-weight bearing and avoidance of direct pressure to the site, which of the following is the most
appropriate next step in management?

0 A) MRI of the right lower extremity


0 B) Recommendation for refitting of the prosthetic socket
0 C) Surgical revision of the amputation site
0 0) Wound culture and antibiotic therapy
0 E) X-ray of the right lower extremity

https://t.me/USMLENBME2CK ti
Previous Next Lab Values Calculator Review Help pause
Exam Section 4: ltem 19 of 50 National Board of Medical Examiners Time Remaining.
an Comprehensive Clinical Science Self-Assessment 4 hr 59 min 17 sec

19. A40-year-old man comes to the physician 6 days after noticing a non painful lump in his neck. He also has a 2-week history of palpitations. He is otherwise asymptomatic. He has no history of serious
illness and takes no medications. His temperature is 37.2C(99F), pulse is 110/min, respirations are 16/min, and blood pressure is 134/60 mm Hg. Examination shows a 1.4-cm, firm thyroid nodule. His
serum thyroid-stimulating hormone concentration is less than 0.05 mlU/l, and serum free thyroxine concentration is 3.5 ng/dL (N=0.9--1.7). Which of the following is the most appropriate next step in
management?

0 A) Fine-needle aspiration biopsy of the nodule


0 B) Measurement of serum calcitonin concentration
0 C) Operative resection of the nodule
0 D) Radioactive iodine uptake scan
0 E) Ultrasonography of the thyroid gland

https://t.me/USMLENBME2CK ti
Previous Next Lab Values Calculator Review Help pause
Exam Section 4: Item 19 of 50 National Board of Medical Examiners°
@ Mark Comprehensive Clinical Science Self-Assessment ease Wait

p ,mi
'

-~JI[_ V - I

t7ii'->
20. A previously healthy 24-year-old man comes to the emergency department because of a 12-hour history of increasing pain to the left of the center of his chest. Initially, the pain was mild and occurred only
when he breathed deeply. Now, the pain is more severe and constant. He rates the pain as an 8 on a 10-point scale. The pain increases when he lies on his back, decreases when he leans forward, and
does not radiate. He has not had palpitations, shortness of breath, nausea, vomiting, or sweating. He says that he is "recovering from a head cold," which he has had for 14 days. He appears moderately
uncomfortable but is in no respiratory distress. His temperature is 37.6°C (99.6°F), pulse is 66/min and regular, respirations are 20/min, and blood pressure is 152/88 mm Hg. Examination, including
cardiovascular examination, shows no abnormalities. His serum creatine kinase activity is 420 mg/dl with an MB fraction of 12% (N<5%). An ECG is shown. Which of the following is the most appropriate
next step in diagnosis?

A) Echocardiography
B) CT scan of the chest
C) Cardiac MRI
D) Coronary angiography
E) Pericardiocentesis

https://t.me/USMLENBME2CK ts e « t
Previous Next Lab Values Calculator Review Help pause
Exam Section 4: Item 21 of 50 National Board of Medical Examiners Time Remaining.
an Comprehensive Clinical Science Self-Assessment 4 hr 59 min 12 sec

21. A21-year-old man is brought to the emergency department by paramedics because of a 10-hour history of altered mental status. The mother says the patient was "fine" last night. This morning, he did not
wake up at his usual lime; when she went to check on him, she could not get him to wake up. He has no history of serious medical or psychiatric illness and takes no medications. His mother says he does
not smoke cigarettes, drink alcohol, or use illicit drugs. On arrival, he is lethargic but not in respiratory distress. He is responsive to painful stimuli. His temperature is 36.7°C (98.0F), pulse is 120/min,
respirations are 26/min, and blood pressure is 135/85 mm Hg. Pulse oximetry on room air shows an oxygen saturation of 100%. Physical examination shows no other abnormalities. Laboratory studies are
shown:
Serum
Na 143 mEq/L
K 3.8 mEq/L
Cl- 110 mEq/L
HCO- 3 13 mEq/L
Urea nitrogen 14 mg/dL
Glucose 120 mg/dL
Osmolality 290 mOsmol/kg HO
Urine
Glucose none
Ketones 3+

Arterial blood gas analysis is shown:


pH 7.43
Pco, 22 mm Hg
Po, 96mm Hg

Which of the following is the most likely diagnosis?

0 A) Alcoholic ketoacidosis
0 B) Diabetic ketoacidosis
0 C) Ethylene glycol toxicity
0 D) Methanol toxicity
0 E) Salicylate toxicity

https://t.me/USMLENBME2CK ti
Previous Next Lab Values Calculator Review Help pause
Exam Section 4: Item 22 of 50 National Board of Medical Examiners Time Remaining.
an Comprehensive Clinical Science Self-Assessment 4 hr 59 min 9 sec

22. A25-year-old man is brought to the emergency department 1 hour after sustaining a gunshot wound to the abdomen. He has pain at the bullet entry site. He is awake, alert, and appropriately conversant.
His temperature is 38.8°C (101.8F), pulse is 85/min, respirations are 18/min, and blood pressure is 128/70 mm Hg. Abdominal examination shows a bullet wound 5 cm to the right of the umbilicus. There
is moderate tenderness over the entry site. The remainder of the examination shows no abnormalities. During operative exploration, 250 ml of succus entericus and eight small bowel holes are found in
the abdomen; the bowel holes are treated by resection of 12 cm of proximal ileum. The intestine is reconstructed with a stapled anastomosis. The midline abdominal fascia is sutured closed. Which of the
following methods of skin and soft tissue wound management at this point is most likely to decrease the risk for postoperative wound infection?

0 A) Delayed primary closure


0 B) Hand-sewn suture closure of the skin
0 C) Use of skin staples
0 0) Application of cyanoacrylate adhesive to the wound
0 E) Split-thickness skin graft

https://t.me/USMLENBME2CK ti
Previous Next Lab Values Calculator Review Help pause
Exam Section 4: Item 23 0f 50 National Board of Medical Examiners@ Time Remaining:
i ark Comprehensive Clinical Science Self-Assessment 4hr59min7 sec

23. A 32-year-old woman comes to the physician because of a 2-week history of fatigue and increased urination. She has had a 2.3-kg (5-lb) weight loss despite eating more during this time. She has had two
urinary tract infections in the past. She takes no medications. She does not smoke cigarettes, drink alcohol, or use illicit drugs. The patient is alert and fully oriented. She is 165 cm (5 ft 5 in) tall and weighs
52 kg (115 lb); BMI is 19 kg/m2. Her temperature is 36.5"C (97.7°F), pulse is 100/min, respirations are 20/min, and blood pressure is 90/60 mm Hg. Pulse oximetry on room air shows an oxygen saturation
of 100%. The oropharynx is dry. The remainder of the examination shows no abnormalities. Laboratory studies show:
Serum
Na 142 mEq/L
K' 5.4 mEq/L
Cl 104 mEq/L
HCO, 16 mEq/L
Glucose 800 mg/dL
Creatinine 1.3 mg/dL
Urine
Specific gravity 1.030
Bili rub in none
Protein none
RSC 0--5/hpf
WBC 25-50/hpf
Nitrites positive
Leukocyte esterase positive
Glucose >1000 mg/24 h

Which of the following serum hormonal profiles is most consistent with this patient's presentation?

Insulin Glucagon Catecholamines


OA Decreased decreased decreased
0 8) Decreased increased decreased
Oo» Decreased decreased increased

OD Decreased increased increased

OE Increased increased decreased

OF Increased decreased increased

O6 Increased decreased decreased

OH Increased increased increased

https://t.me/USMLENBME2CK ts e « t
Previous Next Lab Values Calculator Review Help pause
Exam Section 4: Item 24 0f 50 National Board of Medical Examiners Time Remaining.
an Comprehensive Clinical Science Self-Assessment 4 hr 59 min 5 sec

24. A previously healthy 37-year-old woman comes to the physician because of a2-year history of her hands and fingers turning a bluish color with exposure lo cold. The discoloration has been occurring with
increasing frequency and now occurs with exposure to air conditioning and cold water. Examination of the hands shows no abnormalities. The brachia I and radial pulses are present bilaterally. When the
hands are rinsed in cold water, the fingers become bluish then bright red with rewarming. Which of the following is the most appropriate next step in management?

0 A) 13-Adrenergic blocking agent therapy


0 B) Angiotensin-converting enzyme (ACE) inhibitor therapy
0 C) Calcium-channel blocking agent therapy
0 D) Corticosteroid therapy
0 E) Dorsal sympathectomy

https://t.me/USMLENBME2CK ti
Previous Next Lab Values Calculator Review Help pause
Exam Section 4: Item 25 of 50 National Board of Medical Examiners Time Remaining.
an Comprehensive Clinical Science Self-Assessment 4 hr 59 min 3 sec

25. A 60-year-old man comes to the office for a routine examination. Medical history is remarkable for hypertension and alcohol use disorder. Medications are amlodipine and omeprazole. He has smoked one
pack of cigarettes daily for 40 years, and he drinks approximately twelve beers daily. Vital signs are within normal limits. Physical examination discloses no abnormalities. Results of serum studies are most
likely to show which of the following?

0 A) Hyperkalemia
0 B) Hypernatremia
0 C) Hypomagnesemia
0 D) Hypophosphatemia

https://t.me/USMLENBME2CK
ts e « t
Previous Next Lab Values Calculator Review Help pause
Exam Section 4: Item 26 of 50 National Board of Medical Examiners Time Remaining.
an Comprehensive Clinical Science Self-Assessment 4 hr 58 min 25 sec

26. An 87-year-old woman comes lo the physician because of a 3-week history of nausea and general malaise. She has hypertension. One month ago, her blood pressure was 160/95 mm Hg and
hydrochlorolhiazide therapy was begun. Her only other medication is 81-mg aspirin. She is 157 cm (5 fl 2 in) tall and weighs 50 kg (110 lb); BMI is 20 kg/m2. Today, her pulse is 78/min; her blood pressure
is 155/92 mm Hg in the right upper extremity and 153/90 mm Hg in the left upper extremity. The remainder of the examination shows no abnormalities. Serum studies show:
Na 125 mEq/L
K 3.2mEq/L
CI- 87 mEq/L
HCO, 25 mEq/L
Ca?+ 9.6mg/dL
Urea nitrogen 14 mg/dL
Glucose 107 mg/dL
Creatinine 0.6 mg/dl

Which of the following is the most appropriate next step in pharmacotherapy?

0 A) Add lisinopril to the regimen


0 8) Add losartan to the regimen
0 C) Add triamterene to the regimen
0 D) Begin potassium supplementation
0 E) Discontinue hydrochlorothiazide

https://t.me/USMLENBME2CK
ts e « t
Previous Next Lab Values Calculator Review Help pause
Exam Section 4: Item 27 of 50 National Board of Medical Examiners Time Remaining.
an Comprehensive Clinical Science Self-Assessment 4 hr 58 min 24 sec

27. A 15-year-old girl is brought to the office by her mother because of a 3-day history of pain with urination and an increasingly frequent and urgent need to urinate. She has had three episodes of urinary tract
infection during the past year. She has no other history of serious illness and currently takes no medications. The patient appears subdued and avoids eye contact. The physician asks the mother to leave
the office for the remainder of the interview, and she initially is reluctant. When she leaves the room, the physician asks the patient about her sexual history. The patient appears anxious and asks, "If I
answer these questions, will you promise not to tell my mom?" Which of the following is the most appropriate physician response?

0 A) Ask the patient why she is concerned about confidentiality


0 B) Empathize with the patient's desire to keep her sexual history private
0 C) Inform the patient that her information will be shared only if her safety or that of others is at risk
0 D) Tell the patient that it is worth risking her mother's disapproval to ensure proper treatment of her condition
0 E) Reassure the patient that the need to break confidentiality about her health is unlikely

https://t.me/USMLENBME2CK ti
Previous Next Lab Values Calculator Review Help pause
Exam Section 4: Item 28 of 50 National Board of Medical Examiners Time Remaining.
Comprehensive Clinical Science Self-Assessment 4 hr 58 min 22 sec

28. A 37-year-old man with a2-year history of intermittent right knee pain comes lo the office because of a 2-month history of increased knee pain that began after he collided with another person while skiing.
The pain has particularly worsened during the past 2 days and has made bearing weight on his right leg difficult. Immediately following the collision, he felt a sharp pain in the middle of his kneecap and
sustained several scrapes and cuts on his kneecap, which stopped bleeding within minutes but have continued to slowly ooze a clear discharge thereafter. Medical history otherwise is noncontributory. His
only medications are a daily multivitamin and occasional acetaminophen for pain. His temperature is 38°C (100.4°F), pulse is 76/min, and blood pressure is 110/60 mm Hg. Examination shows an
erythemalous right knee that is warm to touch. The patella is ballotable, and there is moderate joint effusion. Bending the right knee 10 to 15 degrees causes pain; the patient cannot tolerate bending it any
further. Which of the following is most likely diagnosis?

0 A) Anterior cruciate ligament tear


0 B) Meniscal tear
0 C) Patellar tendonitis
0 D) Pyogenic arthritis
0 E) Synovial chondromatosis

https://t.me/USMLENBME2CK
ts e « t
Previous Next Lab Values Calculator Review Help pause
Exam Section 4: Item 29 of 50 National Board of Medical Examiners@ Time Remaining:
i ark Comprehensive Clinical Science Self-Assessment 4hr58min 14 sec

29. A 15-year-old girl is brought to the office because of a 2-week history of a facial rash and fatigue. She has a 9-day history of stiffness and swelling of her fingers on awakening in the morning. She has no
history of serious illness and takes no medications. She appears tired and has mild chest pain with deep inspiration. Temperature is 38.0°C (100.4°F), pulse is 94/min and regular, respirations are
24/min, and blood pressure is 112/76 mm Hg. A photograph of the patient's face is shown. There are three nontender ulcers over the soft palate. Diffuse 1- to 2-cm lymph nodes are palpated in the
anterior and posterior cervical chains. The lungs are clear to auscultation. Heart sounds are normal. Examination of the extremities shows mild erythema, tenderness, and swelling of the proximal
interphalangeal and metacarpophalangeal joints, knees, and metatarsophalangeal joints. Urinalysis shows 2+ protein and 1 + blood. Which of the following is most appropriate in determining
management for this patient?

0 A) Cerebral angiography
0 B) Echocardiography
Oo MRI of the brain
0 0) Renal biopsy
0 E) X-rays of the knees

https://t.me/USMLENBME2CK ts e « t
Previous Next Lab Values Calculator Review Help pause
Exam Section 4: Item 30 of 50 National Board of Medical Examiners8 Time Remaining:
i ark Comprehensive Clinical Science Self-Assessment 4hr58min11 sec

30. A67-year-old woman comes to the physician because of a 6-monlh history of mild pain in her legs with walking and at rest. Her pain is somewhat relieved when she
elevates her legs. She has a3-year history of intermittent swelling of her legs. She has no history of serious illness and takes no medications. She has smoked
one-half pack of cigarettes daily for 18 years. She has been able to perform her normal daily activities. Cardiopulmonary examination shows no abnormalities. There is
non pitting edema of the lower extremities. A photograph of the right lower extremity is shown. Which of the following is the most appropriate next step in diagnosis of the
lower extremity findings?

0 A) Echocardiography
0 B) Lymphangiography
Q C) Skin biopsy
0 D) Angiography of the lower extremities
0 E) No further evaluation is indicated

https://t.me/USMLENBME2CK ts e « t
Previous Next Lab Values Calculator Review Help pause
Exam Section 4: ltem 31 of 50 National Board of Medical Examiners Time Remaining.
an Comprehensive Clinical Science Self-Assessment 4 hr 58 min 10 sec

31. Ten days after undergoing left above-the-knee amputation for severe diabetic soft-tissue infection, a 78-year-old man in the intensive care unit has noisy, labored breathing. His postoperative course has
been complicated by sepsis, delirium, and respiratory insufficiency. His temperature is 37.5°C (99.5°F), pulse is 72/min, respirations are 9/min, and blood pressure is 100/50 mm Hg. Pulse oximetry on 3L
of oxygen by nasal cannula shows an oxygen saturation of 92%. He is not oriented to person, place, or time. Breath sounds are coarse and distant bilaterally. The extremities are cool to the touch and
mottled. After discussions with the patient's family, and consistent with the patient's wishes, the family has chosen to provide comfort care. The family remains concerned about the patient's current loud
breathing and secretions. Which of the following is the most appropriate next step in management?

0 A) Administration of humidified oxygen


0 B) Administration of morphine
0 C) Administration of scopolamine
0 D) Intubation and mechanical ventilation
0 E) Placement of a nasogastric tube

https://t.me/USMLENBME2CK ti
Previous Next Lab Values Calculator Review Help pause
Exam Section 4: Item 32 0f 50 National Board of Medical Examiners@ Time Remaining:
i ark Comprehensive Clinical Science Self-Assessment 4hr58min8 sec

32. A previously healthy 4-year-old boy is brought to the emergency department by his mother on a winier evening because of the sudden onset of
coughing and mild difficulty breathing. His symptoms began late this afternoon when he was playing with other children at a party. His brother
has asthma. The patient's temperature is 37.2°C (99°F), pulse is 92/min, and respirations are 36/min. Physical examination shows intense
intermittent coughing and mild intercostal retractions. Wheezes are heard bilaterally but are greater on the right. An x-ray of the chest is shown.
Which of the following is the most likely diagnosis?

0 A) Asthma
0 B) Congenital lobar emphysema
0 C) Foreign body aspiration
0 D) Laryngotracheitis
0 E ) Pneumonia

https://t.me/USMLENBME2CK ts e « t
Previous Next Lab Values Calculator Review Help pause
Exam Section 4: ltem 33 of 50 National Board of Medical Examiners Time Remaining.
an Comprehensive Clinical Science Self-Assessment 4 hr 58 min 6 sec

33. A 62-year-old man comes to the office with his wife for a follow-up examination 4 weeks after he was hospitalized for management of an acute myocardial infarction that required cardiac catheterization. He
has had a 1.4-kg (3-lb) weight gain since discharge. He has not had shortness of breath or chest pain on exertion. He has been sleeping well. His medications are losartan and atorvastatin; he is
intermittently adherent to his regimen. He has had irregular attendance at an outpatient cardiac rehabilitation program; he says he sometimes skips sessions because he is tired. His wife says he is irritable
and often talks of his father who died of a heart attack at the age of 65 years. The couple has not resumed sexual activity because of the patient's fear of having another heart attack. The patient enjoys
seeing his grandchildren but is no longer interested in playing poker with his friends. He is 168 cm (5 fl 6 in) tall and weighs 85 kg (187 lb); BMI is 30 kg/m2. His pulse is 90/min, respirations are 16/min, and
blood pressure is 130/84 mm Hg. Physical examination shows no pedal edema. On mental status examination, he describes his mood as blah" and has a constricted affect. He reports no suicidal ideation
or suicide plan. Which of the following is the most likely diagnosis?

0 A) Adjustment disorder
0 B) Generalized anxiety disorder
0 C) Hypoactive sexual desire disorder
0 D) Major depressive disorder
0 E) Post-traumatic stress disorder

https://t.me/USMLENBME2CK
ts e « t
Previous Next Lab Values Calculator Review Help pause
Exam Section 4: Item 34 of 50 National Board of Medical Examiners Time Remaining.
an Comprehensive Clinical Science Self-Assessment 4 hr 58 min 5 sec

34. A 17-year-old primigravid patient at 22 weeks' gestation comes to the physician because of a 12-hour history of fever, nausea, and vomiting. She has abdominal tenderness on the right side when she
coughs. Her temperature is 38.3C (101F). Pelvic and rectal examinations show right-sided tenderness. Fetal heart tones are normal. Her leukocyte count is 18,000/mm? with a shift to the left. Her serum
amylase activity is within the reference range. During the next 8 hours, vomiting, fever, and abdominal pain continue. Ultrasonography of the gallbladder shows no abnormalities. Which of the following is
the most appropriate next step in diagnosis?

0 A) Barium enema
0 8) Gallium scan
0 C) laparoscopy
0 0) Peritoneal lavage
0 E) Amniocentesis

https://t.me/USMLENBME2CK ti
Previous Next Lab Values Calculator Review Help pause
Exam Section 4: ltem 35 of 50 National Board of Medical Examiners Time Remaining.
an Comprehensive Clinical Science Self-Assessment 4 hr 58 min 3 sec

35. An 18-year-old man comes to the clinic because of a 3-hour history of right eye pain with mild light sensitivity and blurry vision that have been present since he awakened this morning. He has had no
symptoms in his left eye. He reports being struck in the face by a soccer ball last night but having no symptoms at that lime. Medical history is remarkable for admission to the hospital 2 years ago for acute
chest syndrome secondary to sickle cell disease; he has taken hydroxyurea since that time. He takes no other medications and does not wear corrective lenses. Family history is remarkable for glaucoma
in his maternal grandfather. The patient's vital signs are within normal limits. Examination of the right eye shows blood filling the lower 20% of the anterior chamber. Pupils are equal and reactive to light
and accommodation bilaterally, and visual fields are intact. Visual acuity is 20/60 in the right eye and 20/20 in the left eye. lntraocular pressure is 27 mm Hg in the right eye and 18 mm Hg in the left eye
(N=12--22). Which of the following factors in this patient most strongly increases his risk for complications of his current condition?

O A) Age
0 B) Family history
0 C) Hydroxyurea use
0 D) Sickle cell disease

https://t.me/USMLENBME2CK
ts e « t
Previous Next Lab Values Calculator Review Help pause
Exam Section 4: ltem 36 of 50 National Board of Medical Examiners Time Remaining.
an Comprehensive Clinical Science Self-Assessment 4 hr 58 min 1 sec

36. A previously healthy 16-year-old boy comes to the physician because of constant right-sided scrotal pain for 36 hours. He had sexual intercourse with a new partner 1 week ago. His temperature is 38°C
(100.4F). Abdominal examination shows no abnormalities. There is a scant penile discharge. The right scrotum is swollen and tender, especially posteriorly. The pain is less severe when the scrotum is
elevated. Urinalysis and Gram stain of the urethral discharge show leukocytes. Which of the following is the most appropriate next step in management?

0 A) Semen analysis
0 B) Transillumination of the scrotum
0 C) Doppler ultrasonography
0 D) CT scan
0 E) Antibiotic therapy
0 F) Operative procedure

https://t.me/USMLENBME2CK ti
Previous Next Lab Values Calculator Review Help pause
Exam Section 4: Item 37 of 50 National Board of Medical Examiners@ Time Remaining:
i ark Comprehensive Clinical Science Self-Assessment 4hr57 min59 sec

37. A 52-year-old woman comes to the physician because of a 1-week history of increasingly severe pain and swelling of her right knee. She now is
unable to walk because of the symptoms. Five years ago, she fell and hit the knee on the bathroom floor. She has hypertension treated with
lisinopril. Her temperature is 36.9C(98.4F), pulse is 84/min, respirations are 20/min, and blood pressure is 132/76 mm Hg. The right knee is
erythematous, swollen, and warm. There is diffuse tenderness lo light touch; passive and active range of motion is limited by pain. There is no
crepitus, but ballottement shows moderate knee effusion. Arthrocentesis is performed, and joint fluid analysis shows a leukocyte count of
50,000/mm3. A Gram stain is negative; rhomboid-shaped crystals are present. An x-ray of the knee is shown. Which of the following is the most
appropriate pharmacotherapy?

0 A) Febuxostat
0 B) Ibuprofen
0 C) lnfliximab
0 0) Methotrexate
0 E) Penicillamine

https://t.me/USMLENBME2CK ts e « t
Previous Next Lab Values Calculator Review Help pause
Exam Section 4: ltem 38 of 50 National Board of Medical Examiners Time Remaining.
an Comprehensive Clinical Science Self-Assessment 4 hr 57 min 57 sec

38. A 28-year-old woman comes to the office for a follow-up examination 2 weeks after undergoing a total lhyroidectomy because of a thyroid nodule. Immediately after the operation, the patient had the onset
of marked hypocalcemia and was admitted to the intensive care unit. During her hospital stay, intravenous calcium and calcitriol were administered. Today, she has had recurrent numbness around her
mouth and muscle cramping. Current medications are calcium carbonate three times daily with meals and calcitriol twice daily. Vital signs are within normal limits. Physical examination shows spasm of the
ipsilateral facial muscles when tapping 2 cm anterior to the ears on the zygomatic arch bilaterally. The remainder of the examination shows no abnormalities. Laboratory studies are most likely to show an
increase in which of the following in this patient?

0 A) Serum calcium concentration


0 B) Serum magnesium concentration
0 C) Serum parathyroid hormone concentration
0 D) Serum parathyroid hormone-related protein concentration
0 E) Urinary 24-hour calcium excretion
0 F) Urine potassium excretion

https://t.me/USMLENBME2CK ti
Previous Next Lab Values Calculator Review Help pause
Exam Section 4: ltem 39 of 50 National Board of Medical Examiners Time Remaining.
an Comprehensive Clinical Science Self-Assessment 4 hr 57 min 56 sec

39. A previously healthy 27-year-old woman comes to the physician because of a rash over her chest and back for 5 days. She has a 10-day history of pain with urination, urinary frequency, and nocturia two
to three limes nightly. She took her father's trimethoprim-sulfamethoxazole for 7 days, and her urinary symptoms improved. She currently takes no medications. She does not smoke cigarettes. Her
temperature is 37.5°C (99.5°F), pulse is 110/min, respirations are 16/min, and blood pressure is 110/70 mm Hg. Examination of the skin shows an erythematous, maculopapular rash over the trunk. There
are no oral lesions. There is no splenomegaly. Laboratory studies show:
Hemoglobin 12.8 g/dL
Leukocyte count 3000/mm3
Segmented neutrophils 15%
Eosinophils 5%
Basophils 1%
Lymphocytes 54%
Monocytes 25%
Platelet count 200,000/mm3

Which of the following is the most appropriate next step in management?

0 A) Repeat examination in 1 week


0 B) Granulocyte colony-stimulating factor therapy
0 C) Prednisone therapy
( D) Vitamin B, (cyanocobalamin) supplementation
Q E) Skin biopsy

https://t.me/USMLENBME2CK
ts e « t
Previous Next Lab Values Calculator Review Help pause
Exam Section 4: Item 40 of 50 National Board of Medical Examiners Time Remaining.
an Comprehensive Clinical Science Self-Assessment 4 hr 57 min 54 sec

40. A57-year-old woman who was admitted to the hospital because of pneumonia has had an increase in her serum creatinine concentration from 1 mg/dl to 2.4 mg/dl during the past 3 days. On admission
7 days ago, she had severe shortness of breath and hypotension; she was intubated and mechanically ventilated for 2 days. Six days ago, a CT scan of the chest with contrast showed pneumonia at both
lung bases. She also has type 2 diabetes mellitus. Current medications are ceftriaxone, azithromycin, albuterol, insulin, and lisinopril. She is now breathing 3 L/min of oxygen via nasal cannula and has
mild shortness of breath. Her blood pressure has been stable for 6 days. Her urine output has been 1 L/24 h during the past 2 days. Her pulse is 70/min, and blood pressure is 150/90 mm Hg. Pulse
oximetry on 3 Umin of oxygen via nasal cannula shows an oxygen saturation of 94%. Crackles are heard at both lung bases. Urinalysis shows:
Protein 1+
RBC 10/hpf
WBC 9/hpf
Casts none

A Wright stain of urine shows eosinophils. Which of the following is the most appropriate next step in management?

0 A) Add acetylcysteine to the regimen


0 B) Add nifedipine to the regimen
0 C) Add prednisone to the regimen
0 D) Administer a 1-L bolus of 0.9% saline
0 E) Discontinue ceftriaxone
0 F) No intervention is indicated

https://t.me/USMLENBME2CK ti
Previous Next Lab Values Calculator Review Help pause
Exam Section 4: Item 41 of 50 National Board of Medical Examiners Time Remaining.
an Comprehensive Clinical Science Self-Assessment 4 hr 57 min 52 sec

41. A40-year-old man comes to the emergency department because of a1-day history of nausea and vomiting. He has not had fever, chills, abdominal pain, or diarrhea. He says taking a hot shower alleviates
his symptoms. He has no history of serious illness and takes no medications. He says he occasionally uses illicit drugs but refuses to provide any additional information. His temperature is 36.7°C (98°F),
pulse is 100/min, respirations are 18/min, and blood pressure is 130/90 mm Hg. Examination shows no other abnormalities. Intoxication from which of the following is the most likely cause of this patient's
current condition?

0 A) Cannabis
0 B) Cocaine
0 C) Heroin
Oo LSD
0 E) Methamphetamine
OF PCP (phencyclidine)

https://t.me/USMLENBME2CK ti
Previous Next Lab Values Calculator Review Help pause
Exam Section 4: Item 42 of 50 National Board of Medical Examiners Time Remaining.
an Comprehensive Clinical Science Self-Assessment 4 hr 57 min 50 sec

42. A 62-year-old woman comes to the physician because of a 4-month history of a painless mass in her left breast; the mass has not changed in size. Ten years ago, she had a total abdominal hysterectomy
and bilateral salpingo-oophorectomy. She has hypertension treated with hydrochlorothiazide. Her maternal aunt was diagnosed with breast cancer at the age of 55 years. The patient's vital signs are within
normal limits. Examination shows a 2.5-cm hard, immobile mass in the upper outer quadrant of the left breast. Mammography and fine-needle aspiration biopsy specimen show no abnormalities. Which of
the following is the most appropriate next step in management?

0 A) Follow-up examination in 3 months


0 B) Repeat mammography in 6 months
0 C) MRI of the left breast
0 D) Tamoxifen therapy
0 E) Excisional biopsy

https://t.me/USMLENBME2CK ti e
Previous Next Lab Values Calculator Review Help pause
Exam Section 4: Item 43 of 50 National Board of Medical Examiners Time Remaining.
an Comprehensive Clinical Science Self-Assessment 4 hr 57 min 42 sec

43. A 2-year-old girl is brought to the office by her mother because of bloody drainage from her right ear that the mother first noticed last night when the patient woke up screaming and holding her right ear.
The pain improved after the mother cleaned the ear with a cotton swab and gave the patient ibuprofen, but the mother noted dried, crusted blood on the patient's right ear lobe and pillow when she awoke
this morning. The patient also has a 3-day history of cough, congestion, and runny nose. She has no history of serious illness and receives no routine medications. She is at the 25th percentile for height
and weight and 50th percentile for BMI. Vital signs are within normal limits. The right tympanic membrane cannot be visualized because of seropurulent fluid in the right external auditory canal. The
remainder of the examination, including examination of the left tympanic membrane, discloses no abnormalities. Which of the following is the most likely cause of this patient's ear drainage?

0 A) Basilar skull fracture


0 B) Mastoiditis
0 C) Otitis externa
( D) Traumatic injury to the external auditory canal

0 E) Tympanic membrane perforation

https://t.me/USMLENBME2CK ti
Previous Next Lab Values Calculator Review Help pause
Exam Section 4: Item 44 of 50 National Board of Medical Examiners Time Remaining.
an Comprehensive Clinical Science Self-Assessment 4 hr 57 min 40 sec

44. A 20-year-old man comes to the student health services on his college campus because of painful urination and scant urethral discharge for the past week. He says that he has had no fever, chills, or back
pain. For the past 6 months, he has engaged in unprotected sexual activity with one partner. He has no history of sexually transmitted diseases or other disorders and takes no medications. He appears
well. His temperature is 37C (98.6F), pulse is 64/min, respirations are 12/min, and blood pressure is 115/70 mm Hg. Examination shows no costovertebral angle tenderness, penile lesions, or expressible
discharge. Urinalysis shows no RBCs, 2-5 WBC/hpf, and no casts; throat cultures are pending. Which of the following is the most appropriate management at this time?

0 A) Amoxicillin therapy
0 B) Ceflriaxone and azithromycin therapy
0 C) Metronidazole therapy
0 D) Trimethoprim-sulfamethoxazole therapy
0 E) No treatment until culture and sensitivity test results are known

https://t.me/USMLENBME2CK ti
Previous Next Lab Values Calculator Review Help pause
Exam Section 4: Item 45 of 50 National Board of Medical Examiners Time Remaining.
an Comprehensive Clinical Science Self-Assessment 4 hr 57 min 38 sec

45. A 62-year-old woman comes to the physician for PPD skin testing prior lo beginning volunteer work in a school. She has hypertension treated with hydrochlorothiazide and osteoarthritis treated with
acetaminophen. She was born in the USA. She has had no sick contacts. She does not smoke cigarettes, drink alcohol, or use illicit drugs. Physical examination shows no abnormalities. A PPD skin test
produces 6 mm of induralion at 48 hours. Results of liver function tests are within the reference ranges. A chest x-ray shows no abnormalities. Which of the following is the most appropriate next step in
management?

0 A) Provide clearance to begin volunteer work


0 B) Repeat PPD skin testing to assess for booster phenomenon
0 C) Obtain culture of three induced sputum samples for acid-fast bacilli
0 0) Begin isoniazid and pyridoxine therapy
0 E) Begin isoniazid, rifampin, pyrazinamide, and ethambutol therapy
0 F) Begin isoniazid therapy only

https://t.me/USMLENBME2CK ti
Previous Next Lab Values Calculator Review Help pause
Exam Section 4: Item 46 of 50 National Board of Medical Examiners Time Remaining.
an Comprehensive Clinical Science Self-Assessment 4 hr 57 min 36 sec

46. A42-year-old man comes to the emergency department because of a 2-day history of nausea, vomiting, and malaise. He has bipolar disorder, herpes simplex virus 2, and chronic shoulder pain secondary
to a torn rotator cuff injury sustained 7 years ago. His medications are valproic acid, sertraline, acyclovir, hydrocodone-acetaminophen, and docusate. He does not drink alcohol or use illicit drugs. His
temperature is 37.2°C (99°F), pulse is 102/min, and blood pressure is 122/84 mm Hg. Physical examination shows dry mucous membranes. Abdominal examination shows tenderness to palpation. On
mental status examination, he has a dysphoric mood and congruent affect. Laboratory studies show:
Leukocyte count 8000/mm3
Serum
AST 38 U/L
ALT 52 U/L
Amylase 465 U/L

Which of the following medications is the most likely cause of this patient's current symptoms?

0 A) Acyclovir
0 B) Docusate
0 C) Hydrocodone-acetaminophen
0 D) Sertraline
0 E) Valproic acid

https://t.me/USMLENBME2CK ti
Previous Next Lab Values Calculator Review Help pause
Exam Section 4: Item 47 of 50 National Board of Medical Examiners Time Remaining.
an Comprehensive Clinical Science Self-Assessment 4 hr 57 min 34 sec

47. A 72-year-old woman comes to the office with her son for a follow-up examination. She was diagnosed with Parkinson disease 8 years ago. She says her health has declined substantially during the past
6 months. She feels weaker and has had increasing difficulty walking. She has stopped attending weekly luncheons with her friends. The patient's son says his mother sleeps poorly at night and naps
frequently during the day. He has to encourage her to eat and take her medications. The patient's medications are carbidopa-levodopa and donepezil. She appears sad and is tearful as she discusses her
difficulties. Examination shows a mild resting tremor of the hands and mild bradykinesia. When asked to stand and walk from a seated position, she moves slowly and pushes on the arms of the chair with
her hands; she takes steps with no hesitation but walks slowly. Which of the following is the most appropriate next step in pharmacotherapy?

0 A) Add mirtazapine to the medication regimen


0 B) Add rasagiline to the medication regimen
0 C) Add zolpidem to the medication regimen
0 D) Increase the dosage of carbidopa-levodopa
0 E) Increase the dosage of donepezil

https://t.me/USMLENBME2CK ti
Previous Next Lab Values Calculator Review Help pause
Exam Section 4: Item 48 of 50 National Board of Medical Examiners Time Remaining.
an Comprehensive Clinical Science Self-Assessment 4 hr 57 min 32 sec

48. A 32-year-old primigravid woman at 10 weeks' gestation comes to the physician for her first prenatal visit. She conceived after in vitro fertilization. Three years ago, a bicornuate uterus was noted on
diagnostic laparoscopy for primary infertility. She is otherwise healthy, and her only medication is a prenatal vitamin. She is 163 cm (5 fl 4 in) tall. She weighed 50 kg (110 lb) prior to her pregnancy; BMI
was 19 kg/m?. She has had a 1-kg (2-lb) weight gain during her pregnancy. Her blood pressure is 110/70 mm Hg. Examination shows a uterus consistent in size with a 10-week gestation. This patient is at
greatest risk for which of the following pregnancy complications?

0 A) Abruptio placentae
0 8) Placenta accreta
0 C) Placenta previa
0 D) Preterm delivery
0 E) Uterine rupture

https://t.me/USMLENBME2CK
ts e « t
Previous Next Lab Values Calculator Review Help pause
Exam Section 4: Item 49 of 50 National Board of Medical Examiners Time Remaining.
an Comprehensive Clinical Science Self-Assessment 4 hr 57 min 29 sec

49. A22-year-old man with alcohol use disorder comes to the physician because of a 3-week history of weakness in his right hand. His symptom began after drinking a bottle of whiskey. He has no history of
serious illness and takes no medications. He drinks six cans of beer daily. On examination of the right upper extremity, muscle strength is 2/5 in the supinator, wrist extensors, and finger extensors; strength
is 5/5 in all other muscles. Sensation to pinprick is decreased over the dorsum of the right hand from the wrist to the base of the thumb and index finger. Which of the following is the most appropriate next
step in diagnosis?

0 A) Determination of erythrocyte sedimentation rate


0 B) Electromyography and nerve conduction studies
0 C) MRI of the brachia! plexus
Oo MRI of the brain
0 E) MRI of the cervical spine

https://t.me/USMLENBME2CK ti
Previous Next Lab Values Calculator Review Help pause
Exam Section 4: Item 50 of 50 National Board of Medical Examiners Time Remaining.
an Comprehensive Clinical Science Self-Assessment 4 hr 57 min 27 sec

50. One day after delivery of a healthy newborn at term, a 22-year-old woman, gravida 2, para 2, has fever. Pregnancy and delivery were uncomplicated. Her temperature is 39.4C (103F), pulse is 110/min,
respirations are 22/min, and blood pressure is 125/70 mm Hg. Examination shows normal lochia rubra. The uterus is tender, and the fund us extends to the umbilicus. Her leukocyte count is 17 ,000/mm3
with a shift to the left. While awaiting results of additional laboratory studies, which of the following is the most likely causal organism?

0 A) Candida albicans
0 B) Escherichia coli
0 C) Mycop/asma pneumoniae
0 D) Staphylococcus aureus
0 E) Staphylococcus epidermidis

https://t.me/USMLENBME2CK ti
Previous Next Lab Values Calculator Review Help pause

You might also like